You are on page 1of 104

PRE-BOARD EXAMINATION IN CLINICAL CHEMISTRY (PART 1)

1. All of the following are correct match, EXCEPT:


Conversion of Conventional Unit to SI Unit:
Conventional Unit SI Unit
A Cholesterol 150 mg/dL 3.88 mmol/L
B Triglyceride 10 mg/dL 0.11 mmol/L
C Glucose 70 mg/dL 3.9 mmol/L
D BUN 8 mg/dL 0.47 mmol/L
2. A cholesterol quality control chart has the following data for the normal control:
Mean o f the data (X) = 150 mg/dL
2 Standard deviation = 4
X = 1,372 mg/dL
N = 10
Determine the coefficient of variation for this control:
A. 1.33% B. 2.67% C. 15.0 % D. 7.06%
3. The extent to which measurements agree with the true value of the quantity being measured is known as:
A. Acceptable limits B. Accuracy C. Precision D. Reliability
4. The reproducibility of test measurements is referred to as:
A. Accuracy B. Precision C. Quality control D. Reliability
5. In a Gaussian curve distribution, the 2 SD range includes the following percentage of values:
A. 31.6% B. 68.3% C. 95.5% D. 99.7%
6. Which Westgard rule detects random error?
A. 13s B. 41s C. 22s D. 100
7. The component of a spectrophotometer that is responsible for detecting transmitted light and converting light energy
to electrical energy is the:
A. Detector B. Analytical cell C. Monochromator D. Readout device
8. The process by which fluorescence of an analyte is reduced due to the excited molecule losing some of its energy by
interacting with other substances in a solution is known as:
A. Ionization B. Quenching C. Phosphorescence D. Self-absorption
9. In flow cytometry, the side scatter is related to the:
A. DNA content of the cell C. Size of the cell
B. Granularity of the cell D. Number of cells in G0 and G1
10. Which of the following approaches of automated analyzers uses a rotor to mix reagents?
A. Centrifugal analysis C. Discrete analysis
B. Continuous flow D. Dry chemistry slide analysis
11. Which of the following chemistry analyzers uses “slides” to conatin the entire reagent system?
A. Vitros analyzers C. Paramax analyzers
B. ACA analyzers D. None of the above
12. Ceruloplasmin and haptoglobin migrate electrophoretically as:
A. Alpha1 globulins B. Alpha2 globulins C. Beta globulins D. Gamma globulins
13. In cirrhosis, a predominant characteristic observed in electrophoretic serum pattern is a(an):
A. Increase in alpha2 globulin fraction C. Monoclonal band in gamma globulin region
B. Bridging effect between beta and gamma globulin fraction D. Polyclonal band in gamma globulin region
14. A congenital disorder characterized by a split in the albumin band when serum is subjected to electrophoresis is
known as:
A. Analbuminemia C. Bisalbuminemia
B. Anodic albuminemia D. Prealbuminemia
15. Which enzymes are generally quantitated to help diagnose acute pancreatitis?
A. ALT and CK C. Amylase and lipase
B. ALT and LD D. ALP and cholinesterase
16. Serum levels of total CK are elevated after an acute myocardial infarction and in which of the following conditions?
A. Diabetes mellitus C. Duchenne type muscular dystrophy
B. Chronic renal failure D. Early pregnancy
17. The isoenzymes LD-4 and LD-5 are elevated in:
A. Pulmonary embolism C. Renal disease
B. Liver disease D. Myocardial infarction
18. Which of the following liver enzymes is the best to use to evaluate liver function?
A. AST B. GGT C. ALT D. LD
19. In an adult, a normal GGT level with an elevated serum ALP level may be suggestive of:
A. Bone disease C. Liver disease
B. Heart disease D. Skeletal muscle disease
20. A decreased LDH/HBD ratio is significant for:
A. Myocardial infraction C. Both of these
B. Liver disease D. None of these
21. Isoenzyme of ALP that is most heat stable:
A. Placental ALP C. Intestinal ALP
B. Bone ALP D. Liver ALP
22. Which of the following enzymes exhibit a decreased serum level in liver disease?
A. Alanine aminotransferase D. Cholinesterase
B. Gamma glutamyltransferase E. All of the above
C. Lactate dehydrogenase
2
23. The relative concentration of LD isoenzymes in normal serum:
A. LD 1>2>3>4>5 C. LD 2>1>4>3>5
B. LD 2>1>3>4>5 D. LD 5>4>3>2>1
24. Substrate exhibiting high specificity for prostatic acid phosphatase:
A. Beta-glycerophosphate (Bodansky)
B. Phenylphosphate (Gutman, King-Armstrong)
C. Thymolphthalein phosphate (Roy)
D. Alpha-naphthylphosphate (Babson and Reed)
25. Time course of CK activity in myocardial infarction:
Onset of Elevation Peak Activity Duration of Elevation
A 1-3 h 5-12 h 18-30 h
B 4-8 h 12 h 3-4 d
C 8-12 h 24 h 5d
D 12-24 h 72 h 10 d
26. Serum enzymes typically elevated due to skeletal muscle disease:
A. Aspartate aminotransferase, Creatine kinase, Lactate dehydrogenase
B. Aspartate aminotransferase, Alanine aminotransferase, Alkaline phosphatase
C. Aldolase, Pseudocholinesterase, Trypsin
D. 5'-Nucleotidase, Alkaline phosphatase, Gamma-glutamyl transferase
27. Most sensitive enzyme index of primary muscle involvement:
A. Aldolase B. AST C. CK D. LD
28. In pesticide poisoning, cholinesterase activity is:
A. Normal B. Decreased C. Increased D. Variable
29. Glucose oxidase oxidizes glucose to gluconic acid and:
A. H2O2 B. CO2 C. HCO3 D. H2O
30. Hexokinase catalyzes the formation of --- from glucose and ATP:
A. Acetyl CoA C. Glucose 6-phosphate
B. Fructose 6-phosphate D. Lactose
31. Select the enzyme that is most specific for -D-glucose:
A. Glucose oxidase C. Hexokinase
B. Glucose-6-phosphate dehydrogenase D. Phophohexoisomerase
32. Select the coupling enzyme used in the hexokinase method for glucose:
A. Glucose dehydrogenase C. Glucose-6-phosphate dehydrogenase
B. Glucose-6-phosphatase D. Peroxidase
33. Which of the following methods provides information about glucose metabolism over a 3 month period?
A. Fructosamine C. Glycosylated hemoglobin
B. Glucose oxidase D. Oral glucose tolerance
34. Measurement of glycosylated albumin, which reflects hyperglycemic state within the previous 2 – 3 weeks is:
A. Glucosamine B. Fructosamine C. Urine glucose D. Galactosamine
35. Type I DM maybe described by all of the following, EXCEPT:
A. Insulin dependent B. Insulinopenia C. Ketosis prone D. Adult-onset type
36. Which of the following methods estimates urine sugar concentration by measuring total reducing substances?
A. Copper sulfate C. Hexokinase
B. Glucose oxidase D. Ferricyanide
37. A patient with severe, uncontrolled case of diabetes mellitus exhibits all of the following conditions EXCEPT:
A. Glycosuria C. Ketosis
B. Hyperglycemia D. Metabolic alkalosis
38. Which NPN fraction constitutes nearly half of the NPN substances in the blood?
A. Urea C. Ammonia
B. Creatine D. Uric acid
39. In the Jaffe reaction, a red-orange chromogen is formed when creatinine reacts with:
A. Picric acid C. Diacetyl monoxime
B. Naphthylethylenediamine D. Nitroferricyanide
40. A BUN of 9 mg/dL is obtained by a technologist. What is the urea concentration?
A. 18.3 mg/dL B. 19.3 mg/dL C. 10.3 mg/dL D. 9.3 mg/dL
41. BUN/creatinine ratio in serum in renal disease:
A. >20:1 B. <10:1 C. 10:1 D. Variable
42. Uric acid is the final breakdown product of:
A. Urea metabolism C. Glucose metabolism
B. Purine metabolism D. Bilirubin metabolism
43. The bilirubin fraction that is covalently attached to albumin and contributes to the conjugated bilirubin value is:
A. Delta B. Direct C. Indirect D. Bound
44. In which of the following conditions does decreased activity of glucuronyl transferase result in increased unconjugated
bilirubin and kernicterus in neonates?
A. Gilbert’s disease C. Dubin-Johnson syndrome
B. Rotor’s syndrome D. Crigler-Najjar syndrome
45. A decrease in which of the following indicates increased intravascular hemolysis?
A. Methemoglobin C. Haptoglobin
B. Methemalbumin D. Hemopexin
46. Major carrier of liver synthesized triglycerides:
A. Chylomicrons B. HDL C. LDL D. VLDL
47. The lipoprotein that transports the exogenous triglycerides:
A. HDL B. LDL C. VLDL D. Chylomicrons
3
48. “Floating beta lipoprotein” refers to:
A. B-VLDL B. HDL C. VLDL D. B-LDL
49. An abnormal lipoprotein found in patients with obstructive biliary disease:
A. B-VLDL B. LpX C. Lp(a) D. LDL
50. Which of the following is referred to as the “good cholesterol”?
A. HDL B. LDL C. VLDL D. Free cholesterol
51. Chemical composition of HDL:
Protein (%) Cholesterol (%) Triglyceride (%) Phospholipid (%)
A 1-2 1-3 80-95 3-6
B 6-10 4-8 45-65 15-20
C 18-22 6-8 4-8 18-24
D 45-55 3-5 2-7 26-32
52. Select the order of mobility of lipoproteins electrophoresed on cellulose acetate or agarose at pH 8.6.
A. – Chylomicrons  prebeta  beta  alpha + C. – Chylomicrons beta  prebeta  alpha +
B. – Beta  prebeta  alpha chylomicrons + D. – Alpha  beta  prebeta  chylomicrons +
53. Which phenotype in the Frederickson classification of hyperlipoproteinemia is characterized by an increase in
cholesterol, an increase in beta lipoproteins, normal triglycerides, and the absence of chylomicrons?
A. I B. II C. III D. IV
54. Apolipoprotein A is the primary protein component of:
A. HDL B. LDL C. VLDL D. None of these
55. Apolipoprotein affected in Tangier’s disease:
A. Apo A1 B. Apo B100 C. Apo B48 D. Apo (a)
56. A fasting serum sample from an asymptomatic 43-year-old woman is examined visually and chemically with the
following results:
Initial appearance of serum: Milky Triglyceride level : 2000 mg/dL Cholesterol level : 550 mg/dL
Appearance of serum after overnight refrigeration: Cream layer over turbid serum
This sample contains predominantly:
A. Chylomicrons alone B. chylomicrons and VLDL C. VLDL and LDL D. HDL
57. In lipoprotein phenotyping, chylomicrons are present in the plasma of persons with which of the following lipoprotein
phenotypes?
A. I and IIa B. I and IIb C. II and III D. I and V
58. Which is the major intracellular cation?
A. Calcium B. Magnesium C. Sodium D. Potassium
59. Which is the major extracellular cation?
A. Chloride B. Sodium C. Magnesium D. Calcium
60. In cystic fibrosis, which sweat electrolytes are elevated?
A. Calcium and chloride C. Sodium and potassium
B. Chloride and sodium D. Potassium and chloride
61. Hyponatremia may be caused by each of the following EXCEPT:
A. Hypomagnesemia C. Prolonged vomiting or diarrhea
B. Aldosterone deficiency D. Acute or chronic renal failure
62. Hypokalemia may be caused by each of the following EXCEPT:
A. Acidosis C. Hypomagnesemia
B. Prolonged vomiting or diarrhea D. Hyperaldosteronism
63. A hemolyzed sample will cause falsely increased levels of each of the following EXCEPT:
A. Potassium C. Phosphate
B. Sodium D. Magnesium
64. Fiske-Subbarow reaction is used in the determination of:
A. Proteins C. Inorganic phosphorus
B. Magnesium D. Calcium
65. When measuring potassium using ion-selecetive electrode, the membrane is composed of:
A. Glass B. Plastic C. Silver D. Valinomycin
66. The total iron binding capacity (TIBC) test is performed to assess the blood level of:
A. Ferritin B. Hemopexin C. Iron D. Transferrin
67. Which protein is produced in decreased amount in Wilson’s disease?
A. Albumin B. Ceruloplasmin C. Haptoglobin D. Hemopexin
68. Buffer system/s in the body:
1. Bicarbonate/carbonic acid 3. Plasma proteins
2. Hemoglobin 4. Plasma phosphates
A. 1 and 2 B. 1, 2 and 3 C. 1, 3 and 4 D. 1, 2, 3 and 4
69. In health, when the kidneys and lungs are functioning properly, a 20:1 ratio of HCO 3- to H2CO3 will be maintained,
resulting in a pH of:
A. 7.10 B. 7.30 C. 7.40 D. 7.50
70. The normal ratio of carbonic acid to bicarbonate in arterial blood is:
A. 7.4:6.1 B. 1:20 C. 0.003:1.39 D. 20:1
71. When arterial blood from a “normal patient is exposed to room air, the following happens:
A. PCO2 decreases, PO2 increases C. PCO2 decreases, PO2 decreases
B. PCO2 increases, PO2 decreases D. PCO2 increases, PO2 increases
72. Which of the following is the primary mechanism of compensation for metabolic acidosis?
A. Hyperventilation C. Aldosterone release
B. Hypoventilation D. Bicarbonate excretion
4
73. Severe diarrhea causes:
A. Metabolic acidosis B. Metabolic alkalosis C. Respiratory acidosis D. Respiratory alkalosis
74. In the patient’s circulatory system, bicarbonate leaves the red blood cell and enters the plasma through an exchange
mechanism with:
A. Carbonic acid B. Lactate C. Chloride D. Sodium
75. TSH is produced in the:
A. Hypothalamus B. Pituitary gland C. Adrenal cortex D. Thyroid
76. The major carrier protein of T3 and T4 in the circulation is:
A. Albumin B. Thyroglobulin C. TBG D. Thyroxine-binding prealbumin
77. T3 uptake test os used to quantitate the concentration of:
A. Free T3 B. Protein-bound T3 C. Total T3 D. TBG
78. Thyroid function test in primary hypothyroidism:
T3 T4 TSH
A Decreased Decreased Decreased
B Decreased Decreased Increased
C Normal Increased Decreased
D Normal Normal Normal
79. A patient’s T4 is 10 mg/dL. The THBR is 24% (normal=30%). What is the FT4I?
A. 8 B. 6 C. 4 D. 2
80. All of the following are correct match, EXCEPT:
Hormone classification by structure:
A. Peptide: Insulin, TSH, FSH C. Amino acids: Nor/Epinephrine,
B. Steroid: T4, T3 D. Fatty acids: Prostaglandins
81. All of the following are correct match, EXCEPT:
Tissues and the Hormones they produce:
A. Pancreas: Insulin, Glucagon C. Adrenal medulla: Epinephrine
B. Adrenal cortex: Cortisol D. Thyroid: TSH
82. The principal estrogen produced during pregnancy is:
A. Progesterone B. Estradiol C. Estrone D. Estriol
83. The Kober reaction is used in the assay of:
A. Urinary estrogen B. Glucocorticoids C. Testosterone D. Epinephrine
84. In which of the following would gastrin levels be increased:
A. Peptic ulceration C. Achlorhydria
B. Zollinger-Ellison syndrome D. Amyloidosis
85. Which of the following may be classified as an end product of catecholamine metabolism?
A. Dopamine C. Metanephrine
B. Normetanephrine D. Vanillylmandelic acid
86. Which of the following polypeptide hormones may be described as having alpha chains that are biochemically
identical but beta chains that as biochemically unique?
A. FSH, TSH, ACTH, TRH C. TSH, LH, TRH, HCG
B. LH, ACTH, HCG, TRH D. HCG, FSH, TSH, LH
87. The assay employed for 17-ketosteroids, in which steroids react with m-dinitrobenzene in alcoholic KOH solution
resulting to the formation of a purple color:
A. Kober Reaction C. Porter-Silber Reaction
B. Zimmerman Reaction D. Pisano Method
88. A marked increase in 5-HIAA excretion occurs in patients with:
A. Argentaffinoma C. Diabetes insipidus
B. Pheochromocytoma D. Diabetes mellitus
89. Which of the following drugs would be correctly classified as antiepileptic?
A. Digoxin D. Phenytoin
B. Disopyramide E. Tacrolimus
C. Chloramphenicol
90. Digoxin, procainamide, and quinidine are drugs that may be classified as:
A. Aminoglycosides C. Antideppresants
B. Anticonvulsants D. Cardioactive
91. The medication of choice for treatment of manic-depression is:
A. Carbamazepine C. Phenobarbital
B. Lithium carbonate D. Phenytoin
92. Which of the following drugs may be quantitated colorimetrically using Trinder’s reagent (mercuric chloride, ferric
nitrate and hydrochloric acid)?
A. Acetaminophen B. Phenobarbital C. Salicylate D. Theophylline
93. Cocaine is metabolized to:
A. Carbamazepine B. Codeine C. Hydrocodone D. Benzoylecgonine
94. Four children are admitted with malaise, anorexia, and abdominal pain. Further evaluations reveal mild anemia,
erythrocyte basophilic stippling, and profound pica habits. Poisoning by which heavy metal is most likely responsible?
A. Arsenic B. Iron C. Mercury D. Lead
95. Caffeine is an important metabolite of this drug:
A. Acetaminophen B. Digoxin C. Theophylline D. Phenobarbital
96. Odor of bitter almond:
A. Cyanide poisoning B. Ethanol poisoning C. Arsenic poisoning D. CO poisoning
97. The system of choice for drug analysis because of its specificity and sensitivity is:
A. HPLC B. TLC C. GC/mass spectrometry D. None of these
5

98. The formation of this crystal in urine, although not a constant finding is an important diagnostic clue of ethylene glycol
poisoning:
A. Uric acid B. Ammonium biurate C. Triple phosphate D. Calcium oxalate
99. This common substance of abuse is derived from Cannabis sativa leaves and stems. Which of the following is it?
A. Heroine B. Cocaine C. Marijuana D. Amphetamines
100. Condition characterized by an increased gastrin levels:
A. Peptic ulcer disease C. Achlorhydria
B. Zollinger-Ellison syndrome D. Amyloidosis
101. All of the following are correct match, EXCEPT:
A. Vitamin A: Retinol C. Vitamin C: Ascorbic acid
B. Vitamin B2: Cyanocobalamin D. Vitamin K: Menaquinone
102. Which type of extinguisher should put out a fire caused by electrical equipment?
Type of Extinguisher
A Pressurized water/Dry chemical
B Dry chemical/Carbon dioxide
C Carbon dioxide/Halon/Dry chemical
D Metal X/Special dry chemical
103. Specifications set by CAP define three grades of water. Type 1 reagent water is used for:
1. Preparation of standard solutions 3. Tissue or cell culture methods
2. Ultramicrochemical analyses 4. Washing glasswares
A. 1 and 3 B. 2 and 4 C. 1, 2 and 3 D. 1, 2, 3 and 4
104. Calculate the dilution if 5 mL of serum is diluted with 15 mL of saline solution.
A. 1:2 B. 1:3 C. 1:4 D. 1:5
105. How much 20% alcohol is required to make 1 liter of 10% alcohol?
A. 0.4 L B. 0.5 L C. 0.8 L D. 1.0 L

END OF THE EXAM


PRE-BOARD EXAMINATION IN CLINICAL CHEMISTRY (PART 2)

1. The extent to which measurements agree with the true value of the quantity being measured is known as:
A. Acceptable limits B. Accuracy C. Precision D. Reliability
2. The reproducibility of test measurements is referred to as:
A. Accuracy B. Precision C. Quality control D. Reliability
3. In a Gaussian curve distribution, the 2 SD range includes the following percentage of values:
A. 31.6% B. 68.3% C. 95.5% D. 99.7%
4. Given the following data, calculate the coefficient of variation for glucose.
Analyte Mean Standard deviation
Glucose 76 mg/dL 2.3
A. 3.0% B. 4.6 % C. 7.6% D. 33.0%
5. A cholesterol QC chart has the following data for the normal control:
x = mean of data
x = 137 mg/dL x = 1,918 mg/dL
2 SD = 6 mg/dL N = 14
The coefficient of variation for this control is:
A. 1.14% B. 2.19% C. 4.38% D. 9.49%
6. Which Westgard rule detects random error?
A. 13s B. 41s C. 22s D. 100
7. In addition to the number of true negative (TN), which of the following measurements is needed to calculate
specificity?
A. True positives B. Prevalence C. False negatives D. False positives
8. The component of a spectrophotometer that is responsible for detecting transmitted light and converting light energy
to electrical energy is the:
A. Detector B. Analytical cell C. Monochromator D. Readout device
9. Nephelometry is based on the measurement of light that is:
A. Absorbed by particles in suspension C. Produced by fluorescence
B. Scatterd by particles in suspension D. Produced by excitation of ground-state atoms
10. Which of the following instruments is used in the clinical laboratory or in reference laboratories to detect beta and
gamma emissions?
A. Fluorometer B. Nephelometer C. Spectrophotometer D. Scintillation counter
11. In flow cytometry, the side scatter is related to the:
A. DNA content of the cell C. Size of the cell
B. Granularity of the cell D. Number of cells in G0 and G1
12. The cellulose acetate electrophoresis at pH 8.6 of serum proteins will show an order of migration beginning with the
fastest migration as follows:
A. Albumin, alpha-1 globulin, alpha-2 globulin, beta globulin, gamma globulin
B. Alpha-1 globulin, alpha-2 globulin, beta globulin, gamma globulin, albumin
C. Albumin, alpha-2 globulin, alpha-1 globulin, beta globulin, gamma globulin
D. Gamma globulin, beta globulin, alpha-2 globulin, alpha-1 globulin, albumin
13. Ceruloplasmin and haptoglobin migrate electrophoretically as:
A. Alpha1 globulins B. Alpha2 globulins C. Beta globulins D. Gamma globulins
14. Which of the following serum protein fractions is most likely to be elevated in patients with nephrotic syndrome?
A. Alpha-1 globulin B. Albumin C. Alpha-2 globulin D. Beta globulin and gamma globulin
15. In cirrhosis, a predominant characteristic observed in electrophoretic serum pattern is a(an):
A. Increase in alpha2 globulin fraction C. Monoclonal band in gamma globulin region
B. Bridging effect between beta and gamma globulin fraction D. Polyclonal band in gamma globulin region
16. Which hemoglobin may be differentiated from other hemoglobins on the basis of its resistance to denature in alkaline
solution?
A. A1 B. A2 C. C D. F
17. Which of the following ezymes does not belong to the class of enzymes known as hydrolases?
A. Aldolase B. Amylase C. Lipase D. Alkaline phosphatase
18. Which of the following enzymes is a transferase?
A. ALP B. CK C. Amylase D. LD
19. When myocardial infarction occurs, in what order (list from first to last) will the enzymes aspartate aminotransferase
(AST), creatine kinase (CK), and lactate dehydrogenase (LD) become elevated in the serum?
A. AST, LD, CK B. CK, LD, AST C. CK, AST, LD D. LD, CK, AST
20. Time course of CK activity in myocardial infarction:
Onset of Elevation Peak Activity Duration of Elevation
A 1-3 h 5-12 h 18-30 h
B 4-8 h 12 h 3-4 d
C 8-12 h 24 h 5d
D 12-24 h 72 h 10 d
21. An electrophoretic separation of lactate dehydrogenase isoenzymes that demonstrates an elevation in LD-1 and LD-2
in a “flipped” pattern is consistent with:
A. Myocardial infarction B. Viral hepatitis C. Pancreatitis D. Renal failure
22. Which of the following enzymes exhibit a decreased serum level in liver disease?
A. Alanine aminotransferase D. Cholinesterase
B. Gamma glutamyltransferase E. All of the above
C. Lactate dehydrogenase
2

23. Which enzymes are generally quantified to help diagnose acute pancreatitis?
A. ALT and CK B. ALT and LD C. Amylase and lipase D. ALP and cholinesterase
24. Select the most sensitive marker for alcoholic liver disease.
A. AST B. ALT C. GGT D. None of these
25. In an adult, a normal GGT level with an elevated serum ALP level may be suggestive of:
A. Bone disease B. Heart disease C. Liver disease D. Skeletal muscle disease
26. The relative concentration of LD isoenzymes in normal serum:
A. LD 1>2>3>4>5 B. LD 2>1>4>3>5 C. LD 2>1>3>4>5 D. LD 5>4>3>2>1
27. A decreased LDH/HBD ratio is significant for:
A. Myocardial infarction B. Liver disease C. Both of these D. None of these
28. Isoenzyme of ALP that is most heat stable:
A. Placental ALP B. Bone ALP C. Intestinal ALP D. Liver ALP
29. Which of the following hormones does not promote an increase in blood glucose levels?
A. Growth hormone B. Cortisol C. Glucagon D. Insulin
30. Of the following blood glucose levels, which would you expect to result in glucose in the urine?
A. 60 mg/dL B. 120 mg/dL C. 150 mg/dL D. 225 mg/dL
31. An individual has a plasma glucose level of 110 mg/dL. What would be the approximate glucose concentration in the
patient’s cerebrospinal fluid?
A. 33 mg/dL B. 55 mg/dL C. 66 mg/dL D. 110 mg/dL
32. Select the enzyme that is most specific for -D-glucose:
A. Glucose oxidase C. Hexokinase
B. Glucose-6-phosphate dehydrogenase D. Phophohexoisomerase
33. Select the coupling enzyme used in the hexokinase method for glucose:
A. Glucose dehydrogenase C. Glucose-6-phosphate dehydrogenase
B. Glucose-6-phosphatase D. Peroxidase
34. Which of the following methods provides information about glucose metabolism over a 3 month period?
A. Fructosamine C. Glycosylated hemoglobin
B. Glucose oxidase D. Oral glucose tolerance
35. Measurement of glycosylated albumin, which reflects hyperglycemic state within the previous 2 – 3 weeks is:
A. Glucosamine B. Fructosamine C. Urine glucose D. Galactosamine
36. Type I DM maybe described by all of the following, EXCEPT:
A. Insulin dependent B. Insulinopenia C. Ketosis prone D. Adult-onset type
37. What is the compound that comprises the majority of the nonprotein-nitrogen fractions in serum?
A. Uric acid B. Creatinine C. Ammonia D. Urea
38. What analyte is measured using the Jaffe reaction?
A. Urea B. Uric acid C. Ammonia D. Creatinine
39. The creatinine clearance test is routinely used to assess the glomerular filtration rate. Given the following information
for an average-size adult, calculate the creatinine clearance.
Urine creatinine - 120 mg/dL
Plasma creatinine - 1.2 mg/dL
Urine volume for 24 hours - 1520 mL
A. 11 mL/min B. 63 mL/min C. 95 mL/min D. 106 mL/min
40. Given the following data, calculate the creatinine clearance, Serum creatinine = 1.2mg/dL; urine creatinine =- 120
mg/dL; urine volume = 1.75 L/day; surface area = 1.80 m 2.
A. 16 mL/min B. 117 mL/min C. 126 mL/min D. 168 mL/min
41. During chemotherapy for leukemia, which of the following analytes would most likely be elevated in the blood?
A. Uric acid B. Urea C. Creatinine D. Ammonia
42. A BUN of 9 mg/dL is obtained by a technologist. What is the urea concentration?
A. 18.3 mg/dL B. 19.3 mg/dL C. 10.3 mg/dL D. 9.3 mg/dL
43. Blood ammonia levels are usullay measured in order to evaluate:
A. Renal failure B. Acid-base status C. Hepatic coma D. Malabsorption
44. Indirect-reacting bilirubin may be quantified by reacting it initially in which reagent?
A. Dilute hydrochloric acid C. Caffeine-sodium benzoate
B. Dilute sulfuric acid D. Sodium hydroxide
45. The bilirubin fraction that is covalently attached to albumin and contributes to the conjugated bilirubin value is:
A. Delta B. Direct C. Indirect D. Bound
46. In which of the following conditions does decreased activity of glucuronyl transferase result in increased unconjugated
bilirubin and kernicterus in neonates?
A. Gilbert’s disease B. Dubin-Johnson syndrome C. Rotor’s syndrome D. Crigler-Najjar syndrome
47. A decrease in which of the following indicates increased intravascular hemolysis?
A. Methemoglobin B. Methemalbumin C. Haptoglobin D. Hemopexin
48. Select the order of mobility of lipoproteins electrophoresed on cellulose acetate or agarose at pH 8.6.
A. – Chylomicrons  prebeta  beta  alpha + C. – Chylomicrons beta  prebeta  alpha +
B. – Beta  prebeta  alpha chylomicrons + D. – Alpha  beta  prebeta  chylomicrons +
49. Chemical composition of HDL:
Protein (%) Cholesterol (%) Triglyceride (%) Phospholipid (%)
A 1-2 1-3 80-95 3-6
B 6-10 4-8 45-65 15-20
C 18-22 6-8 4-8 18-24
D 45-55 3-5 2-7 26-32
3

50. Which phenotype in the Frederickson classification of hyperlipoproteinemia is characterized by an increase in


cholesterol, an increase in beta lipoproteins, normal triglycerides, and the absence of chylomicrons?
A. I B. II C. III D. IV
51. Apolipoprotein A is the primary protein component of:
A. HDL B. LDL C. VLDL D. None of these
52. The lipoprotein that transports the exogenous triglycerides:
A. HDL B. LDL C. VLDL D. Chylomicrons
53. “Floating beta lipoprotein” refers to:
A. B-VLDL B. HDL C. VLDL D. B-LDL
54. Which of the following is referred to as the “good cholesterol”?
A. HDL B. LDL C. VLDL D. Free cholesterol
55. Which of the following diseases results from a familial absence of high density lipoprotein?
A. Krabbe disease B. Gaucher disease C. Tangier disease D. Tay-Sachs disease
56. The chemical composition of HDL cholesterol corresponds to:
A. Triglyceride 60% Cholesterol 15% Protein10% C. Triglyceride 5% Cholesterol 15% Protein 50%
B. Triglyceride 10% Cholesterol 45% Protein 25% D. Triglyceride 85% Cholesterol 5% Protein 2%
57. A fasting serum sample from an asymptomatic 43-year-old woman is examined visually and chemically with the
following results:
Initial appearance of serum: Milky Triglyceride level : 2000 mg/dL Cholesterol level : 550 mg/dL
Appearance of serum after overnight refrigeration: Cream layer over turbid serum
This sample contains predominantly:
A. Chylomicrons alone B. chylomicrons and VLDL C. VLDL and LDL D. HDL
58. In lipoprotein phenotyping, chylomicrons are present in the plasma of persons with which of the following lipoprotein
phenotypes?
A. I and IIa B. I and IIb C. II and III D. I and V
59. A patient’s total cholesterol is 300 mg/dL, his HDL cholesterol is 50 mg/dL, and his triglyceride is 200 mg/dL. What is
this patient’s calculated LDL cholesterol?
A. 200 B. 210 C. 290 D. 350
60. Which is the major intracellular cation?
A. Calcium B. Magnesium C. Sodium D. Potassium
61. Which is the major extracellular cation?
A. Chloride B. Sodium C. Magnesium D. Calcium
62. In cystic fibrosis, which sweat electrolytes are elevated?
A. Calcium and chloride C. Sodium and potassium
B. Chloride and sodium D. Potassium and chloride
63. A hemolyzed sample will cause falsely increased levels of each of the following EXCEPT:
A. Potassium B. Sodium C. Phosphate D. Magnesium
64. When measuring potassium using ion-selective electrode, the membrane is composed of:
A. Glass B. Plastic C. Silver D. Valinomycin
65. The total iron binding capacity (TIBC) test is performed to assess the blood level of:
A. Ferritin B. Hemopexin C. Iron D. Transferrin
66. Which protein is produced in decreased amount in Wilson’s disease?
A. Albumin B. Ceruloplasmin C. Haptoglobin D. Hemopexin
67. At blood pH 7.40, what is the ratio of bicarbonate to carbonic acid?
A. 15:1 B. 20:1 C. 25:1 D. 30:1
68. The reference range for blood pH is:
A. 7.28 – 7.34 B. 7.33 – 7.37 C. 7.35 – 7.45 D. 7.45 – 7.50
69. The most important buffer pair in plasma is the:
A. Phosphate / biphosphate pair C. Bicarbonate / carbonic acid pair
B. Hemoglobin / imidazole pair D. Sulfate / bisulfate pair
70. When arterial blood from a “normal patient is exposed to room air, the following happens:
A. PCO2 decreases, PO2 increases C. PCO2 decreases, PO2 decreases
B. PCO2 increases, PO2 decreases D. PCO2 increases, PO2 increases
71. A common cause of respiratory alkalosis is:
A. Vomiting B. Starvation C. Asthma D. Hyperventilation
72. Which of the following is the primary mechanism of compensation for metabolic acidosis?
A. Hyperventilation B. Hypoventilation C. Aldosterone release D. Bicarbonate excretion
73. Severe diarrhea causes:
A. Metabolic acidosis B. Metabolic alkalosis C. Respiratory acidosis D. Respiratory alkalosis
74. TSH is produced in the:
A. Hypothalamus B. Pituitary gland C. Adrenal cortex D. Thyroid
75. Thyroid function test in primary hypothyroidism:
T3 T4 TSH
A Decreased Decreased Decreased
B Decreased Decreased Increased
C Normal Increased Decreased
D Normal Normal Normal
76. The major carrier protein of T3 and T4 in the circulation is:
A. Albumin B. Thyroglobulin C. TBG D. Thyroxine-binding prealbumin
77. T3 uptake test is used to quantitate the concentration of:
A. Free T3 B. Protein-bound T3 C. Total T3 D. TBG
4

78. A patient’s T4 is 10 mg/dL. The THBR is 24% (normal=30%). What is the FT4I?
A. 8 B. 6 C. 4 D. 2
79. The principal estrogen produced during pregnancy is:
A. Estrone B. Estradiol C. Estriol D. Corticosterone

80. The Kober reaction is used in the assay of:


A. Urinary estrogen B. Glucocorticoids C. Testosterone D. Epinephrine
81. In which of the following would gastrin levels be increased:
A. Peptic ulceration C. Achlorhydria
B. Zollinger-Ellison syndrome D. Amyloidosis
82. Which of the following polypeptide hormones may be described as having alpha chains that are biochemically
identical but beta chains that as biochemically unique?
A. FSH, TSH, ACTH, TRH C. TSH, LH, TRH, HCG
B. LH, ACTH, HCG, TRH D. HCG, FSH, TSH, LH
83. The assay employed for 17-ketosteroids, in which steroids react with m-dinitrobenzene in alcoholic KOH solution
resulting to the formation of a purple color:
A. Kober Reaction B. Porter-Silber Reaction C. Zimmerman Reaction D. Pisano Method
84. A marked increase in 5-HIAA excretion occurs in patients with:
A. Argentaffinoma B. Diabetes insipidus C. Pheochromocytoma D. Diabetes mellitus
85. Which vitamin would be affected if a patient was diagnosed with a disorder involving fat absorption?
A. Vitamin B12 B. Ascorbic acid C. Thiamine D. Vitamin K
86. Which of the following drugs would be correctly classified as antiepileptic?
A. Digoxin B. Chloramphenicol C. Phenytoin D. Disopyramide
87. Digoxin, procainamide, and quinidine are drugs that may be classified as:
A. Aminoglycosides B. Anticonvulsants C. Antideppresants D. Cardioactive
88. The medication of choice for treatment of manic-depression is:
A. Carbamazepine B. Lithium carbonate C. Phenobarbital D. Phenytoin
89. A drug that relaxes the smooth muscles of the bronchial passages is:
A. Acetaminophen B. Lithium C. Phenytoin D. Theophylline
90. Which of the following drugs may be quantified colorimetrically using Trinder’s reagent (mercuric chloride, ferric nitrate
and hydrochloric acid)?
A. Acetaminophen B. Phenobarbital C. Salicylate D. Theophylline
91. Cocaine is metabolized to:
A. Carbamazepine B. Codeine C. Hydrocodone D. Benzoylecgonine
92. Four children are admitted with malaise, anorexia, and abdominal pain. Further evaluations reveal mild anemia,
erythrocyte basophilic stippling, and profound pica habits. Poisoning by which heavy metal is most likely responsible?
A. Arsenic B. Iron C. Mercury D. Lead
93. Caffeine is an important metabolite of this drug:
A. Acetaminophen B. Digoxin C. Theophylline D. Phenobarbital
94. Odor of bitter almond:
A. Cyanide poisoning B. Ethanol poisoning C. Arsenic poisoning D. CO poisoning
95. The formation of this crystal in urine, although not a constant finding is an important diagnostic clue of ethylene glycol
poisoning:
A. Uric acid B. Ammonium biurate C. Triple phosphate D. Calcium oxalate
96. This common substance of abuse is derived from Cannabis sativa leaves and stems. Which of the following is it?
A. Heroine B. Cocaine C. Marijuana D. Amphetamines
97. Which type of extinguisher should put out a fire caused by electrical equipment?
Type of Extinguisher
A Pressurized water/Dry chemical
B Dry chemical/Carbon dioxide
C Carbon dioxide/Halon/Dry chemical
D Metal X/Special dry chemical
98. 4 mL of water is added to 1 mL of serum. This represents which of the following serum dilutions?
A. 1:3 B. 1:4 C. 1:5 D. 1:6
99. What is the molarity of a solution that contains 18.7 g of KCl (MW =74.5) in 500 mL of water?
A. 0.1 B. 0.5 C. 1.0 D. 5.0
100. All of the following are correct match, EXCEPT:
Conversion of Conventional Unit to SI Unit:
Conventional Unit SI Unit
A Cholesterol 150 mg/dL 3.88 mmol/L
B Triglyceride 10 mg/dL 0.11 mmol/L
C Glucose 70 mg/dL 3.9 mmol/L
D BUN 8 mg/dL 0.47 mmol/L

END OF THE EXAM


PRE-BOARD EXAMINATION IN CLINICAL CHEMISTRY (PART3)

1. What is the prefix for factor 10-15?


A. Nano B. Atto C. Pico D. Femto
2. Fasting serum glucose has a concentration of 110 mg/dL. In SI, this value is equivalent to:
A. 6.05 mmol/L B. 3.03 mmol/L C. 6.49 mmol/L D. 4.99 mmol/L
3. A normal BUN value was reported at 25mg/dL. What is its equivalent in SI units?
A. 8.93umol/L B. 8.93mmol/L C. 7.14umol/L D. 7.14mmol/L
4. Lactescence occurs when serum triglyceride level exceeds what concentration?
A. 25 mg/L B. 25 mg/dL C. 400 mg/L D. 400 mg/dL
5. Given that a method mean is 25 mg/dL and the standard deviation is 1.2 mg/dL, what would be the coefficient
of variation?
A. 2.1% B. 2.4% C. 4.8% D. 9.6%
6. Considering a normal Gaussian curve distribution, how many values from a population will be within 2 SD?
A. 95.45% B. 75.30% C. 68.27% D. 99.73%
7. A delta check:
A. Relates control difference from mean C. Evaluates statistical drift
B. Reports patient value difference from previous analysis D. Flags abnormal results
8. Which of the following instruments is used in the clinical laboratory to detect beta and gamma emissions?
A. Fluorometer B. Nephelometer C. Scintillation counter D. Spectrophotometer
9. In potentiometry, the following are types of reference electrodes, EXCEPT:
A. Glass electrode C. Saturated calomel electrode
B. Standard hydrogen electrode D. Silver-silver chloride electrode
10. Which of the following substances are introduced in a continuous-flow analyzer to minimize diffusion of
reagents and mixing between samples?
A. Membranes B. Resins C. Air bubbles D. Gel polymers
11. The protein fraction that migrates the fastest toward the anode:
A. Albumin B. Beta-globulin C. Alpha1-globulin D. Gamma-globulin
12. Protein also known as orosomucoid:
A. 1-Acid glycoprotein B. 2- Macroglobulin C. -Lipoprotein D. C-reactive protein
13. Which of the following substances is markedly increased in nephrotic syndrome?
A. Ceruloplasmin B. Alpha-2-macroglobulin C. Alpha-1-antitrypsin D. Albumin
14. The neocuproine method for glucoses is based on:
A. Glucose oxidase reaction C. Condenstaion reaction
B. Copper reduction by glucose D. Hexokinase reaction
15. Select the enzyme most specific for beta D-glucose.
A. Hexokinase C. Phosphohexisomerase
B. Glucose-6phosphate dehydrogenase D. Glucose oxidase
16. All of the following are characteristics of Type II diabetes mellitus except:
A. Insulin levels may or may not be abnormal
B. It is more common than Type I diabetes
C. It requires insulin therapy to control hyperglycemia
D. It is associated with obesity and more common in persons greater than 40 years old
17. Select the order of mobility of lipoproteins electrophoresed on cellulose acetate or agarose at pH 8.6.
A. – Chylomicrons  prebeta  beta  alpha +
B. – Beta  prebeta  alpha chylomicrons +
C. – Chylomicrons beta  prebeta  alpha +
D. – Alpha  beta  prebeta  chylomicrons +
18. Select the lipoprotein fraction that carries most of the endogenous triglycerides.
A. VLDL B. LDL C. HDL D. Chylomicrons
19. The lipoprotein that transports the exogenous triglycerides:
A. HDL B. LDL C. VLDL D. Chylomicrons
20. Apolipoprotein A is the primary protein component of:
A. HDL B. LDL C. VLDL D. None of these
21. “Floating beta lipoprotein” refers to:
A. B-VLDL B. HDL C. VLDL D. B-LDL
22. An abnormal LDL lipoprotein, also known as the “sinking pre-B-lipoprotein” associated with a higher risk for
atherosclerosis:
A. B-VLDL B. LpX C. Lp(a) D. LDL
23. An abnormal lipoprotein found in patients with obstructive biliary disease:
A. B-VLDL B. LpX C. Lp(a) D. LDL
24. Which of the following is referred to as the “good cholesterol”?
A. HDL B. LDL C. VLDL D. Free cholesterol
25. A type IIA hyperlipoproteinemia phenotype is associated with an increased level of:
A. Chylomicrons B. HDL C. VLDL D. LDL
26. The bilirubin fraction that is covalently attached to the albumin is:
A. Direct B. Indirect C. Delta D. Bound
27. What is the second largest constituent of the plasma non-protein nitrogen?
A. Urea B. Uric acid C. Ammonia D. Amino acids
28. Which of the following is the most abundant amino acid in plasma?
A. Lysine B. Glutamine C. Ornithine D. Cysteine
2

29. In the diacetyl monoxime method, diacetyl reacts directly with:


A. Urea B. Uric acid C. Ammonia D. Ammonium ion
30. It absorbs light at 293 nm in the uricase method:
A. Uric acid B. Uricase C. Allantoin D. None of these
31. The Wacker method uses what reaction?
A. Pyruvate  (LD) Lactate C. Creatine phosphate  (CK) creatine
B. Lactate  (LD) Pyruvate D. Creatine  (CK) Creatine phosphate
32. Which LDH isoenzyme migrates the farthest from the origin on electrophoresis?
A. LD1 B. LD2 C. LD4 D. LD5
33. The presence of a flipped LD1/LD2 pattern in the presence of CK-MB is suggestive of:
A. Hepatitis B. Cirrhosis C. MI D. Bone disease
34. The isoenzymes LD-4 and LD-5 are elevated in:
A. Acute pancreatitis C. Hepatic injury or inflammation
B. Hemolyzed specimen D. Myocardial infarct
35. Which of the following methods applies to the principle below?
Creatine + ATP  (CK) Creatine phosphate + ADP
A. Szaz B. Wroblewski C. Tanzer-Gilvarg D. Oliver-Rosalki
36. Which isoenzyme of creatine kinase (CK) has the fastest electrophoretic mobility at pH 8.6?
A. MM B. MB C. BB D. Macro-CK
37. Which of the isoenzymes of ALP is most anodal?
A. Placental ALP B. Bone ALP C. Intestinal ALP D. Liver ALP
38. Which enzyme is a useful indicator of possible insecticide poisoning?
A. 5’nucleotidase B. ACP C. Cholinesterase D. LDH
39. Which of the isoenzymes of ALP is most heat stable?
A. Placental ALP B. Bone ALP C. Intestinal ALP D. Liver ALP
40. There are several alkaline phosphatase isoenzymes that are characteristic of cancer.
Electrophoretic mobility of these fractions is similar to what ALP isoenzyme:
A. Placental ALP B. Bone ALP C. Liver ALP D. Intestinal ALP
41. The Szasz assay determines the activity of:
A. CK B. ALP C. GGT D. LDH
42. Formula for serum osmolality:
A. (2Na+) + (Glucose/20) + (BUN/3) C. Na+ + 3Glucose + 2BUN
B. (Na+/3) + (2Glucose) + (BUN/20) D. Na+ - ( Cl- + total CO2)
43. Later models of flame photometer uses_____ as an internal standard that allows the operator to measure
sodium, potassium and lithium directly.
A. Lithium B. Cesium C. Magnesium D. Calcium
44. When measuring potassium using an ion selective electrode, the membrane is composed of:
A. Glass B. Plastic C. Silver D. Valinomycin
45. Which electrolytes are elevated in cystic fibrosis?
A. Calcium and chloride C. Sodium and potassium
B. Chloride and sodium D. Potassium and chloride
46. In the blood, bicarbonate leaves the red blood cell and enters the plasma through an exchange mechanism
with:
A. Sodium B. Potassium C. Chloride D. Phosphate
47. It is an essential component of the glucose tolerance factor, thought to play a role in maintaining normal insulin
response to glucose load by affecting membrane response to insulin:
A. Chromium B. Zinc C. Nickel D. Molybdenum
48. Increased chloride levels as determined by colorimetric and ion-selective electrode methods may be caused by
the presence in the solution of:
A. Bromide B. Calcium C. Hydrogen D. Chylomicrons
49. Which of the following is the primary mechanism causing respiratory alkalosis?
A. Hyperventilation C. Deficient pulmonary perfusion
B. Hypoventilation D. Parasympathetic inhibition
50. If a blood gas specimen is left exposed to the air for an extended period of time, which of the following changes
will occur?
A. PO2 and pH increase; PCO2 decreases C. PO2 increases; pH and PCO2 decrease
B. PO2 and pH decrease; PCO2 increases D. PO2 decreases; pH and PCO2 increase
51. Sealed heparinized arterial blood was left at room temperature for 2 hours. The most likely changes in PO 2
(mmHg), PCO2(mm Hg), and pH, respectively, are:
A. Increase, increase and increase C. Decrease, increase, and decrease
B. Decrease, decrease, and decrease D. Decrease, decrease, and increase
52. The adrenal medulla secretes which of the following in the greatest quantity?
A. Metanephrine B. Noradrenaline C. Epinephrine D. Dopamine
53. Homovanillic acid is the principal urine metabolite of:
A. Norepinephrine B. Epinephrine C. Adrenaline D. Dopamine
54. Diurnal variation is important to consider when collecting blood for the assay of:
A. Catecholamines B. Creatinine C. Cortisol D. Thyroid hormones
55. T-3 uptake is actually a measurement of:
A. T-3 B. T-4 C. TBG D. Free thyroxine
3

56. Active hormonal form of T3 an T4:


A. Those bound to TBG C. Those bound to transthyretin
B. Those bound to albumin D. Those in free from
57. The principle is based on the reaction of urinary estrogen with a mixture of phenol and sulfuric acid to produce
pink color. This refers to:
A. Kober reaction B. Trinder reaction C. Zimmermann reaction D. Porter-Silber reaction
58. The Kober reaction is used in the assay of:
A. Urinary estrogen B. Glucocorticoids C. Testosterone D. Epinephrine
59. In the Porter-Silber assay, the dihydroxyacetone side chain of the steroid hormone reacts with:
A. Sulfuric acid-hydroquinone and forms reddish-brown color
B. m-dinitrobenzene and forms purple color
C. Ceric and arsenite compound and forms a yellow product
D. 2, 4 –dinitrophenylhydrazizne and forms a yellow derivative
60. The assay employed for 17-ketosteroids, in which steroids react with m-dinitrobenzene in alcoholic KOH
solution resulting to the formation of a purple color:
A. Kober Reaction B. Zimmerman reaction C. Porter-Silber Reaction D. Pisano Method
61. Zollinger-Ellison syndrome is characterized by elevated blood levels of:
A. Cholecystokinin B. Trypsin C. Pepsin D. Gastrin
62. Tumor marker most useful in the detection of familial medullary carcinoma of the thyroid:
A. Calcitonin B. CA 125 C. CEA D. CA 19-9
63. What metal toxin in urine is detected by the Reinsch test?
A. Lead B. Mercury C. Bromide D. Zinc
64. Trinder’s reagent (mercuric chloride, HCl, and ferric nitrate) is used in the colometric assay for:
A. Acetaminophen B. Salicylate C. Theophylline D. Ethanol
65. Caffeine is an important metabolite of this drug, which is assayed in newborns and young children to monitor its
therapeutic level. What is this?
A. Acetaminophen B. Digoxin C. Theophylline D. Phenobarbital
66. Odor of bitter almond gives a clue of:
A. Cyanide poisoning B. Ethanol poisoning C. Arsenic poisoning D. Carbon monoxide poisoning
67. Benzoylecgonine is the major metabolite of:
A. Heroin B. Marijuana C. Cocaine D. Phencyclidine
68. The formation of this crystal in urine, although not a constant finding is an important diagnostic clue of ethylene
glycol poisoning:
A. Uric acid B. Ammonium biurate C. Triple phosphate D. Calcium oxalate
69. What is the major carrier of drugs in the circulation?
A. Albumin B. Globulin C. Transferrin D. Hemoglobin
70. Fire extinguishers designated as Class A are used for:
A. Paper and wood B. Electrical equipment fire C. Flammable liquids and gases D. All of the above
71. It is a specialized colorimeter designed to scan and quantitate electrophoresis patterns:
A. Densitometer B. Detector C. Atomizer D. Monochromator
72. The element that distinguishes proteins from carbohydrate and lipid compounds is:
A. Carbon B. Oxygen C. Nitrogen D. Phosphorus
73. Parfentjev’s method is for the determination of:
A. Fibrinogen B. Albumin C. Globulin D. Amylase
74. Apolipoprotein A is the primary protein component of:
A. HDL B. IDL C. LDL D. VLDL
75. It is biologically important, as it serves as the starting point in many metabolic pathways including Vitamin D
synthesis, steroid hormone synthesis, and bile acid metabolism.
A. Cholesterol B. Phospholipid C. Triglycerides D. Free fatty acids
76. What is the current reference method for cholesterol analysis?
A. Abell-Kendall method B. Bloor’s method C. Salkowski method D. Lieberman-Burchardt
77. A mild condition that appears to result from a genetic defect in transport of bilirubin from sinusoidal blood into
the hepatocyte:
A. Gilbert Syndrome B. Crigler-Najjar Syndrome C. Dubin-Johnson D. Rotor Syndrome
78. What reagent is used in the Evelyn-Malloy method to dissociate the unconjugated bilirubin from protein?
A. Methanol B. Ethanol C. Caffeine D. Acetic acid
79. The Jaffe reaction is employed for the quantitation of:
A. Urea B. Creatinine C. Protein D. Uric acid
80. Lloyd’s reagent improves the specificity of what colorimetric method of determination?
A. Jaffe B. Caraway C. Lieberman-Burchardt D. Biuret
81. What is the major end product of protein and amino acid catabolism?
A. Urea B. Uric acid C. Creatine D. Creatinine
82. Uric acid when oxidized by the enzyme uricase is transformed to:
A. Allantoin B. Monosodium urate C. Xanthine D. Ammonia
83. The sweat chloride test is useful in the diagnosis of:
A. Dehydration B. Cystic fibrosis C. Azotemia D. Diabetes
84. Which trace metal accumulates in Wilson’s disease?
A. Cobalt B. Copper C. Nickel D. Zinc
85. What is the anticoagulant of choice for blood gas analysis?
A. EDTA B. Heparin C. Oxalate D. Citrate
4

86. The pH of blood is critically maintained at what level:


A. 7.00-7.50 B. 7.50-7.70 C. 7.15-7.35 D. 7.35-7.45
87. In which of the following are the thyroid hormones classified:
A. Amino acid derivatives B. Steroid hormones C. Fatty acid derivatives D. Peptide hormones
88. Which of the following polypeptide hormones may be described as having alpha chains that are biochemically
identical but beta chains that are biochemically unique?
A. FSH, TSH, ACTH, LH B. TSH, LH, TRH, HCG C. LH, ACTH, HCG, TRH D. HCG, FSH, TSH, LH
89. The thyroid gland produces all of the following hormones EXCEPT:
A. TSH B. Calcitonin C. Thyroxine D. Triidothyronine
90. In hypothyroidism, one would expect the total T4 level to be _____, and the T3 uptake to be ____.
A. Increased, increased B. Decreased, decreased C. Decreased, increased D. Increased, decreased
91. How can primary hypothyroidism be differentiated from secondary hypothyroidism?
A. T3 B. T4 C. TSH D. Both A and B
92. 5-Hydroxyindoleacetic acid is the primary metabolite of:
A. Epinephrine B. Prolactin C. Norepinephrine D. Serotonin
93. A marked increase in 5-HIAA excretion occurs in patients with:
A. Argentaffinoma B. Pheochromocytoma C. Diabetes insipidus D. Diabetes mellitus
94. Digoxin, procainamide and quinidine are drugs that may be classified as:
A. Aminoglycosides B. Anticonvulsants C. Antidepressant D. Cardioactive
95. Lithium therapy is widely used in the treatment of:
A. Hypertension B. Hyperactivity C. Aggression D. Manic-depression
96. A drug that relaxes the smooth muscles of the bronchial passages is:
A. Acetaminophen B. Lithium C. Phenytoin D. Theophylline
97. Which of the following statements pertains to the effect of ethanol?
1. Ethanol functions as a depressant of the central nervous system
2. Initial effect is an increase in heart rate and blood pressure
3. Long-term abuse can impair most organs of the body; primary tissue affected is the liver
4. Blood alcohol content of 0.35 to 0.50 % is associated with coma and possible death
A. 1 and 3 B. 2 and 4 C. 1, 2 and 3 D. 1, 2, 3 and 4
98. This toxin has high affinity to keratin, can be identified from hair and nails:
A. Lead B. Cyanide C. Mercury D. Arsenic
99. This common substance of abuse is derived from Cannabis sativa leaves and stems. Which of the following is
it?
A. Heroine B. Cocaine C. Marijuana D. Amphetamines
100. All of the following vitamins are lipid in nature and classified as fat-soluble, EXCEPT:
A. Vit. A B. Vit. C C. Vit. D D. Vit. K

END OF THE EXAM


1

PRE-BOARD EXAMINATION IN CLINICAL MICROSCOPY (PART 1)

1. Urine from a patient with polyuria has a high specific gravity. The patient should be evaluated for:
A. Urinary tract infection C. Diabetes insipidus
B. Diabetes mellitus D. Uremia
2. An unpreserved specimen collected at 8 AM and remaining at room temperature until the aftern
shift arrives can be expected to have:
1. Decreased glucose and ketones 3. Decreased pH and turbidity
2. Increased bacteria and nitrite 4. Increased cellular elements
A. 1, 2 and 3 B. 1, 2 and 4 C. 1 and 2 only D. 4 only
3. Red cells will disintegrate more rapidly in a urine that is:
A. Concentrated and acidic C. Dilute and acidic
B. Concentrated and alkaline D. Dilute and alkaline
4. A negative urine pregnancy performed on a random specimen may need to be repeated using a:
A. Clean-catch specimen C.Fasting specimen

B. First morning specimen D.24-hour specimen

5. Persons taking diuretics can be expected to produce:

A. Proteinuria B. Polyuria C.Pyuria D. Oliguria

6. The renal threshold for glucose is:

A. 50 – 100 mg/dL B. 160 – 180 mg/dL C.220 – 240 mg/dL D. Over 240 mg/dL

7. Increased production of vasopressin:

A. Produces a low urine volume C.Increased ammonia excretion

B. Produces a high urine volume D.Affects proximal convoluted tubule


transport
8. Osmometers utilizing the freezing point colligative property of solutions are based on the
principle that:
A. 1 osmole of nonionizing substance dissolve in 1 kilogram of water raises the freezing
point 1.86oC
B. O
1 osmole of nonionizing substance dissolve in 1 kilogram of water lowers the freezing
point 1.86oC
C. Increased solute concentration will raise the freezing point of water in direct proportion
to NaCl standard
D. Decreased solute concentration will decrease the freezing point of water in direct
proportion to NaCl standard
9. The PAH test is dependent on:
1. Renal blood flow 3. Tubular secretion

2. Tubular reabsorption 4. Glomerular filtration

A. 1 and 3 B.2 and 4 C. 1 only D.4 only


2

10. Which biochemical component would be present in an increased amount in dark yellow-
amber-colored urine?
A. Biliverdin B. Drugs C. Bilirubin D. Urobilin
11. Specimens from patients receiving treatment for UTI, frequently appear:
A. Clear and red C. Dilute and pale yellow
B. Viscous and orange D. Cloudy and red
12. When should a 2-hour postprandial urine be collected?
A. 2 hours after fluid ingestion C. 2 hours after eating
B. 2 hours after voiding a fasting specimen D. 2 hours after fluid ingestion
13. Refractive index compares:
A. Light velocity in solutions with light velocity in solids
B. Light velocity inCair with light velocity in⑳
solutions
C. Light scattering in air with light scattering in solutions
D. Light scattering by particles in solution
14. What is the principle of the colorimetric reagent strip determination of specific gravity in
urine?
A.
B.
O
Ionic strength alters pKa of a polyelectrolyte
Sodium and other cations are chelated by a ligand that changes color
C. Anions displace a pH indicator from a mordant, making it water soluble
D. Ionized solutes catalyze oxidation of an azo dye
15. A urine specimen with a specific gravity of 1.008 has been diluted 1:5. The actual specific
gravity is:

16.
A. 1.008 B. 1.040 C. 1.055 D. 5.040
What are the most common renal stones encountered in the clinical laboratory?
A. Calcium phosphate C. Cystine
T
B. Calcium oxalate D. Uric acid
17. When using polarized light microscopy, which urinary sediment component exhibits Maltese
cross formation?
A. RBCs B. Oval fat bodies C. Yeasts D. WBCs
18. Which of the following urine biochemical results would be obtained in hemolytic anemia?
A. Positive glucose C. Positive nitrate B. Positive bilirubin D. Positive urobilinogen
19. In renal tubular acidosis, the pH of urine is:
A. Consistently acid C. Neutral
B. Consistently alkaline D. Variable, depending upon diet
20. What is a commonly used drug noted to produce a bright orange-red color in urine?
A. Furadantin B. Levodopa C. Rifampin D. Riboflavin
21. Which of the following would produce a yellow foam in urine when shaken which could be
mistaken for bilirubin:
A. Pyridium B. Protein C. Urates D. Glucose
22. A negative glucose oxidase test and a positive test for reducing sugars in urine indicates:
A. True glycosuria C. A false-negative oxidase reaction
B. Presence of nonglucose reducing sugar D. A trace quantity of glucose
3

23. Acetoacetic acid is detected in urine by reaction with:


A. Sodium nitroprusside C. m-Dinitrobenzene
B. o-Toluidine D. m-Dinitrophenylhydrazine
24. Hemoglobin in urine can be differentiated from myoglobin using:
A. 0
80% ammonium sulfate to precipitate hemoglobin
B. Sodium dithionite to reduce hemoglobin
C. O-Dianisidine instead of benzidine as the color indicator
D. Microscopic exam
25. Which of the following reagents below is used to detect urobilinogen in urine:
A. p-Dinitrobenzene C. p-Dimethylaminobenzaldahyde
B. p-Aminosalicylate D. p-Dichloroaniline
26. A patient has a 1+ protein reaction on a specimen collected in the doctor’s office. The doctor
instructs the patient to collect a specimen immediately upon arising the next day, the specimen is
negative for protein. This indicates:
A. Nocturnal proteinuria C. Microalbuminuria
B. Orthostatic proteinuria D. Diurnal proteinuria
27. The finding of a 2+ reagent strip reaction for blood in the urine of a patient with severe lower back
pain can aid in confirming a diagnosis of:
A. Pyelonephritis B. Appendicitis C. Renal calculi D. Multiple myeloma
28. Bence Jones protein excreted in urine in cases of multiple myeloma has a unique characteristic of:
A. Reacting with reagent strips and not sulfosalicylic acid
B. Precipitating in acetic acid and heat and not with sulfosalicylic acid
C. Precipitating when heated at 60oC and dissolving at 100oC
D. Precipitating when heated at 100oC and dissolving at 60oC
29. Ammonium sulfate was added to clear urine. After filtration the supernatant remains red and has
a positive reaction on the blood portion of the reagent strip. This is caused by the presence of:
A. Myoglobin B. Medication C. Hemoglobin D. Methemoglobin
30. A positive urine bilirubin with a normal test for urobilinogen indicates:
A. Intravascular hemolysis C. Hepatitis B. Biliary duct obstruction D. Cirrhosis
31. A urinalysis report shows:
Bilurubin: negative Blood: Small Urobilinogen: 4 EU Nitrite: negative
This suggests:

A. Abnormal destruction of RBCs in the body C. Severe UTI


B. Inflammation of the liver D. A normal urine specimen
32. A specimen that produces a cherry-red color with Ehrlich’s reagent is extracted with chloroform
and butanol. If the positive reaction is caused by porphobilinogen, you would expect the
extraction to show:
A. Extraction into chloroform and butanol C. Extraction into chloroform but not butanol

B. Extraction into butanol but not chloroform D. No extraction into chloroform or butanol

33. What do the following results suggest:

Color: yellow hazy Bilirubin: negative


4

Sp. Gr.: 1.019 Blood: negative

pH: 8 Urobilinogen: 0.1 EU

Protein: trace Nitrite: positive

Glucose: negative Leukocytes: positive


Ketones: negative
A. Diebetes mellitus C. Urinary tract infection

B. Unsatisfactory specimen D. Normal female specimen

34. Examination of urine sediment for the presence of casts should be


performed with: A. Increased light under high power

C. Reduced light under high power

B. Increased light under low power D. Reduced light


under low power

35. Which of the following dyes are used in Sternheimer-Malbin stain?

A. Hematoxylin and eosin C. Methylene blue and eosin

B. Crystal violet and safranin D. Methylene blue and


sfranin

36. The predecessor of the standardized urine microscopic examination was the:
A. Sternheimer count C. Kova system
B. Addis count D. T-system
37. The ova of which parasite may be found in the urinary sediment?
A. Trichomonas vaginalis C. Schistosoma haematobium
B. Entamoeba histolytica D.Trichuris trichiura

38. Oval fat bodies are often seen in:

A. Chronic glumerulonephritis C. Acute tubular nephrosis

B. Nephrotic syndrome D.Renal failure

39. Identification of oval fat bodies can be verified using:

A. Bright-field microscopy C.Polarized light

B. Phase contrast D.Interference-contrast


microscopy
40. A sediment with moderate hematuria and RBC casts most likely results from:
A. Chronic pyelonephritis C. Acute glomerulonephritis
B. B. Nephrotic syndrome D. Lower urinary tract obstruction
41. Urine sediment characterized by pyuria with bacteria and WBC casts indicates:
A. Nephrotic syndrome C. Polycystic kidney disease
B. Pyelonephritis D. Cystitis
5

42. Hexagonal uric acid crystals can be distinguished from cystine crystals because:
A. Uric acid is insoluble in hydrochloric acid and cystine is not
B. Cystine gives a positive nitroprusside etst after reduction with sodium cyanide
C. Cystine crystals are colorless
D. All of the above
43. The presence of tyrosine and leucine crystals together in urine sediment usually indicates:
A. Renal failure C. Hemolytic anemia
B. Chronic liver disease D. Hartnup’s disease
44. Which of the following crystals is considered nonpathological?
A. Hemosiderin B. Ammonium biurate C. Bilirubin D. Cholesterol
45. Which crystals appear in urine as a long, thin hexagonal plate, and are linked to ingestion of
benzoic acid?
A. Cystine B. Hippuric acid C. Oxalic acid D. Uric acid
46. Oval fat bodies are derived from:
A. Renal tubular epithelium C. Degenerated WBCs
B. Transitional epithelium D. Mucoprotein matrix
47. Which condition is associated with the greatest proteinuria?
A. Acute glomerulonephritis C. Nephrotic syndrome
B. Chronic glomerulonephritis D. Acute pyelonephritis
48. The finding of increased hyaline and granular cast in the urine of an otherwise healthy person may
be the result of:
A. Fecal contamination C. Early UTI
B. Recent strenuous exercise D. Analyzing an old specimen
49. The Guthrie test is a:
A. Bacterial inhibition test
B. Fluorometric procedure
C. Chemical procedure measured by spectrophotometer
D. Bacterial agglutination test
50. The abnormal metabolite that is present in the urine in alkaptonuria is:
A. Homogentisic acid C. Phenylpyruvate
B. Alkaptonpyruvate D. Tyrosine
51. A mousy odor in the urine is associated with:
A. Phenylketonuria B. Isovaleric acidemia C. Cystinuria D. Cystinosis
52. The finding of increased amount of the serotonin degradation product 5-HIAA in the urine is
indicative of:
A. Platelet disorders B. Intestinal obstruction C. Malabsorption D. Carcinoid tumor
invol. argentaffin cells
53. Hurler’s and Sanfilippo’s syndromes present with mental retardation and increased urinary:
A. Porphyrins B. Amino acids C. Maltose D. Mucopolysaccharides
54. Regarding CSF, all of the following are indications of a traumatic tap, EXCEPT:
A. Clearing of the fluid as it is aspirated C. Xanthochromia
B. A clear supernatant after centrifugation D. Presence of clot in the sample
55. The term used to denote high WBC count in the CSF is:
A. Empyemia C. Pleocytosis
6

B. Neutrophilia D. Hyperpycorrhachia
56. The limulus lysate test on CSF is a sensitive assay for:
A. Demyelinating diseases of the spinal cord C. Gram-negative bacterial
endotoxin
B. Cryptococcal meningitis D. Open neural tube defects
57. Pronounced reduction of CSF glucose can be seen in the following conditions, EXCEPT:
A. Viral meningitis C. Fungal meningitis
B. Bacterial meningitis D. Tuberculous meningitis
58. An elevated IgG level in CSF and an abnormal band on electrophoresis of CSF are findings
consistent with the diagnosis of:
A. Multiple sclerosis C. Meningeal involvement in leukemia
B. Muscular dystrophy D. Secondary stage of syphilis
59. CSF specimen is usually collected in 3 sterile tubes labeled in the order they are drawn. What
laboratory test is done with tube number 3?
A. Chemistry B. Microbiology C. Cell count D. Serology
60. Total volume of CSF in adults:
A. 20 mL B. 140-170 mL C. 10-60 mL D. 220-280 mL
61. The normal CSF protein is:
A. 15 to 45 mg/dL B. 15 to 45 g/dL C. 50 to 100 mg/dL D. 50 to 100 g/dL
62. To determine if fluid draining from the ear of the patient with severe head injury is CSF, the fluid
should be:
A. Centrifuged and examined for the presence of ependymal cells
B. Electrophoresed for the presence of transferring isoforms
C. Analyzed for the presence of glutamine
D. Tested for low protein concentration
63. A major CSF chemical that is measured in suspected cases of Reye’s syndrome is:
A. Glucose B. Glutamine C. Lactate D. Lactate dehydrogenase
64. Which of the following terms is another name for ascitic fluid?
A. Peritoneal B. Pericardial C. Synovial D. Pleural
65. All of the following statements about serous fluids are true, EXCEPT:
A. An effusion is an abnormal accumulation of a serous fluid
B. Thoracentesis refers the collection of pericardial fluid
C. Ascites refer specifically to peritoneal fluid
D. The term ‘chest fluid” usually refers to pleural fluid

66. A transudative effusion is usually caused by all of the following, EXCEPT:


A. Congestive heart failure C. Nephrotic syndrome
B. Malignancy D. Cirrhosis
67. An exudative pleural fluid can be caused by:
A. Malignancy C. SLE or rheumatoid arthritis
B. Pulmonary infarction or infection D. All of the above
68. Measurement of blood urea nitrogen and creatinine in peritoneal fluid are requested when there
is a concern about:
A. Tubercular peritonitis C. Malignacy
B. Ruptured bladder D. Gastrointestinal perforations
7

69. These fluid have high protein content:


A. Exudate B. Urates C. Sulfates D. Transudates
70. Useful adjunct test for tuberculous pericarditis in cases with negative stains when tuberculosis is
suspected:
A. Lactate dehydrogenase activity C. Adenosine deaminase activity
B. Bilirubin level D. Lactate level
71. All of the statements below about synovial fluid are true, EXCEPT: A. Arthrocentesis is
performed only to evaluate arthritis.
B. Synovial fluid is present only in movable joints
C. Joint fluid resembles plasma in viscosity
D. Joint fluid has approximately the same glucose as plasma
72. Which of the following crystals is the cause of gout?
A. Uric acid or monosodium urate C. Calcium oxalate
B. Calcium pyrophosphate D. Cholesterol
73. Which crystal causes “pseudogout”?
A. Oxalic acid C. Calcium oxalate
B. Calcium pyrophosphate D. Cholesterol
74. Using compensated polarized microscopy, which synovial fluid crystals appear yellow when
observed with the long axes of the crystals parallel to the optical field?
A. Uric acid B. Ca pyrophosphate C. Cyteine D. Cholesterol
75. The synovial fluid glucose level is low in all of the following types of arthritis, EXCEPT:
A. Septic arthritis C. Hemorrhagic arthritis
B. Inflammatory arthritis D. Gout
76. Neutrophils that contain precipitated rheumatoid factor in their cytoplasm are called:
A. LE cells B. Reiter cells C. Ragocytes D. Macrophages
77. Which test correlates with the severity of hemolytic disease of the newborn? A. Rh antibody
titer of the mother C. Amniotic fluid bilirubin
B. L/S ratio D. Urinary estridiol

78. The best available guide in the assessment of gestational age is the amniotic fluid determination
of:
A. Bilirubin B. Uric acid C. Creatinine D. Protein
79. Microviscosity of amniotic fluid is measured by:
A. Thin-layer chromatography C. Spectrophotometer
B. Immunologic agglutination D. Fluorescence polarization
80. Bronchoalveloar lavage may be performed in patients with AIDS to detect presence of:
A. Increased helper T cells C. Kaposi sarcoma B. Bacterial pneumonitis D. Pneumocystic carinii
81. The most abundant cell seen in a bronchoalveolar lavage is the:
A. Neutrophil C. Macrophage
B. Clilited columnar bronchial epithelial cell D. Lymphocyte
82. The milky fluid that that contains acid phosphatase and proteolytic enzymes in the seminal fluid
comes from which part of the male genitalia?
A. Prostate B. Testis C. Seminal vesicles D. Epididymis
83. To detect the presence of semen, which of the following enzyme activities should be measured?
8

A. Citric acid B. ALP C. ACP D. AST


84. Seminal vesicles produce:
A. Glucose B. Fructose C. Sucrose D. Mannose
85. Semen specimens should be analyzed:
A. Immediately upon receipt C. After liquefaction
B. Prior to liquefaction D. One hour after collection
86. An abnormal amount of prostatic fluid in a semen specimen will:
A. Lower the pH C. Increase the viscosity B. Raise the pH D. Decrease the
viscosity
87. Which of the following stains is used to determine sperm viability?
A. Eosin B. Methylene blue C. Papanicolaou D. Hematoxylin
88. Serum gastrin levels would be greatest in:
A. Atrophic gastritis C. Pernicious anemia B. Zollinger-Ellison syndrome D. Cancer of the
stomach
89. The preferred stimulant of gastric acidity for routine analysis is:
A. Histamine B. Histalog C. Pentagastrin D. Insulin
90. The gastric stimulant used to determine a successful vagotomy procedure is:
A. Insulin B. Pepsin C. Histamine D. Pentagastrin
91. A condition in which a patient shows no response to gastric stimulation is:
A. Pernicious anemia B. Zollinger-Ellison syn C. Ulcers D. Dibates
92. It is used as a safer alternative for testing vagal stimulation of gastric acid secretion. The
procedure also requires gastric intubation, however, instead of administering insulin, patients are
given a sandwich to chew and spit out.
A. Sweat test B. Sham feeding C. Secretin test D. Breath test
93. The test is based on the measurement of 14CO2 in expired air following the ingestion of various 14C-
labeled triglycerides. Steatorrhea from either pancreatic or other causes results in a decreased
absorption of triglycerides by the digestive system. This in turn results in a decrease in expired CO 2
derived from metabolism of triglyceride fatty acids.
A. Sweat test B. Sham feeding C. Secretin test D. Breath test
94. It is a valuable test for the differential diagnosis of malabsorption. In this procedure, a 25 g dose of
pentose is administered orally. Blood level is determined two hours later; urine excretion over a
five-hour postadministration period is also determined.
A. D-xylose test B. Sham feeding C. Secretin test D. Breath test
95. Most common cause of malabsorption in developed countries:
A. Celiac disease B. Cystic fibrosis C. Lactase deficiency D. None of these
96. It serves as the laboratory procedure for the definitive diagnosis of steatorrhea:
A. Titrimetric method of Van de Kamer C. Breath test
B. D-Xylose test D. Sweat test
97. Which of the following pairings of stool appearance and cause does not match?
A. Black, tarry: blood C. Pale, frothy: steatorrhea B. Yellow-green: barium sulfate D.
Yellow-gray: bile duct obstrcution
98. In the Apt test, maternal hemoglobin will produce a:
A. Pink solution
B. Yellow-brown precipitate after standing for 2 minutes
9

C. Yellow-brown supernatant after standing for 2 minutes


D. Red-brown supernatant after standing for 2 minutes
99. When testing for occult blood, the portion of the stool specimen used is:
A. The outer surface B. An emulsion C. The center D. The very end
100. Due to variability in bowel habits, the most representative timed fecal sample is:
A. 2-day collection B. 3-day collection C. 4-day collection D. 5-day collection

END OF EXAM

PRE-BOARD EXAMINATION IN CLINICAL MICROSCOPY (PART 2)

1. In what sequence does urine formation occur?


A. Proximal convoluted tubule, loop of Henle, distal convoluted tubule, collecting tubule
B. Glomerulus, Bowman’s space, proximal convoluted tubule, loop of Henle, distal convoluted
tubule, collecting duct
C. Bowman’s space, glomerulus, proximal convoluted tubule, loop of Henle, distal convoluted
tubule, collecting duct
D. Glomerulus, proximal convoluted tubule, distal convoluted tubule, loop of Henle, Bowman’s
space
2. While performing urinalysis, a technologist notices that the urine has a fruity odor. This
patient’s urine most likely contains:
A. Acetone B. Bilirubin C. Coliform bacilli D. Porphyrin
3. Renal clearance tests are used to evaluate which of the following parameters?
A. Concentrating ability C. Glomerular filtration rate
B. Tubular reabsorption D. Glomerular permeability
4. Which renal function is assessed using specific gravity and osmolality measurements?
A. Tubular excretion ability C. Concentrating ability B. Glomerular filtration ability
D. Tubular secretion ability
5. The color of normal urine is mainly due to the pigment:
A. Bilirubin B. Urobilin C. Urochrome D. Uroerythrin

6. To quantify formed elements such as white blood cells, red blood cells, and casts in a 12-hour
urine specimen, which of the following procedures is used?
A. Guthrie count B. Rothera count C. Addis count D, Folin-Lowry count
7. Which of the following pigments will deposit on urate and uric acid crystals to form a precipitate
described as “brick dust”?
A. Bilirubin B. Urobilin C. Uroerythrin D. Urochrome
8. A white precipitate in normal alkaline urine is most likely caused by:
A. Uric acid crystals C. Amorphous phosphate
B. Radiographic contrast media D. Amorphous urates
9. With which crystal are urinary uric acid crystals often confused?
A. Calcium pyrophosphate C. Calcium oxalate
B. Cystine D. Leucine
10

10. The method of choice for performing a specific gravity measurement of urine following
administration of x-ray contrast dyes is:
A. Reagent strip B. Refractometer C. Urinometer D. Densitometer
11. Which of the following methods for determining the urine’s specific gravity does NOT detect the
presence of urinary protein or glucose?
A. The urinometer method C. The refractometer method
B. The falling drop method D. The reagents trop method
12. The “double indicator system” employed by commercial reagent strips to determine urine pH uses
which two indicator dyes?
A. Methyl orange and bromphenol blue C. Methyl red and bromthymol blue
B. Phenol red and thymol blue D. Phenolphthalein and litmus
13. Most often, the predominant protein excreted in excess in urine is:
A. Lactoferrin B. Albumin C. δ-globulin D. Globulin
14. Bence Jones protein is:
A. An abnormal protein associated with UTI
B. An abnormal protein associated with hemolysis
C. An abnormal globulin associated with multiple myeloma
D. A normal serum protein
15. Which of the following aids in the differentiation of hemoglobinuria and hematuria?
A. Urine pH C. Microscopic examination
B. Urine color D. Leukocyte esterase test
16. Myoglobinuria is MOST likely to be noted in urine specimens from patients with which of the
following disorders?
A. Hemolytic anemias C. Myocardial infarctions
B. Lower UTI D. Paroxysmal nocturnal hemoglobinuria
17. Ammonium sulfate was added to red urine. The urine had a positive reaction for blood, but no
RBCs were seen on microscopic examination. After centrifugation, the supernatant fluid is red.
The abnormal color is caused by:
A. Pyridium B. Hemoglobin C. Porphyrins D. Myoglobin
18. Which statement about hemoglobin and myoglobin is true?
A. They are both heme-containing proteins involved in oxygen transport
B. Their presence is suspected when both the urine and the serum are colored red
C. Their presence in serum is associated with high creatine kinase values
D. They precipitate out of solution when the urine is 80 % saturated with ammonium sulfate
19. A false negative urine reaction when using a dipstick pad for hemoglobin can occur after ingestion
of:
A. Vitamin A B. Ascorbic acid C. Vitamin B6 D. Cholesterol
20. Which of the following substances if present in the urine will result in a negative Clinitest?
A. Fructose B. Lactose C. Sucrose D. Galactose
21. The glucose specificity of the “double sequential enzyme reaction” employed on reagent strip
tests is due to the use of:
A. Gluconic acid C. Glucose oxidase-peroxidase
B. Hydrogen peroxide D. Neocuproine
11

22. A 17-year-old girl decided to go on a starvation diet. After one week of starving herself, what
substance would most likely be found in her urine?
A. Protein B. Ketones C. Glucose D. Blood
23. Which of the following ketones are NOT detected by the reagent strip or tablet test?
A. Acetone C. B-hydroxybutyrate
B. Acetoacetate D. Acetone and acetoacetate
24. The ketone reagent strip and tablet tests are based on the reactivity of ketones with:
A. Ferric chloride B. Nitroprusside C. Ferric nitrate D. Nitroglycerin
25. Which of the following are characteristic urine findings from a patient with hemolytic jaundice?
A. A positive test for bilirubin and an increased amount of urobilinogen
B. A positive test for bilirubin and a decreased amount of urobilinogen
C. A negative test for bilirubin and an increased amount of urobilinogen
D. A negative test for bilirubin and a decreased amount of urobilinogen
26. Which of the following results are characteristic urine findings from a patient with an obstruction
of the bile duct?
A. A positive test for bilirubin and an increased amount of urobilinogen
B. A positive test for bilirubin and a decreased amount of urobilinogen
C. A negative test for bilirubin and an increased amount of urobilinogen
D. A negative test for bilirubin and a decreased amount of urobilinogen
27. The classic Ehrlich’s reaction is based on the reaction of urobilinogen with:
A. Diazotized dichloroaniline C. p-dichlorobenzene diazonium salt
B. p-dimethylaminobenzaldehyde D. p-aminobenzoic acid
28. Red colored-compound produced with the Watson-Schwartz test that is insoluble in chloroform
and butanol is:
A. Uroporphyrin C. Urobilinogen B. Porphobilinogen D. Protoporphyrin
29. Nitrite in urine specimen suggests the presence of:
A. White blood cells C. Bacteria
B. Ammonia D. Urea
30. Which of the following blood cells will NOT be detected by the leukocyte esterase pad because it
lacks esterases?
A. Eosinophils B. Lymphocytes C. Monocytes D. Neutrophils
31. Which of the following reagent strip tests can be affected by ascorbic acid, resulting in falsely low
or false negative results?
1. Blood 2. Bilirubin 3. Glucose 4. Nitrite

A. 1, 2 and 3 are correct C. 4 is correct


B. 1 and 3 are correct D. All are correct
32. Which of the following dyes are used in Sternheimer-Malbin stain?
A. Hematoxylin and eosin C. Methylene blue and eosin
B. Crystal violet and eosin D. Crystal violet and safranin
33. The microscopic identification of hemosiderin is enhanced when the urine sediment is stained
with:
A. Gram stain B. Hansel stain C. Sudan stain D. Prussian blue stain
12

34. When a laboratorian performs the microscopic examination of urine sediment, which of the
following are enumerated using the low-power magnification?
A. White blood cells C. Red blood cells
B. Casts D. Renal tubular cells
35. The ova of which parasite may be found in the urinary sediment?
A. Trichomonas vaginalisC. Entamoeba histolytica
B. Schistosoma haematobium D. Trichuris trichiura
36. A technologist is having trouble differentiating between red blood cells, oil droplets, and yeast
cells on a urine microscopy. Acetic acid should be added to the sediment to:
A. Lyse the yeast cells C. Dissolve the oil droplets B. Lyse the red blood cells D. Crenate the
red blood cells
37. Glitter cells are a microscopic finding of:
A. Red blood cells in hypertonic urine C. White blood cells in hypertonic urine
B. Red blood cells in hypotonic urine D. White blood cells in hypotonic urine
38. Urine sediment could have which of the following formed elements and still be considered
“normal”?
A. 5 to 10 red blood cellsC. A few bacteria B. 0 to 2 hyaline casts D. A few yeast
cells
39. Urinary casts are formed in the:
A. Distal tubules and loops of Henle C. Proximal and distal tubules B. Distal and collecting
tubules D. Proximal tubules and loops of Henle
40. Urinary casts are formed with a core matrix of:
A. Albumin C. Bence Jones protein
B. Tamm-Horsfall mucoprotein D. Transferrin
41. Which of the following crystals, when found in the urine sediment, most likely indicates an
abnormal metabolic condition?
A. Calcium oxalate C. Triple phosphate
B. Bilirubin D. Uric acid
42. Oval fat bodies are derived from:
A. Renal tubular epithelium C. Degenerated WBCs
B. Transitional epithelium D. Mucoprotein matrix
43. In a patient with nephrotic syndrome, the microscopic examination of the urine sediment
often reveals:
A. Granular casts C. Red blood cell casts
B. Waxy casts D. Leukocyte casts
44. The most frequently observed cast in urine is:
A. Red cell B. Hyaline C. WaxyD. Fatty
45. When using polarized light microscopy, which urinary sediment component exhibits Maltese
cross formation?
A. RBC B. Oval fat bodies C. Yeast D. Parasites
46. Which of the following frequently occurs following a bacterial infection of the skin or throat?
A. Acute glomerulonephritis C. Membranous glomerulonephritis
B. Chronic glomerulonephritis D. Rapidly progressive glomerulonephritis
47. Phenylketonuria is an inherited metabolic disease in which there is deficiency of an enzyme:
13

A. Phenylalanine oxidase C.Phosphoalanine hydroxylase


B. Phenylalanine hydroxylase D. Phosphoalanine phosphatase
48. Which of the following disorders is characterized by the urinary excretion of large amounts of
arginine, cystine, lysine, and ornithine?
A. Cystinosis B. Cystinuria C. Lysinuria D. Tyrosinuria
49. Which of the following hereditary diseases results in the accumulation and excretion of large
amounts of homogentisic acid?
A. Melanuria B. Alkaptonuria C. PhenylketonuriaD. Tyrosinuria
50. Which of the following substances oxidizes with exposure to air, causing the urine to turn
brown or black?
A. Porphyrin B. Melanin C. Tyrosine D. Urobilinogen
51. A disease that causes large amounts of valine, leucine and isoleucine to be excreted in urine is:
A. Hurler’s syndrome C. Maple syrup urine disease
B. Cystinuria D. Alkaptonuria
52. Increased amounts of 5-hydroxy indoleacetic acid in the urine is indicative of:
A. Platelet disorders C. Malabsorption syndromes
B. Intestinal obstruction D. Argentaffin cell tumors
53. During a lumbar puncture procedure, the first collection tube of CSF removed should be used
for:
A. Microbiology studies C. Hematology tests
B. Cytology studies D. Chemistry tests
54. All of the following proteins are normally present in the CSF, EXCEPT:
A. Transferrin B. Albumin C. Prealbumin D. Fibrinogen
55. Pleocytosis is a term used to describe:
A. A pink, orange, or yellow CSF specimen
B. An increased protein content in the CSF owing in cellular lysis
C. Inflammation and sloughing of the cells from the choroids plexus
D. An increased number of cells in the CSF
56. Regarding CSF, all of the following are indications of a traumatic tap, EXCEPT:
A. Clearing of the fluid as it is aspirated C. Xanthochromia
B. A clear supernatant after centrifugationD. Presence of clot in the sample
57. All of the following proteins are normally present in the CSF except:
A. Albumin B. Fibrinogen C. Prealbumin D. Transferrin
58. An unknown fluid can positively identified as being CSF by:
A. Determining the lactate concentration
B. Determining the albumin concentration
C. Determining the presence of oligoclonal banding on electrophoresis
D. Determining the presence of carbohydrate-deficient transferrin on electrophoresis
59. How many leukocytes are normally present in the CSF obtained from an adult?
A. 0-30 cells/uL B. 0-20 cells/uL C. 0-10 cells/uL D. 0-5 cells/uL
60. Which of the following cells may be present in small numbers in normal CSF?
A. Erythrocytes B. Macrophages C. Plasma cells D. Lymphocytes
61. Which of the following cell types predominate in CSF during a classic case of bacterial
meningitis?
14

A. Lymphocytes B. Macrophages C. Monocytes D. Neutrophils


62. Which of the following cell types predominate in CSF during a classic case of viral meningitis?
A. Lymphocytes B. Macrophages C. Monocytes D. Neutrophils
63. A web-like clot or pellicle is formed in the CSF from patients infected with:
A. Neisseria meningitidis C. Mycobacterium tuberculosis
B. Haemophilus influenzae D. Cryptococcus neoformans
64. India ink preparations and microbial antigen tests can aid in the diagnosis of:
A. Bacterial meningitis C. Viral meningitis B. Tuberculous meningitis D. Fungal
meningitis
65. Limulus Lysate test will detect the presence of:
A. Gram-positive bacteria C. Acid-fast organisms
B. Gram-negative bacteria D. All microorganisms
66. Normal CSF lactate levels (below 25 mg/dL) are commonly found in patients with:
A. Bacterial meningitis C. Tuberculous meningitis
B. Fungal meningitis D. Viral meningitis
67. In suspected case of Reye’s syndrome, it is useful to measure:
A. Glucose B. Ammonia C. Lactic acid D. Glutamine
68. Which of the following parameters directly relates to and provides a check of the spermatozoa
motility evaluation?
A. The agglutination evaluation C. The morphology assessment
B. The concentration determination D. The viability assessment
69. The concentration of which of the following substances can be used to positively identify a
fluid as seminal fluid?
A. Zinc C. Fructose
B. Citric acid D. Acid phosphatase
70. The most common cause of male infertility is:
A. Mumps B. Varicocele C. Malignancy D. Klinefelter’s syndrome
71. Which of the following is a characteristic of normal synovial fluid?
A. It forms small clots upon standing C. It is slightly turbid
B. It is dark yellow D. It is viscous
72. A synovial fluid has a high cell count and requires dilution to be counted. Which of the
following diluents should be used?
A. Dilute acetic acid C. Dilute methanol B. Normal saline D. Phosphate buffer
solution
73. Normal joint fluid is viscous because of a high concentration of:
A. δ-globulin B. Hyaluronate C. Mucin D. Neutrophils
74. Differentiation of synovial fluid crystals, based on their birefringence, is achieved using:
A. Transmission electron microscopy C. Direct polarizing microscopy
B. Compensated polarizing microscopy D. Phase contrast microscopy
75. Which of the following crystals is characteristically seen in patients with gout?
A. Cholesterol crystals
B. Monosodium urate monohydrate crystals
C. Hydroxyapatite crystals
D. Calcium pyrophosphate dihydrate crystals
15

76. Which crystal causes “pseudogout”?


A. Oxalic acid C. Calcium oxalate
B. Calcium pyrophosphate D. Cholesterol
77. Neutrophils that contain a precipitated rheumatoid factor in their cytoplasm:
A. LE cells B. Reiter cells C. Ragocytes D. Macrophages
78. What is the preferred stimulant of gastric acidity for routine analysis?
A. Histalog B. Histamine C. Insulin D. Pentagastrin
79. A gastric stimulant to determine a successful vagotomy procedure is:
A. Insulin B. Pepsin C. Histamine D. Pentagastrin
80. Serum gastrin levels would be greatest in:
A. Atrophic gastritis C. Pernicious anemia
B. Zollinger-Ellison syndrome D. Cancer of the stomach
81. A condition in which the patient shows no response to gastric stimulation is:
A. Pernicious anemia C. Ulcers
B. Zollinger-Ellison syndrome D. Diabetes
82. Select the term used to describe a decreased volume of amniotic fluid present in the amniotic
sac:
A. Ahydramnios C. Hydramnios
B. Oligohydramnios D. Polyhydramnios
83. Amniotic fluid is immediately protected from light to preserve which of the following
substances?
A. Phospholipids B. Fetal cells C. Meconium D. Bilirubin
84. Which of the following causes erythroblastosis fetalis?
A. Immaturity of the fetal liver C. Inadequate fetal surfactants
B. Decreased amount of amniotic fluid D. Maternal immunization by fetal antigens
85. The best available guide in the assessment of gestational age is the amniotic determination of:
A. Bilirubin C. Creatinine
B. Uric acid D. Protein
86. Which of the following amniotic fluid measurements is increased in neural tube disorder?
A. Lecithin/sphingomyelin ratio C. Acetylcholinesterase activity
B. Bilirubin D. Phospatidylinositol
87. Which of the following terms is another name for peritoneal fluid?
A. Synovial B. Ascitic C. Pelvic D. Abdominal
88. Thoracentesis refers specifically to the removal of fluid from the:
A. Abdominal cavity C. Peritoneal cavity
B. Pleural cavity D. Pericardial cavity
89. A pleural or peritoneal fluid amylase level 2 times higher than the serum amylase level may be
found in effusions resulting from:
A. Hepatic cirrhhosis C. Rheumatoid arthritis B. Pancreatitis D. Lymphatic obstruction
90. Bronchoalveolar lavage is useful in the detection of:
A. Tuberculosis C. Malignant melanoma
B. Pneumocystis carinii D. Klebsiella pneumoniae
91. The most abundant cell seen in a BAL is:
A. Neutrophil C. Macrophage
16

B. Bronchial epithelial cell D. Lymphocyte


92. Which of the following tests is required in order to diagnose steatorrhea?
A. Fecal fat C. Fecal occult blood
B. Fecal carbohydrates D. fecal osmolality
93. Which of the following is responsible for the characteristic color of normal feces?
A. Urobilin B. Hemoglobin C. Bilirubin D. Urobilinogen
94. Mass screening for occult blood in the feces is primarily performed to detect:
A. Colorectal cancer C. Ulcer
B. Hemorrhoids D. Malabsorption
95. Which of the following dietary substances can cause a false negative fecal occult blood slide
test?
A. Ascorbic acid B. Fish C. Red meat D. Fruits and vegetables
96. In the Apt test, maternal hemoglobin will produce a:
A. Pink solution
B. Yellow-brown precipitate after standing for 2 minutes
C. Yellow-brown supernatant after standing for 2 minutes
D. Red-brown supernatant after standing for 2 minutes
97. Due to variability in bowel habits, the most representative timed fecal sample is:
A. 2-day collection C. 4-day collection
B. 3-day collection D. 5-day collection
98. Which of the following is the indicator of choice in fecal occult blood slide tests?
A. Guaiac B. Benzidine C. Orthotoluidine D. Tetramethylbenzidine
99. Yellowish or gray caseous bodies with the size of a pinhead, found in sputum:
A. Cheesy masses C. Broncholiths
B. Bronchial casts D. Dittrich plugs
100. Eosinophils and Curschmann’s spirals may be found in which of the following conditions:
A. Bronchiectasis C. Pneumonia
B. Chronic bronchitis D. Bronchial asthma

END OF THE EXAM

PRE-BOARD EXAMINATION IN CLINICAL MICROSCOPY (PART3)

1. The fluid leaving the glomerulus has a specific gravity of:


a. 1.005 b. 1.010 c. 1.015 d. 1.020
2. An unidentified fluid is received in the laboratory with a request to determine if the fluid is urine or
another body fluid. Using routine laboratory tests, what tests would determine that the fluid is most
probably urine:
a. glucose and ketones b. urea and creatinine c. uric acid and amino acids d.
protein and amino acids
3. A patient presenting with polyuria, nocturia, polydipsia and a high urine specific gravity is exhibiting
symptoms of what disorder?
a. diabetes insipidus b. diabetes mellitus c. UTI d. uremia
17

4. An unpreserved urine specimen collected at 8 a.m. and remaining at room temperature until the
afternoon shift arrives can be expected to have:
1. decreased glucose and ketones 3. decreased pH and turbidity
2. increased bacteria and nitrite 4. increased cellular elements

a. 1, 2 and 3 b. 1, 2 and 4 c. 1 and 2 d. 4 only


5. What three changes will affect the results of microscopic examination of urine if not tested within 2
hours?
a. decreased bacteria, decreased red blood cells, decreased casts
b. increased bacteria, increased red blood cells, increased casts
c.
d.
increased bacteria, decreased red blood cells, decreased casts
decreased bacteria, increased red blood cells, increased casts
↑RBC ,
cast
6. What chemical can be used to preserve a specimen for a culture and a routine urinalysis?
a. boric acid b. formalin c. sodium fluoride d. thymol
7. The substance normally found in urine that is principally responsible for the yellow coloration of
urine is:
a. bilirubin b. melanin c. urochrome d. carotene
8. A yellow-brown specimen that produces a yellow foam when shaken can be suspected of
containing:
a. bilirubin b. carrots c. hemoglobin d. Rhubarb
9. Specimens from patients receiving treatment for UTI frequently appear:
a. clear and red b. viscous and orange c. dilute and pale yellow d.
cloudy and red
10. Under what conditions will a port-wine urine color be observed in a urine specimen?
a. the patient has eaten Clorets c. urine containe porphyrins
-

b. melanin is present d. the patient has Pseudomonas infection


11. The pH of a freshly voided urine specimen may range between:
a. 3.5 and 8.0 b. 3.5 and 9.0 c. 4.0 and 8.5 d. 4.5 and 8.0
12. A correlation exists between a specific gravity of 1.050 and a:
a. 2+ glucose b. 2+ protein c. first morning specimen d. radiographic dye
infusion
13. A urine specimen has a specific gravity of 1.008 has been diluted 1:5. The actual specific gravity is:
a. 1.008 b. 1.040 c. 1.055 d. 5.040
14. The method for determining a specific gravity that is based on the principle that the frequency of a
sound wave entering a solution changes in proportion to the density of the solution is:
a. colorimeteric b. harmonic oscillation densitometry c. refractometry d.
urinometry
15. The reagent strip used for detection of protein in urine is most reactive to:
a. albumin b. hemoglobin c. beta globulins d. gamma globulins

16. The principle of the reagent strip for glucose is based on which of the following reactions?
a. hexokinase b. copper reduction c. alkaline ferricyanide d. glucose-oxidase
peroxidase

17. The primary reagent in the reagent strip test for ketones is:
18

a. glycine b. lactose c. sodium hydroxide d. sodium


18. Ketonuria may be caused by all of the following except: nitroprusside
a. bacterial infections b. diabetic acidosis c. starvation d. vomiting
19. A speckled pattern on the blood pad of the reagent strip indicates:
a. hematuria b. hemoglobinuria c. myoglobinuria d. all of the above
20. An elevated urine bilirubin with a normal urobilinogen is indicative of:
a. cirrhosis of the liver b. hemolytic disease c. hepatitis d. biliary obstruction
21. In the Watson-Schwartz differentiation test, the substance(s) not extracted into butanol is/are:

a. urobilinogen b. porphobilinogen c. Ehrlich reactive substances d. all of the above


22. The Hoesch test is used to monitor or screen patients for the presence of:

a. urobilinogen b. nitrite c. porphobilinogen d. leukocyte esterase

23. The reagent strip for nitrite used the:

a. Greiss reaction b. Hoesch reaction c. Peroxidase reaction d. Pseudoperoxidase


reaction

24. All of the following can be detected by the leukocyte esterase reaction except:

a. neutrophils b. eosinophils c. lymphocytes d. basophils

25. Screening tests for urinary infection combine the leukocyte esterase test with the test for:
a. pH b. nitrite c. protein d. blood
26. To quantify formed elements such as white blood cells, red blood cells, and casts in a 12-hour urine
specimen, which of the following procedures is used?
a. Guthrie count b. Addis count c. Rothera count d. Folin-Lowry count
27. Which of the following reported as number per LPF?
a. RBCs b. WBCs c. crystals d. casts
28. The final phase of degeneration that granular casts undergo is represented by which of the following
casts?
a. broad b. cellular c. waxy d. fatty
29. Which of the following lipids is/are stained by Sudan III?
a. cholesterol b. neutral fats c. triglycerides d. both B and C
30. Which of the following lipids is/are capable of polarizing light?

a. cholesterol b. neutral fats c. triglycerides d. both B and C


31. The purpose of the Hansel stain is to identify:
a. neutrophils b. renal tubular cells c. eosinophils d. monocytes

32. Crenated RBCs are seen in urine that is:


a. hyposthenuric b. hypersthenuric c. highly acidic d. highly alkaline
33. The finding of dysmorphic RBCs is indicative of:
a. glomerular bleeding b. renal calculi c. traumatic injury d. coagulation
disorders
34. When pyuria is detected in a sediment, the slide should be carefully checked for the presence of:
19

a. RBCs b. bacteria c. hyaline casts d. mucus


35. The predecessor of the oval fat body is the:
a. histiocyte b. urotehlial cell c. monocyte d. renal tubular
36. The finding of yeast cells in the urine is commonly associated with: cell
a. cystitis b. diabetes mellitus c. pyelonephritis d. liver disorders
37. The majority of casts are formed in the:
a. proximal convoluted tubules c. distal convoluted tubules
b. ascending loop of Henle d. descending loop of Henle
38. A person submitting a urine specimen following a strenuous exercise routine can normally have all
of the following in the sediment except:
a. hyaline casts b. granular casts c. RBC casts d. WBC casts
39. WBC casts are primarily associated with:
a. pyelonephritis b. cystitis c. glomerulonephritis d. viral infections
40. Crystals associated with liver disease include all of the following except:
a. bilirubin b. leucine c. cystine d. tyrosine
41. Differentiation between cystitis and pyelonephritis is aided by the presence of:
a. WBC casts b. RBC casts c. bacteria d. granular casts
42. The most common composition of renal calculi is:
a. calcium oxalate b. magnesium ammonium phosphate c. cystine d. uric
acid
43. The Guthrie test is a:
a. bacterial inhibition test c. chemical procedure measured by
spectrophotometer

b. fluorometric procedure d. bacterial agglutination test


44. A mousy odor in the urine is associated with:

a. phenylketonuria b. isovaleric acidemia c. cystinuria d. Cystinosis


45. Hartnup disease is a disorder associated with the metabolism of:
a. organic acids b. tryptophan c. cystine d. phenylalanine
46. Elevated levels of 5-HIAA are associated with:
a. carcinoid tumors b. Hartnup disease c. cystinuria d. platelet
disorders
47. Hurler’s and Sanfilippo’s syndromes present with mental retardation and increased urinary:
a. porphyrins b. amino acids c. maltose d. mucopolysaccharides
48. The identification of the presence of human chorionic gonadotropin in a urine specimen indicates:
a. glomerulonephritis b. nephroyic syndrome c. diabetes mellitus
d.pregnancy
49. The functions of the CSF include all of the following except:
a. removal of metabolic wastes c. supplying nutrients to the CNS
b. producing an ultrafiltrate of plasma d. protection of the brain and spinal
50. The CSF tube labeled 3 is sent to: cord
a. the hematology department c. the microbiology department
b. the chemistry department d. the serology department
51. The CSF tube that should be refrigerated is:
20

a. tube 1 b. tube 2 c. tube 3 d. tube 4


52. A web like pellicle in a refrigerated CSF specimen is indicative of:

a. tubercular meningitis b. multiple sclerosis c. primary CNS malignancy d. viral


meningitis 53. The normal value of CSF protein is:

a. 6-8 g/dL b. 15-45 g/dL c. 6-8 mg/dL d. 15-45 mg/dL

54. CSF can be differentiated from plasma by the presence of:


a. albumin b. globulin c. prealbumin d. tau transferrin
55. In plasma, the second most prevalent protein is IgG; in CSF, the second most prevalent protein is:
a. transferring b. prealbumin c. IgA d. ceruloplasmin
56. Elevated CSF protein values can be cause by all of the following except:
a. meningitis b. multiple sclerosis c. fluid leakage d. CNS malignancy
57. The finding of oligoclonal bands in the CSF and not in the serum is seen with:
a. multiple myeloma b. CNS malignancy c. multiple sclerosis d. viral
infections
58. A CSF glucose of 15 mg/dL, WBC count of 5000, 90% neutrophils, and protein of 80 mg/dL is
suggestive of:
a. fungal meningitis b. viral meningitis c. tubercular meninigitis
d.bacterial meningitis
59. A patient with blood glucose of 120 mg/dL would have a normal CSF glucose of:
a. 20 mg/dL b. 60 mg/dL c. 80 mg/dL d. 120 mg/dL

60. The limulus lysate test on CSF is a sensitive assay for:


a. demyelinating diseases of the spinal cord c. Gram negative bacterial
endotoxin
b. cryptococcal meningitis d. Open neural tube defects
61. Pronounced reduction of CSF glucose can be seen in the following conditions, EXCEPT:
a. viral meningitis c. fungal meningitis
b. bacterial meningitis d. tuberculous meningitis
62. Regarding CSF, all of the following are indications of a traumatic tap, EXCEPT:
a. clearing of the fluid as it is aspirated c. xanthochromia
b. a clear supernatant after centrifugation d. presence of clot in the sample
63. A major CSF chemical that is measured in suspected cases of Reye’s syndrome is:
a. glucose b. glutamine c. lactate d. lactate dehydrogenase
64. Maturation of spermatozoa takes place in the:
a. Sertoli cells b. seminiferous tubules c. epididymis d. seminal vesicles

65. Enzymes for the coagulation and liquefaction of semen are produced by the:
a. seminal vesicles b. bulbourethral glands c. ductus deferens d.
prostate gland

66. The normal sperm concentration is:

a. below 20 million per microliter c. below 20 million per milliliter


b. above 20 million per milliliter d. above 20 million per microliter
21

67. Given the following information, calculate the sperm concentration: dilution 1:20; sperm counted
in 5 RBC squares on each side of the hemacytometer, 80 and 86; volume, 3 mL.
a. 80 million per milliliter c. 86 million per milliliter
b. 83 million per milliliter d. 169 million per microliter
68. All of the following are grading criteria for sperm motility except:
a. rapid straight-line movement c. no forward progression
b. rapid lateral movement d. no movement
69. The immunobead test for antisperm antibodies:
a. detects the presence of male antibodies c. determines the location of the
antisperm antibodies
b. determines the pres of IgG, IgA and IgM abs d. all of the above
70. Measurement of α-glucosidase is performed to detect a disorder of the:
a. seminiferous tubules b. epididymis c. prostate gland d. bulbourthral
gland
71. Normal synovial fluid resembles:
a. egg white b. normal serum c. dilute urine d. lipemic serum

72. Powdered anticoagulants should not be sued in tubes for synovial fluid testing because it
interferes with:
a. cell counts b. glucose tests c. crystal examination d. differentials
73. To determine if a fluid is synovial fluid, it should be mixed with:
a. sodium hydroxide b. hypotonic saline c. hyaluronidase d. acetic acid
74. When diluting a synovial fluid WBC count, all of the following are acceptable except:
a. acetic acid b. isotonic saline c, hypotonic saline d. saline with
saponin
75. Synovial fluid crystals that occur as a result of purine metabolism or chemotherapy for leukemia
are:
a. monosodium urate b. cholesterol c. calcium pyrophosphate d.
apatite
76. Crystals associated with psedogout are:
a. Monosodium urate b. calcium pyrophosphate c. apatite d.
corticosteroid

77. Neutrophils that contain precipitated rheumatoid factor in their cytoplasm are
called:

a. LE cells b. Reiter cells c. ragocytes d. Macrophages

78. A milky appearing pleural fluid is indicative of:

a. thoracic duct leakage b. chronic inflammation c. microbial infection d. both


A and B

79. Detection of CA-125 tumor marker in peritoneal fluid is indicative of:

a. colon cancer b. ovarian cancer c. gastric malignancy d. prostate cancer


80. A transudative effusion is usually caused by all of the following, EXCEPT:
22

a. congestive heart failure c. Nephrotic syndrome


b. malignancy d. Cirrhosis
81. A useful adjunct test for tuberculous pericarditis in cases with negative stains when tuberculosis is
suspected:
a. Lactate dehydrogenase activity c. Adenosine deaminase activity
b. Bilirubin level d. Lactate level
82. Amniotic fluid procedure used to detect neural tube defects is:
a. L/S ratio b. creatinine c. absorbance at 450 nm d. alpha fetoprotein
83. A significant rise in the OD of amniotic fluid at 450 nm indicates the presence of which analyte?
a. bilirubin b. lecithin c. oxyhemoglobin d. sphingomyelin
84. The foam or shake test is a screening test for which amniotic fluid substance?
a. bilirubin b. lecithin c. alpha fetoprotein d. creatinine
85. What is the fecal test that requires a 3-day specimen?
a. fecal occult blood test c. Elastase I
b. APT test d. Quantitative fecal fat testing
86. The normal brown color of feces is produced by:
a. cellulose b. pancreatic enzyme c. undigested foodstuffs d. urobilin
87. A black tarry stool is indicative of:
a. upper GI bleeding b. lower GI bleeding c. excess fat d. excess
carbohydrate
88. Chemical screening tests performed on feces include all of the following except:
a. APT test b. Clinitest c. pilocarpine iontophoresis d. trypsin
digestion
89. Annual testing for fecal occult blood has a high predictive value for the detection of:
a. colorectal cancer c. pancreatic deficiencies
b. malabsorption syndromes d. ulcers
90. What is the significance of an APT test that remains pink after addition of sodium hydroxide?
a. fecal fat is present c. fecal trypsin is present
b. fetal hemoglobin is present d. vitamin C is present
91. It is used as a safer alternative for testing vagal stimulation of gastric acid secretion. The
procedure also requires gastric intubation, however, instead of administering insulin, patients are
given a sandwich to chew and spit out.
a. Sweat test b. Sham feedingc. Secretin test d. Breath test
92. The test is based on the measurement of 14CO2 in expired air following the ingestion of various 14C-
labeled triglycerides. Steatorrhea from either pancreatic or other causes results in a decreased
absorption of triglycerides by the digestive system. This in turn results in a decrease in expired CO 2
derived from metabolism of triglyceride fatty acids.
a. Sweat test b. Sham feedingc. Secretin test d. Breath test
93. It is a valuable test for the differential diagnosis of malabsorption. In this procedure, a 25 g dose of
pentose is administered orally. Blood level is determined two hours later; urine excretion over a
five-hour postadministration period is also determined.
a. D-xylose test b. Sham feedingc. Secretin test d. Breath test
94. The most abundant cell seen in the bronchoalveolar lavage is the:
23

a. neutrophil b. ciliated columnar bronchial epithelial cell c. macrophage


d. lymphocyte
95. Bronchoalveolar lavege may be performed on patients with AIDS to detect the presence of:
a. increased helper T cells b. bacterial pneumonia c, Kaposi sarcoma d. Pneumocytis
carinii

96. The preferred stimulant of gastric acidity for routine analysis is:

a. histamine b. histalog c. pentagastrin d. insulin

97. The gastric stimulant to determine a successful vagotomy procedure is:


&

a. Cinsulin b. pepsin c. histamine d. pentagastrin


98. Serum gastrin levels would be greatest in:
a. atrophic gastritis b. Zollinger-Ellison syn c. pernicious anemia d. cancer of
the stomach
99. A condition in which a patient shows no response to gastric stimulation is:
-

a. pernicious anemia b. Zollinger-Ellison syn c. ulcers d. diabetes

100. Yellowish or gray caseous bodies with the size of a pinhead, found in sputum:

a. cheesy masses b. broncholiths c. bronchial casts d. Dittrich plugs

END OF THE EXAM


PRE-BOARD EXAMINATION IN HEMATOLOGY (PART 1)

1. Gauge of the needle indicating the largest bore is:


a. 16 gauge b. 19 gauge c. 21 gauge d. 23 gauge
2. What is the anticoagulant of choice for the osmotic fragility test?
a. heparin b. double oxalate c. EDTA d. potassium oxalate
3. Hemoglobin migration pattern on cellulose acetate from point of application to anode is:
a. C < F < A2 < A < S c. C and A2 < S < F < A
b. C < S < A and A2 < F d. C < A < F < S < A2
4. Insufficient centrifugation will result in:
a. a false increase in hematocrit value c. no effect in hematocrit value
b. a false decrease in hematocrit value d. all of the above, depending on the patient
5. Calculate the mean cell hemoglobin concentration (MCHC) using the following values:
Hgb: 15 g/dL (150 g/L) RBC 4.50 x 106/µL (4.50 x 1012/L)
Hct: 47% (0.47)
a. 9.5% b. 10.4% c. 31.9% d. 33.3%
6. A 7-mL EDTA tube is received in the laboratory containing only 2 mL of blood. If the laboratory is using manual
techniques, which of the following tests will most likely be erroneous?
a. WBC count b. hemoglobin c. hematocrit d. none of these
7. A decreased OFT would be associated with which of the following conditions?
a. sickle cell anemia c. haemolytic disease of the newborn
b. hereditary spherocytosis d. acquired haemolytic anemia
8. What effect would using a buffer at pH 6.0 have on Wright’s-stained smear?
a. red cell would be stained too pink c. red cells would be stained blue
b. white cell cytoplasm would be stained too blue d. red cells would lyse on the slide
9. Which of the following erythrocyte inclusions can be visualized with supravital stain but cannot be detected on a
Wright-stained blood smear?
a. basophilic stippling b. Heinz bodies c. Howell-Jolly bodies d. siderotic granules
10. A Miller disk is an ocular device used to facilitate counting of:
a. platelets b. reticulocytes c. sickle cells d. N-RBCs
11. RBC indices obtained on a patient are as follows: MCV 88 fL; MCH 30 pg; MCHC 34%. The RBCs on the peripheral
smear would appear:
a. microcytic, hypochromic c. normocytic, normochromic
b. microcytic, normochromic d. normocytic. hypochromic
12. All of the following may influence the ESR, except:
a. blood drawn into a sodium citrate tube c. plasma proteins
b. anisocytosis, poikilocytosis d. calibre of the tube
13. What staining method is used most frequently to stain and count reticulocytes?
a. immunofluorescence c. Romanowsky staining
b. supravital staining d. cytochemical staining
14. The Coulter principle for counting of cells is based upon the fact that:
a. isotonic solutions conduct electricity better than cells do
b. conductivity varies proportionally to the number of cells
c. cells conduct electricity better than saline does
d. isotonic solutions cannot conduct electricity
15. A correction is necessary for WBC counts when nucleated RBCs are seen on the peripheral smear because:
a. the WBC count would be falsely lower c. N-RBCs are counted as leukocytes
b. the RBC count is too low d. N-RBCs are confused with giant platelets
16. Using a Coulter counter analyzer, an increased RDW should correlate with:
a. spherocytosis b. anisocytosis c. leukocytosis d. presence of N-RBCs
17. Given the following values, which set of red blood cell indices suggests spherocytosis?
a. MCV 76 fL, MCH 19.9 pg, MCHC 28.5% c. MCV 80 fL, MCH 36.5 pg, MCHC 38.0%
b. MCV 90 fL, MCH 30.5 pg, MCHC 32.5% d. MCV 81 fL, MCH 29.0 pg, MCHC 34.8%
18. Which of the following is considered a normal hemoglobin?
a. carboxyhemoglobin b. methemoglobin c. sulfhemoglobin d. deoxyhemoglobin
2

19. Which condition will shift the oxyhemoglobin dissociation curve to the right?
a. acidosis b. alkalosis c. hb S or C d. multiple blood transfusions
20. In which stage of erythrocytic maturation does hemoglobin formation begin?
a. reticulocyte b. pronormoblast c. basophilic normoblast d.polychromatophilic normoblast
21. Which of the following can shift the hemoglobin oxygen dissociation curve to the right?
a. increase 2,3 DPG b. acidosis c. hypoxia d. all of these
22. Which of the following hemoglobin configurations is characteristic of hemoglobin H?
a. γ4 b. α2 γ2 c. β4 d. α2 β2
23. Autoagglutination of red cells at room temperature can result in which of the following?
a. low RBC count b. high MCV c. low hematocrit d. all of these
24. Which of the following organs is responsible for the “pitting process” for RBCs?
a. liver b. spleen c. kidney d. lymph nodes
25. Which of the following disorders has an increase in osmotic fragility?
a. iron deficiency anemia c. hereditary stomatocytosis
b. hereditary elliptocytosis d. hereditary spherocytosis
26. What is the major hemoglobin found in the RBCs of patients with sickle cell trait?
a. hgb S b. hgb F c. hgb A2 d. Hgb A
27. Select the amino acid substitution that is responsible for sickle cell anemia?
a. lysine is substituted for glutamic acid at the sixth position of the alpha chain
b. valine is substituted for glutamic acid at the sixth position of the beta chain
c. valine is substituted for glutamic acid at the sixth position of the alpha chain
d. glutamine is substituted for glutamic acid at the sixth position of the beta chain
28. All of the following are usually found in hemoglobin C disease, except:
a. Hemoglobin C crystals c. lysine substituted for glutamic acid at sixth position of β-chain
b. Target cells d. fast mobility of hemoglobin C at pH 8.6
29. Which of the following hemoglobins migrates to the same position as hgb A 2 at pH 8.6?
a. hgb H b. hgb F c. hgb C d. hgb S
30. Which of the following electrophoretic results is consistent with a diagnosis of sickle cell trait?
a. hgb A: 40%, hgb S 35%, hgb F 5% c. hgb A: 0%, hgb A2 5%, hgb F 95%
b. hgb A: 60%, hgb S 40%, hgb A2 2% d. hgb A: 80%, hgb S 10%, hgb A2 10%
31. Which of the following is true of paroxysmal nocturnal hemoglobinuria (PNH)?
a. it is an acquired hemolytic anemia c. it is inherited as an autosomal dominant trait
b. it is inherited as a sex-linked trait d. it is inherited as an autosomal recessive trait
32. Hemolytic uremic syndrome (HUS) is characterized by all of the following, except:
a. hemorrhage b. thrombocytopenia c. hemoglobinuria d. reticulocytopenia
33. An autohemolysis test is positive in all of the following except:
a. G6PD deficiency b. HS c. pyruvate kinase deficiency d. PNH
34. Which antibody is associated with paroxysmal cold hemoglobinuria (PCH)?
a. anti-I b. anti-i c. anti-M d. anti-P
35. All of the following are associated with hemolytic anemia except:
a. methemoglobinemia c. hemoglobinemia
b. hemoglobinuria d. increased haptoglobin
36. Bite cells are usually seen in patients with:
a. Rh null trait b. CGD c. G6PD deficiency d. PK deficiency
37. The morphological classification of anemias is based on which of the following?
a. M:E ratio b. Prussian blue stain c. RBC indices d. reticulocyte count
38. Microangioathic hemolytic anemia is characterized by:
a. target cells and Cabot rings c. Pappenheimer bodies and basophilic stipplings
b. toxic granules and Dohle bodies d. schistocytes and nucleated RBCs
39. Which antibiotic is most often implicated in the development of aplastic anemia?
a. sulfonamides b. penicillin c. tetracycline d. chloramphenicol
40. Which of the following conditions may produce spherocytes in a peripheral smear?
3

a. Pelger-Huet anomaly c. autoimmune hemolytic anemia


b. perncious anemia d. sideroblastic anemia
41. Reticulocytosis usually indicates:
a. response to inflammation c. aplastic anemia
b. neoplastic process d. red cell regeneration
42. The OFT result in a patient with thalassemia major would most likely be:
a. increased b. decreased c. normal d. all of these
43. Iron deficiency anemia may be distinguished from anemia of chronic infection by:
a. serum iron level c. red cell indices
b. red cell morphology d. total iron binding capacity
44. Which anemia has red cell morphology similar to that seen in IDA?
a. sickle cell anemia b. thalassemia syndrome c. pernicious anemia d. HS
45. Which morphological classification is characteristic of megaloblastic anemia?
a. normocytic, normochromic c. macrocytic, hypochromic
b. microcytic, normochromic d. macrocytic, normochromic
46. All of the following are characteristics of megaloblastic anemia except:
a. pancytopenia c. hypersegmented neutrophil
b. elevated reticulocyte count d. macrocytic erythrocyte indices
47. Which of the disorders below causes ineffective erythropoiesis?
a. G6PD deficiency b. liver disease c. Hgb C disease d. pernicious anemia

48. Which of the following may be seen in the peripheral blood smear of a patient with obstructive liver disease?
a. schistocytes b. macrocytes c. Howell-Jolly bodies d. microcytes
49. The macrocytes typically seen in megaloblastic processes are:
a. creascent-shaped b. teardrop-shaped c. ovalocytic d. pencil-shaped
50. Which inclusions may be seen in leukocytes?
a. Dohle bodies b. basophilic stippling c. malarial parasite d. Howell-Jolly bodies
51. Which of the following is contained in the primary granules of the neutrophil?
a. lactoferrin b. myeloperoxidase c. histamine d. alkaline phosphatase
52. The morphological characteristic associated with Chediak-Higashi syndrome is:
a. pale blue cytoplasmic inclusiosns c. small, dark staining granules and condensed nuclei
b. giant lysosomal granules d. nuclear hyposegmentation
53. Auer rods may be seen in all of the following except:
a. acute myelomonocytic leukemia (M4) c. acute myeloid leukemia without maturation (M1)
b. acute lymphoblastic leukemia d. acute promyelocytic leukemia (M3)
54. Which type of anemia is usually present in a patient with acute leukemia?
a. microcytic, hyperchromic c. normocytic, normochromic
b. microcytic, hypochromic d. macrocytic, normochromic
55. In leukemia, which term describes a peripheral blood finding of leukocytosis with a shift to the left, accompanied by
occasional nucleated red cells?
a. megaloblastosis b. dysplasia c. leukoerythroblastosis d. none of these
56. DIC is most often associated with which of the following types of acute leukemia?
a. acute myeloid leukemia without maturation c. acute myelomonocytic leukemia
b. acute promyelocytic leukemia d. acute monocytic leukemia
57. A peripheral smear shows 75% blasts. These stain positive for both Sudan black B (SBB) and peroxidase (Px). Given
these values, which of the following disorders is most likely?
a. AML b. CML c. AUL d. ALL
58. Sodium fluoride may be added to the naphthyl ASD acetate (NASDA) esterase reaction. The fluoride is added to
inhibit a positive reaction with:
a. megakaryocytes b. monocytes c. erythrocytes d. granulocytes
59. In essential thrombocythemia, the platelets are:
a. increased in number and functionally abnormal c. decreased number and functional
4

b. normal in number and functionally abnormal d. decreased in number and functionally abnormal
60. Which of the following cells is considered pathognomonic for Hodgkin’s disease?
a. Niemann-Pick cells b. reactive lymphocytes c. flame cells d. Reed-Sternberg cells
61. In myelofibrosis, the characteristic abnormal red blood cell morphology is that of:
a. target cells b. schistocytes c. teardrop cells d. ovalocytes
62. PV is characterized by:
a. increased plasma volume c. decreased oxygen saturation
b. pancytopenia d. absolute increase in total red cell mass
63. The erythrocytosis seen in relative polycythemia occurs because of:
a. decreased arterial oxygen saturation c. increased erythropoietin levels
b. decreased plasma volume of circulating blood d. increased erythropoiesis in the bone marrow
64. The leukocyte alkaline phosphatase stain of a patient gives the following results: 10 (0); 48(1+); 38(2+); 3(3+); 1(4+).
Calculate the LAP score:
a. 100 b. 117 c. 137 d. 252
65. CML is distinguished from leukomoid reaction by which of the following?
a. CML low LAP; leukomoid high LAP c. CML high WBC; leukomoid normal WBC
b. CML high LAP; leukomoid low LAP d. CML high WBC; leukomoid high WBC
66. What influence does the Philadelphia chromosome have on the prognosis of patients with chronic myelocytic
leukemia?
a. it is not predictive c. prognosis is worse is Ph1 is present
b. prognosis is better if Ph is present d. disease usually transforms into AML when Ph 1 is present
1

67. What is the characteristic finding is seen in the peripheral smear of a patient with multiple myeloma?
a. microcytic hypochrmic cells c. rouleaux
b. intracellular inclusion bodies d. hypersegmented neutrophils
68. In which of the following conditions does LAP show the least activity?
a. leukomoid recations b. idiopathic myelofibrosis c. PV d. CML
69. Cells that exhibit a positive stain with acid phosphatase and are not inhibited with tartaric acid are characteristically
seen in:
a. infectious mononucleosis c. hairy cell leukemia
b. infectious lymphocytosis d. T-cell acute lymphoblastic leukemia
70. The anticoagulant of choice for most routine coagulation studies is:
a. sodium oxalate b. sodium citrate c. EDTA d. heparin
71. Which ratio of anticoagulant to blood is correct for coagulation procedures?
a. 1:4 b. 1:5 c. 1:9 d. 1:10
72. What reagents are used in the PT test?
a. thromboplastin and sodium chloride c. thromboplastin and calcium
b. thromboplastic and potassium chloride d. actin and calcium chloride
73. Which test would be abnormal in a patient with Stuart-Prower factor (factor X) deficiency?
a. PT only b. APTT only c. PT and APTT d. thrombin time
74. Which clotting factor is not measured by PT and APTT tests?
a. factor VIII b. factor IX c. factor V d. factor XIII
75. Which coagulation test(s) would be abnormal in a vitamin K-deficient patient?
a. PT only b. PT and APTT c. fibrinogen level d. thrombin time
76. Which of the following is correct regarding the international normalized ratio (INR)?
a. it uses the international sensitivity ratio(ISR) c. standardizes the APTT results
b. standardizes PT results d. used to monitor heparin therapy
77. Which protein is the primary inhibitor of the fibrinolytic system?
a. protein C b. protein S c. α2 antiplasmin d. α2 macroglobulin
78. A protein that plays a role in both coagulation and platelet aggregation is:
a. factor I b. factor VIII c. factor IX d. factor XI
79. Aspirin prevents platelet aggregation by inhibiting the action of which enzyme?
a. phospholipase b. cyclo-oxygenase c. thromboxane A2 synthetase d. prostacyclin synthetase
5

80. Bernard-Soulier syndrome is associated with:


a. decreased bleeding time c. thrombocytopenia and giant platelets
b. decreased factor VIII assay d. abnormal platelet aggregation to ADP
81. When performing platelet aggregation studies, which set of platelet aggregation result would most likely be associated
with Bernard-Soulier syndrome?
a. normal platelet aggregation to collagen, ADP and ristocetin
b. normal platelet aggregation to collagen, ADP, epinephrine; decreased aggregation to ristocetin
c. normal platelet aggregation to epinephrine and ristocetin; decreased aggregation to collagen and ADP
d. normal platelet aggregation to epinephrine, ristocetin and collagen; decreased aggregation to ADP
82. Which set or platelet responses would most likely be associated to Glanzmann’s thrombasthenia?
a. normal platelet aggregation to ADP and ristocetin; decreased aggregation to collagen
b. normal platelet aggregation to collagen; decreased aggregation to ADP and ristocetin
c. normal platelet aggregation to ristocetin; decreased aggregation to collagen, ADP and epinephrine
d. normal platelet aggregation to ADP; decreased aggregation to collagen and ristocetin
83. Storage pool deficiencies are defects of:
a. platelet adhesion b. platelet aggregation c. platelet granules d. platelet production
84. Hereditary hemorrhagic telangiectasia is a disorder of:
a. platelets b. clotting proteins c. fibrinolysis d. connective tissue
85. Which defect characterizes Gray’s syndrome?
a. platelet adhesion defect c. alpha granule defect
b. dense granule defect d. coagulation defect
86. Normal serum contains:
a. factor I b. factor V c. factor VIII d. factor VII
87. In the APTT procedure the time taken for clot formation is measured after the addition of:
a. tissue thromboplastin b. calcium chloride c. phospholipid d. activator
88. A prolonged APTT and PT are corrected when mixed with normal plasma. Which factor is most likely deficient?
a. V b. VIII c. IX d. XI
89. Refer to the following results:
PT prolonged APTT prolonged platelet count decreased bleeding time increased
Which disorder may be indicated?
a. factor VIII deficiency b. vWD c. DIC d. factor IX deficiency
90. The following results were obtained on a patient: prolonged bleeding time, normal platelet count, normal PT, and
prolonged APTT. Which of the following disorders is most consistent with these results?
a. hemophilia A b. hemophilia B c. vWD d. Glanzmaann’s thrombasthenia
91. Refer to the following results:
PT normal APTT prolonged Bleeding time increased
Platelet count normal Platelet aggreagation to ristocetin abnormal
a. factor VIII deficiency b. DIC c. vWD d. factor IX deficiency
92. Which results are associated with hemophilia A?
a. prolonged APTT, normal PT c. prolonged PT, normal APTT
b. prolonged PT and APTT d. normal PT and APTT
93. Which of the following test is abnormal in Hemophilia B
a. platelet count b. BT c. PT d. APTT
94. Normal PT and APTT results in a patient with poor wound healing may be associated with:
a. factor VII deficiency b. factor VIII deficiency c. factor XII deficiency d. factor XIII deficiency
95. The lupus anticoagulant is directed against:
a. factor VIII b. factor IX c. factor X d. phospholipid
96. What test is used to monitor heparin therapy?
a. INR b. APTT c. PT d. none of these
97. What test is commonly used to monitor warfarin therapy?
a. INR b. APTT c. TT d. BT
98. Which clotting factors (cofactors) are inhibited by protein S?
6

a. V and X b. Va and VIIIa c. VIII and IX d. VIII and X


99. Which of the following tests is most likely to be abnormal in patients taking aspirin?
a. platelet morphology b. platelet count c. bleeding time d. prothrombin time
100. A prolonged thrombin time and a normal reptilase time are indicative of:
a. afibrinogenemia b. hypofibrinogenemia c. aspirin therapy d. heparin therapy

END OF THE EXAM


7

PRE-BOARD EXAMINATION IN HEMATOLOGY (PART 2)

1. Platelet alpha granules contain:


1. Platelet factor 4 3. Platelet derived growth factor (PDGF)
2. Beta thromboglobulin 4. Fibrinogen
A. 1 and 3 C. 1, 2 and 3
B. 2 and 4 D. 1, 2, 3 and 4
2. The activated partial thromboplastin time (APTT) is prolonged in:
1. Hemophilia B 3. Parahemophilia
2. Hemophilia A 4. Thrombocytopenia
A. 1, 2 and 3 C. 2 and 4
B. 1 and 3 D. Only 4
3. Bleeding disorders associated with vascular abnormality:
1. Hemorrhagic telangiectasia 3. Scurvy
2. Ehlers-Danlos syndromes 4. Senile purpura
A. 1 and 3 C. 1, 2 and 3
B. 2 and 4 D. 1, 2, 3 and 4
4. The test reagent in PT contains which of the following substance(s)?
1. Calcium ions 3. Tissue thromboplastin
2. Kaolin 4. Celite
A. 1, 2 and 3 C. 2 and 4
B. 1 and 3 D. Only 4
5. The test reagent in APTT contains which of the following substance(s)?
1. Citrated plasma 3. Tissue thromboplastin
2. Calcium ions 4. Phospholipids
A. 1, 2 and 3 C. 2 and 4
B. 1 and 3 D. Only 4
6. Bleeding disorder(s) in which platelets fail to aggregate with ristocetin:
1. von Willebrand’s disease 3. Bernard-Soulier syndrome
2. Glanzmann’s disease 4. Storage pool disease
A. 1, 2 and 3 C. 2 and 4
B. 1 and 3 D. Only 4
7. The fibrinogen group of coagulation factors is:
1. Present in serum 3. Adsorbed by barium sulfate
2. Not vitamin K dependent 4. Consumed during coagulation
A. 1, 2 and 3 C. 2 and 4
B. 1 and 3 D. Only 4
8. Thrombocytosis may be associated with:
1. Post-splenectomy 3. Polycythemia vera
2. DIC 4. Megaloblastic anemia
A. 1, 2 and 3 C. 2 and 4
B. 1 and 3 D. Only 4
9. Which coagulation factor(s) is(are) removed by barium sulfate or aluminum hydroxide?
1. Factor II 3. Factor IX
2. Factor VIII 4. Factor I
A. 1, 2 and 3 C. 2 and 4
B. 1 and 3 D. Only 4
10. Coagulation factor(s) affected by coumarin drugs is(are):
1. II 3. IX
2. VII 4. X
A. 1 and 3 C. 1, 2 and 3
B. 2 and 4 D. 1, 2, 3 and 4
11. The hemorrhagic problems associated with scurvy are due to a deficiency of which of the following:
A. Vitamin C C. Vitamin K
B. Prothrombin D. Protein C
12. The number of platelets an average megakaryocyte generates is approximately:
A. 25 - 50 C. 200 - 500
B. 200 - 500 D. 2000 - 4000
13. Which of the following is NOT a cause of thrombocytopenia?
A. Splenomegaly C. Increased thrombopoietin
8

B. Chemotheraphy D. Aplastic anemia


14. Which of the following is NOT a normal maturation stage for platelets?
A. Megakaryoblast C. Micromegakaryocyte
B. Promegakaryocyte D. Megakaryocyte
15. The recommended type of microscopy for the performance of manual platelet counts is:
A. Electron C. Light
B. Darkfield D. Phase contrast
16. Which of the following is not synthesized in the liver?
A. Protein C C. von Willebrand factor
B. Plasminogen D. All of these
17. Which of the following will NOT cause the thrombin time to be prolonged?
A. Fibrin degradation products C. Factor I deficiency
B. Heparin D. Factor II deficiency

18. A patient on therapeutic warfarin will most likely have a(an):


A. Normal PT/INR, increased APTT, prolonged bleeding time, low platelet count
B. Increased PT/INR, increased APTT, normal bleeding time, normal platelet count
C. Normal PT/INR, normal APTT, normal bleeding time, normal platelet count
D. Increased PT/INR, normal APTT, prolonged bleeding time, low platelet count
19. The activity of the lupus anticoagulant and anticardiolipin antibodies appears to be directed against:
A. Factor V C. Factor IX
B. Factor VIII D. Phospholipid
20. Thrombocytosis is characteristic of:
A. DIC C. Polycythemia vera
B. Splenomegaly D. Idiopathic thrombocytopenic purpura
21. If a physician suspects a qualitative platelet defect, the most useful test to order is the:
A. Platelet count C. Urea solubility test
B. Prothrombin time D. Bleeding time
22. Reversal of heparin overdose can be achieved by administration of:
A. Vitamin K C. Antithrombin
B. Protamine sulfate D. Warfarin
23. The prothrombin time will detect deficiencies in which pathway(s)?
A. Extrinsic C. Intrinsic
B. Extrinsic and common D. Intrinsic and common
24. Relative polycythemia is characterized by:
1. Increase total red cell mass 3. Normal plasma volume
2. Normal total red cell mass 4. Decreased plasma volume
A. 1, 2 and 3 C. 2 and 4
B. 1 and 3 D. Only 4
25. Microcytic, hypochromic anemia:
1. Aplastic anemia 3. Pernicious anemia
2. Acute blood loss 4. Chronic blood loss
A. 1, 2 and 3 C. 2 and 4
B. 1 and 3 D. Only 4
26. Most patients with beta thalassemia have:
1. Normocytic, normochromic anemia 3. Decreased hemoglobin F
2. Occasional target cells 4. Increased hemoglobin A2
A. 1, 2 and 3 C. 2 and 4
B. 1 and 3 D. Only 4
27. Peripheral blood picture of megaloblastic anemia:
1. Hypersegmented neutrophil 3. Oval macrocytes
2. Thrombocytopenia 4. Leukopenia
A. 1 and 3 C. 1, 2 and 3
B. 2 and 4 D. 1, 2, 3 and 4
28. Laboratory findings in hereditary spherocytosis include:
1. Increased autohemolysis corrected by glucose 3. Reticulocytosis
2. Decreased osmotic fragility 4. Positive direct antiglobulin test
A. 1, 2 and 3 C. 2 and 4
B. 1 and 3 D. Only 4
9

29. Reticulocytes:
1. Immature red cells 3. Increased after hemorrhage
2. Contain remnants of RNA 4. Stained with Romanowsky stains
A. 1, 2 and 3 C. 2 and 4
B. 1 and 3 D. Only 4
30. Hemolytic anemias are associated with:
1. Increase in reticulocytes 3. Reduced RBC survival
2. Increase serum haptoglobin 4. Decrease free hemoglobin
A. 1, 2 and 3 C. 2 and 4
B. 1 and 3 D. Only 4
31. The hemoglobins which can be differentiated by using only cellulose acetate electrophoresis at pH 8.6 are:
1. D and S 3. C and E
2. O and C 4. A1 and H
A. 1, 2 and 3 C. 2 and 4
B. 1 and 3 D. Only 4
32. Heinz bodies are formed in which of the following conditions?
1. G6PD deficiency 3. Presence of unstable hemoglobin
2. Hereditary spherocytosis 4. Microangiopathic hemolytic anemia
A. 1, 2 and 3 C. 2 and 4
B. 1 and 3 D. Only 4
33. Which of the following red cell inclusions cannot be seen by Romanowsky stain?
1. Basophilic stipplings 3. Pappenheimer bodies
2. Howell-Jolly bodies 4. Heinz bodies
A. 1, 2 and 3 C. 2 and 4
B. 1 and 3 D. Only 4

34. The structural proteins of red cell is made up of:


1. Spectrin 3. Actin
2. Glycophorins 4. Transport protein
A. 1, 2 and 3 C. 2 and 4
B. 1 and 3 D. Only 4
35. Abnormal osmotic fragility test:
1. Immediately after acute hemorrhage 3. Aplastic anemia
2. Thalassemia 4. Hereditary spherocytosis
A. 1 and 3 C. 1, 2 and 3
B. 2 and 4 D. 1, 2, 3 and 4
36. What is the first type of cell produced by the developing embryo?
A. Erythrocyte C. Lymphocyte
B. Granulocyte D. Thrombocyte
37. In an adult, what are the two best areas for obtaining active bone marrow by aspiration?
A. Vertebra, tibia C. Anterior iliac crest, tibia
B. Sternum, vertebra D. Posterior iliac crest, sternum
38. Antigen-independent lymphopoiesis occurs primary lymphoid tissue located in the:
A. Liver and kidney C. Peyer’s patches and spleen
B. Spleen and lymph nodes D. Thymus and bone marrow
39. Programmed cell death is called:
A. Necrosis C. Cellular senescence
B. Apoptosis D. Terminal differentiation
40. Interleukin and colony stimulating factors are cytokines produced by:
A. B lymphocytes and erythrocytes C. Monocytes and T lymphocytes
B. erythrocytes and thrombocytes D. Neutrophils and monocytes
41. What is the approximate total blood volume in an adult?
A. 1 L C. 6 L
B. 2 L D 12 L
42. The largest hematopoietic cells in normal bone marrow are:
A. Osteoblasts C. Megakaryocytes
B. Osteoclasts D. Plasma cells
43. As most blood cells mature, which of the following is characteristic?
A. Cell diameter increases C. nuclear chromatin becomes less condensed
10

B. Nucleus to cytoplasm ratio decreases D. Basophilia of the cytoplasm increases


44. Which of the following depicts the structure of the hemoglobin molecule?
A. Two heme groups, two globin chains C. Two heme groups, four globin chains
B. Four heme groups, two globin chains D. Four heme groups, four globin chains
45. The majority of iron found in an adult is a constituent of:
A. Ferritin C. Hemoglobin
B. Myoglobin D. Peroxidase
46. Howell-Jolly bodies are composed of:
A. DNA C. Reticulum
B. Iron D. RNA
47. When spherocytes are reported, what is observed on the peripheral blood smear?
A. Red cells without a central pallor C. Red cells with sharp projections
B. Red cells with blunt projections D. Red cells with rod-shaped crystals
48. Rouleaux of red blood cells when seen in the monolayer of a blood smear is characteristic of:
A. Hypersplenism C. Cold agglutinin disease
B. Hypogammaglobulinemia D. Multiple myeloma
49. Which of the following factors will result in an immediate increase in oxygen delivery to the tissues?
A. Increased pH C. High altitudes
B. Increased hb binding of 2,3-BPG D. Increased renal release of erythropoietin
50. Which of the following red blood cell precursors is the last stage to undergo mitosis?
A. Pronormoblast C. Polychromatophilic normoblast
B. Basophilic normoblast D. Orthochromic normoblast
51. Defective nuclear maturation commonly results in the production of red cells that are:
A. Normocytic C. Macrocytic
B. Hypochromic D. Microcytic
52. Which of the following conditions is NOT usually associated with marked reticulocytosis?
A. Four days after a major hemorrhage C. Sickle cell anemia
B. Drug-induced AIHA D. Pernicious anemia
53. Hereditary stomatocytosis is manifested physiologically by changes in:
A. Hemoglobin oxygen affinity C. Efficiency of hemoglobin reduction
B. Membrane cation permeability D. Glycolytic ATP production
54. Which of the following values can be sued to indicate the presence of a hemolytic anemia?
A. Hemoglobin level C. Erythrocyte count
B. Hematocrit level D. Reticulocyte count
55. What causes the hemolytic process in glucose-6-phosphate dehydrogenase deficiency following oxidant exposure?
A. Coating of red cells by antibody C. Complement attachment
B. Osmotic pressure changes D. Precipitation of denatured hemoglobin

56. Which of the following is an acquired red cell membrane defect that result in increased sensitivity to complement
binding?
A. March hemoglobinuria C. Paroxysmal cold hemoglobinuria
B. Paroxysmal nocturnal hemoglobinuria D. Methemoglobinemia
57. An increase in erythropoietin is NOT a normal compensating mechanism in which of the following conditions?
A. Renal tumors C. Cardiovascular disease
B. Heavy smoking D. Pulmonary disease
58. The LAP activity will usually be increased in:
1. Chronic myelogenous leukemia 3. Paroxysmal nocturnal hemoglobinuria
2. Third trimester of pregnancy 4. Polycythemia vera
A. 1, 2 and 3 C. 2 and 4
B. 1 and 3 D. Only 4
59. In differentiating a neutrophilic leukomoid reaction from chronic granulocytic leukemia, which of the following is(are)
helpful:
1. LAP score 3. Chromosome studies
2. Bone marrow cellularity 4. Sudan black stain
A. 1, 2 and 3 C. 2 and 4
B. 1 and 3 D. Only 4
60. The neutrophil mitotic pool includes:
1. Promyelocytes 3. Myelocytes
2. Myeloblasts 4. Metamyelocytes
A. 1, 2 and 3 C. 2 and 4
11

B. 1 and 3 D. Only 4
61. The neutrophil post-mitotic pool includes:
1. Promyelocytes 3. Myelocytes
2. Metamyelocytes 4. Bands
A. 1, 2 and 3 C. 2 and 4
B. 1 and 3 D. Only 4
62. FAB classification of myelodysplastic syndrome (MDS) includes:
1. Refractory anemia 3. Refractory anemia with excess blasts
2. Refractory anemia with ringed sideroblasts 4. Chronic myelomonocytic leukemia
A. 1 and 3 C. 1, 2 and 3
B. 2 and 4 D. 1, 2, 3 and 4
63. Tartrate-resistant acid phosphatase (TRAP) activity is diagnostic of:
1. Acute lymphoblastic leukemia 3. Chronic lymphocytic leukemia
2. Hodgkin’s lymphoma 4. Hairy cell leukemia
A. 1, 2 and 3 C. 2 and 4
B. 1 and 3 D. Only 4
64. The most mature granulocyte precursor that can undergo mitosis is the:
A. Myeloblast C. Myelocyte
B. Promyelocyte D. Metamyelocyte
65. Vasodilation and bronchostriction are the result of dgranulation by which of the following blood cells?
A. Eosinophils C. Neutrophils
B. Monocytes D. Basophils
66. In patients with infectious mononucleosis, which blood cells are infected by the causative agent?
A. Monocytes C. B lymphocytes
B. T lymphocytes D. Histiocytes
67. In which of the following are eosinophils NOT increased?
A. Cushing syndrome C. Skin disorders
B. Allergic disorders D. Parasitic infection
68. A Gaucher cell is best described as a macrophage with:
A. Wrinkled cytoplasm due to an accumulation of glucocerebroside
B. Foamy cytoplasm filled with unmetabolized sphingomyelin
C. Pronounced vacuolization and deposits of cholesterol
D. Abundant cytoplasm containing storage iron and cellular remnants
69. Which of the following is NOT classified as a myeloproliferative disorder?
A. Polycythemia vera C. Multiple myeloma
B. Essential thromcythemia D. Chronic myelogenous leukemia
70. In which of the following would an absolute monocytosis NOT seen?
A. Tuberculosis C. Collagen disorders
B. Infectious mononucleosis D. Recovery stage of acute bacterial infection
71. Alder-Reilly anomaly is an abnormality of:
A. Lysosomal fusion C. Oxidative metabolism
B. Nuclear maturation D. Mucopolysaccharide metabolism
72. What is the key diagnostic test for Hodgkin lymphoma?
A. Bone marrow biopsy C. Spinal tap
B. Lymph node biopsy D. Skin biopsy
73. The presence of both immature neutrophils and nucleated erythrocytes in the peripheral blood is most accurately
called:
A. Neutrophilic left shift C. Neutrophilic leukomoid reaction
B. Regenerative left shift D. Leukoerythroblastic reaction

74. Which of the following is NOT a characteristic finding in polycythemia vera?


A. Blood pancytosis C. Increased erythropoietin level
B. Increased red cell mass D. Increased blood viscosity
75. Which of the following can differentiate metamyelocyte from other stages of granulocyte maturation?
A. Presence of specific granules C. Absence of nucleoli
B. Indentation of the nucleus D. Color of cytoplasm
76. What combination of reagents is used to measure hemoglobin?
A. Hydrochloric acid and p-dimethyl-aminobenzaldehyde
B. Potassium ferricyanide and potassium cyanide
C. Sodium bisulfate and sodium metabisulfite
12

D. Sodium citrate and hydrogen peroxide


77. The slowest moving hemoglobin(s) on an alkaline electrophoresis at pH 8.6 is (are):
A. A C. F
B. A2, C, E and O D. S, D and G
78. If 60 reticulocytes are counted in 1000 red blood cells, what is the reticulocyte count?
A. 0.06% C. 0.6%
B. 6 % D. 60.0 %
79. What is the depth between the counting platform and the coverslip on a hemacytometer?
A. 0.01 mm C. 1.00 mm
B. 0.10 mm D. 0.1 cm
80. Side angle scatter in a laser-based cell counting system is used to measure:
A. Cell size C. Cell number
B. Cytoplasmic granularity D. Immunologic identification
81. To evaluate normal platelet numbers in an appropriate area of a blood smear, approximately how many platelets,
should be observed per oil immersion field?
A. 1 - 4 C. 8 - 20
B. 4 - 10 D. 20 - 50
82. For which of the following procedures would heparin be recommended anticoagulant?
A. Platelet count C. Smear-based red cell morphology
B. Coagulation tests D. Osmotic fragility
83. Falsely elevated hemoglobin values by the cyanmethemoglobin method may be caused by:
1. Lipemia 3. Extremely high WBC count
2. Hemoglobin C 4. Excessive anticoagulant
A. 1, 2 and 3 C. 2 and 4
B. 1 and 3 D. Only 4
84. Using the automated cell counter, which of the following laboratory results might be expected in the presence of
cold agglutinins?
1. Low RBC count 3. High MCV
2. Low MCH 4. Low MCHC
A. 1, 2 and 3 C. 2 and 4
B. 1 and 3 D. Only 4
85. Calculate the mean cell hemoglobin concentration (MCHC) using the following values:
Hgb: 15 g/dL (150 g/L) RBC: 4.50 x 106/µL (4.50 x 1012)
Hct 47 mL/dL (0.47)
A. 9.5% C. 10.4%
B. 31.9% D. 33.3%
86. A decreased osmotic fragility test would be associated which of the following conditions?
A. Sickle cell anemia C. Hereditary spherocytosis
B. HDN D. Acquired hemolytic anemia
87. Which of the following erythrocyte inclusions can be visualized with supravital stain but cannot be detected on a
Wright’s-stained blood smear?
A. Basophilic stippling C. Heinz bodies
B. Howell-Jolly bodies D. Siderotic granules
88. A Miller disk is an ocular device used to facilitate counting of:
A. Platelets C. Reticulocytes
B. Sickle cells D. Nucleated RBCs
89. SITUATION: RBC indices obtained on a patient are as follows: MCV 88 µm3 (fL); MCH 30 pg; MCHC 34%. The
RBCs on the peripheral smear would appear:
A. Microcytic, hypochromic C. Normocytic, normochromic
B. Microcytic, normochrmic D. Normocytic, hypochromic
90. Select the amino acid substitution that is responsible for sickle cell anemia.
A. Lysine is substituted for glutamic acid at the sixth position of the alpha chain
B. Valine is substituted for glutamic acid at the sixth position of the beta chain
C. Valine is substituted for glutamic acid at the sixth position of the alpha chain
D. Glutamine is substituted for glutamic acid at the sixth position of the beta chain
91. Which morphological classification is characteristic of megaloblastic anemia?
A. Normocytic, normochromic C. Macrocytic, hypochromic
B. Microcytic, normochromic D. Macrocytic, normochromic
92. Sodium fluoride maybe added to naphthyl ASD (NASDA) esterase reaction. The fluoride is added to inhibit a
positive reaction with:
13

A. Megakaryocytes C. Monocytes
B. Erythrocytes D. Granulocytes

93. What influence does the Philadelphia (Ph1) chromosome have on the prognosis of patients with chronic myelocytic
leukemia?
A. It is not predictive C. The prognosis is worse if Ph1 is present
1
B. The prognosis is better if Ph is present D. The disease usually transforms into AML when Ph 1 is
94. Laboratory findings in hereditary spherocytosis do NOT include:
A. Decreased osmotic fragility C. Reticulocytosis
B. Increased autohemolysis corrected by glucose D. Shortened erythrocyte survival
95. The laboratory findings on a patient are as follows:
MCV 55fL MCHC 25% MCH 17 pg
A stained blood film of this patient would most likely reveal a red cell picture that is:
A. Microcytic, hypochromic C. Normocytic, normochromic
B. Macrocytic, hypochromic D. Microcytic, normochromic
96. Which of the following laboratory findings is associated with Factor XIII deficiency?
A. Prolonged activated partial thromboplastin time C. Prolonged thrombin time
B. Clot solubility in a 5 molar urea solution D. Prolonged prothrombin time
97. Which test would be abnormal in patient with Stuart-Prower factor (factor X) deficiency?
A. PT only C. APTT only
B. PT and APTT D. Thrombin time
98. Aspirin prevents platelet aggregation by inhibiting the action of which enzyme?
A. Phospholipase C. Cyclooxygenase
B. Thromboxane A2 synthetase D. Prostacyclin synthetase
99. Which of the following is most likely to be abnormal in patients taking aspirin?
A. Platelet morphology C. Platelet count
B. Bleeding time D. Prothrombin time
100. Which results are associated with Hemophilia A?
A. Prolonged APTT, normal PT C. Prolonged PT, normal APTT
B. Prolonged PT and APTT D. Normal PT and APTT

END OF THE EXAM


14

PRE-BOARD EXAMINATION IN HEMATOLOGY (PART3)

1. The sequence of erythropoiesis from prenatal life to adulthood is:


a. Yolk sac, red bone marrow, liver and spleen c. Red bone marrow, yolk sac, liver and spleen
b. Yolk sac, liver and spleen, red bone marrow d. Liver and spleen, yolk sac, red bone marrow
2. The correct maturation order of erythrocyte morphologic stages is:
a. Prorubricyte, rubricyte, rubriblast, metarubricyte c. Rubriblast, prorubricyte, rubricyte, metarubricyte
b. Rubriblast, metarubricyte, rubricyte, prorubricyte d. Rubriblast, rubricyte, prorubricyte, metarubricyte
3. Compared to a rubricyte, a metarubricyte looks different because of its:
a. Dark blue cytoplasm b. Larger size c. Pyknotic nucleus d. Nucleoli
4. Primary granules in the neutrophil appear at which stage:
a. Myeloblast b. Promyelocyte c. Myelocyte d. Metamyelocyte
5. Specific (secondary) granules of the neutrophilic granulocyte:
a. appear first at the myelocyte stage c. are formed on the mitochondria
b. contain lysosomal enzymes d. are derived from azurophil (primary) granules
6. The earliest stage in the myeloid series at which it is possible to classify a given cell:
a. Myeloblast b. Promyelocyte c. Myelocyte d. Metamyelocyte
7. The last cell in the granulocytic series capable of mitosis is the:
a. Myeloblast b. Myelocyte c. Promyelocyte d. Metamyelocyte
8. In the neutrophil series of leukocyte development, the earliest stage to normally appear in the peripheral blood is the:
a. Myeloblast b. Promyelocyte c. Myelocyte d. Band
9. Which of the following is characteristic of cellular changes as megakaryoblasts mature into megakaryocytes within the
bone marrow?
a. progressive decrease in overall cell size c. nuclear division without cytoplasmic division
b. increasing basophilia of cytoplasm d. fusion of the nuclear lobes
10. The type of nuclear reproduction seen in megakaryocytes is:
a. Polyploid mitosis b. Endomitosis c. Meiosis d. Binary fission
11. These pairs of chains make up the majority of hemoglobin in normal adults:
a. α2 β2 b. α2 γ2 c. α2 δ2 d. ζ2 λ2
12. With respect to globin chain genes, which of the following statements is correct?
a. The genes for the alpha globin chains are located on chromosome 16 and the genes for the gamma, delta and
beta chains are located on chromosome 11
b. The genes for the gamma, delta and beta globin chains are located on chromosome 16 and the genes for the
alpha chains are located on chromosome 11
c. The genes for the alpha globin chains are located on chromosome 10 and the genes for the gamma, delta and
beta chains are located on chromosome 15
d. The genes for the alpha and delta globin chains are located on chromosome 11 and the genes for the gamma
and beta chains are located on chromosome 16
13. Most of the erythrocyte’s energy comes from the:
a. Embden-Meyerhof pathway c. Hexose monophosphate shunt
b. Rapoport-Leubering pathway d. Methemoglobin pathway
14. This metabolic pathway facilitates the oxygen release from hemoglobin to tissues:
a. Embden-Meyerhof pathway c. Hexose monophosphate shunt
b. Rapoport-Leubering pathway d. Methemoglobin pathway
15. A shift to the right in the ODC occurs when there is a(an):
a. Increase in O2 b. Increase in CO2 c. Increase in pH d. Decrease in CO2
16. The substance that is present in the urine in increased amounts if extravascular hemolysis is increased but there is
no intravascular hemolysis:
a. Methemoglobin b. Urobilinogen c. Hemoglobin d. Hemosiderin
17. Hemolytic anemia is not indicated by a(an):
a. Positive urine hemosiderin c. Positive fecal occult blood
b. Increased in plasm unconjuagted bilirubin d. Decreased serum haptoglobin
18. The following are compounds formed in the synthesis of heme:
1. coproporphyrinogen 3. uroporphyrinogen
2. porphobilinogen 4. protoporphyrinogen
Which of the following responses lists these compounds in the order in which they are formed?
15

a. 4,3,1,2 b. 2,3,1,4 c. 2,4,3,1 d. 2,1,3,4


19. Asynchronous development of hematopoietic cells within the bone marrow is the result of:
a. Inadequate levels of RNA c. Defective stem cells
b. Decreased erythropoietin d. Impaired DNA synthesis
20. Megaloblastic anemia is characterized by all of the following, EXCEPT:
a. Decreased WBC and retics c. Oval macrocytes
b. Hypersegmented neutrophils d. Increased platelets
21. Bite cells are associated with:
a. Pyruvate kinase deficiency b.PNH c. G6PD d. H. pyropoikiloctosis
22. What disorder is associated with erythrocytes that are thermally unstable and fragment when heated to 45 o to 46oC?
a. Hereditary spherocytosis c. PNH
b. Hereditary elliptocytosis d. Hereditary pyropoikilocytosis

23. Many microspherocytes and schistocytes and budding off of spherocytes can be seen on peripheral blood smears of
patients with:
a. hereditary spherocytosis c. acquired autoimmune hemolytic anemia
b. disseminated intravascular coagulation (DIC) d. extensive burns
24. Which of the following is most likely to be seen in lead poisoning?
a. iron overload in tissue b. codocytes c. basophilic stippling d. ringed sideroblasts
25. What cell shape is MOST commonly associated with an increased MCHC?
a. teardrop cells b. target cells c spherocytes d. sickle cells
26. Microangiopathic hemolytic anemia is characterized by:
a. Target cells and Cabot rings c. Pappenheimer bodies and basophilic stipplings
b. Toxic granules and Dohle bodies d. Schistocytes and nucleated RBCs
27. In myelofibrosis, the characteristic abnormal red cell morphology is:
a. Target cells b. Schistocytes c. Teardrop cells d. Ovalocytes
28. An enzyme deficiency associated with a moderate to severe hemolytic anemia after the patient is exposed to certain
drugs and that is characterized by red cell inclusions formed by denatured hemoglobin is:
a. lactate dehydrogenase deficiency c. pyruvate kinase deficiency
b. G-6-PD deficiency d. hexokinase deficiency
29. A 15-year-old girl is taking primaquine for a parasitic infection and notices her urine is a brownish color. A CBC
shows mild anemia. The laboratorian performing the reticulocyte count notices numerous irregular shaped granules
near the periphery of the RBC. These cellular inclusions are most likely:
a. Howell-Jolly bodies b. basophilic stippling c. Heinz bodies d. Pappenheimer bodies
30. What primary defect does the amino acid substitution in the -chain of Hb S have on the hemoglobin molecule?
a. Increases its oxygen affinity c. Decreases its solubility
b. Alters its stability d. Increases its solubility
31. Which of the following hemoglobin electrophoresis results is most typical of sickle cell trait?
a. 85% HbS and 15% HbA c. 45% HbS and 55% HbA
b. 85% HbF and 15% HbS d. 55% HbF and 45% HbS
32. What is the pathophysiology of β-thalassemia?
a. Decreased synthesis of α- chains c. Synthesis of structurally abnormal β-chains
b. Decreased synthesis of β-chains d. Abnormal heme synthesis
33. Iron deficiency anemia may be distinguished from anemia of chronic infection by:
a. Serum iron level b. Red cell morphology c. Red cell indices d TIBC
34. Which antibiotic is most often implicated in the development of aplastic anemia?
a. Sulfonamides b. Chloramphenicol c. Teracycline d. Penicillin
35. Which of the following is a pure red cell aplasia?
a. Bernard-Soulier syndrome c. Diamond-Blackfan anemia
b. DiGuglielmo’s disease d. Fanconi’s anemia
36. Spherocytes may be seen in the peripheral blood of patients with:
a. Autoimmune hemolytic anemia c. Iron deficiency anemia
b. Vitamin B12 deficiency d. Pyruvate kinase deficiency
16

37. In a jaundiced 16-year-old, laboratory studies indicate that the MCHC is increased, the Coombs' test is negative, and
osmotic fragility of the red cells is increased. Erythrocytes on a peripheral smear are most likely to be:
a. Acanthocytes b. Spherocytes c. Teardrop cells d. Target cells
38. Which of these hemoglobin derivatives cannot be reduced back to normal hemoglobin?
a. Methemoglobin b. Oxyhemoglbin c. Sulfhemoglobin d. Carboxyhemoglobin
39. Which of the following is associated with Chediak-Higashi syndrome?
a. membrane defect of lysosomes c. two-lobed neutrophils
b. Dohle bodies and giant platelets d. mucopolysaccharidosis
40. Which of the following is associated with Alder-Reilly inclusions?
a. membrane defect of lysosomes c. two-lobed neutrophils
b. Dohle bodies and giant platelets d. mucopolysaccharidosis
41. A differential was performed on an asymptomatic patient. The differential included 60% neutrophils: 55 of which had
2 lobes and 5 had 3 lobes. There were no other abnormalities. This is consistent with which of the following
anomalies?
a. Pelger-Huet b. May-Hegglin c. Alder-Reilly d. Chediak-Higashi
42. Of the following, the disease most closely associated with pale blue inclusions in granulocytes and giant platelets is:
a. Gaucher’s disease b. May-Hegglin anomaly c. Alder-Reilly anomaly d. Pelger-Huet anomaly
43. Of the following, the disease most closely associated with glucocerebrosidase deficiency is:
a. Gaucher’s disease b. Pelger-Huet anomaly c. Chediak-Higashi syndrome d. May-Hegglin anomaly
44. Bone marrow slide shows foam cells ranging from 20 to 100 m in size with vacuolated cytoplasm containing
sphingomyelin and is faintly PAS positive. This cell type is most characteristic of:
a. Gaucher’s disease b. Di Guglielmo disease c. myeloma with Russell bodies d. Niemann-Pick disease
45. In the French-American-Bristish (FAB) classification, myelomonocytic leukemia would be:
a. M1 and M2 b. M3 c. M4 d. M5
46. DIC is most often associated with which of the following FAB designation of acute leukemia?
a. M1 b. M3 c. M4 d. M5
47. Auer rods are most likely present in which of the following?
a. chronic granulocytic leukemia c. erythroleukemia
b. myelofibrosis with myeloid metaplasia d. acute granulocytic leukemia
48. Which of the following stains is helpful in the diagnosis of suspected erythroleukemia?
a. peroxidase b. periodic acid-Schiff c. nonspecific esterase d. acid phosphatase
49. The FAB classification of ALL seen most commonly in children is:
a.L1 b. L2 c. L3 d. Burkitt’s type
50. A 50-year-old woman who has been receiving busulfan for 3 years for chronic myelogenous leukemia becomes
anemic. Laboratory tests reveal:
Thrombocytopenia
Many peroxidase-negative blast cells in the peripheral blood
Bone marrow hypercellular in blast transformation
Markedly increased bone marrow TdT
Which of the following complications is this patient most likely to have?
a. acute lymphocytic leukemia c. acute myelomonocytic leukemia
b. acute myelocytic leukemia d. busulfan toxicity
51. A peripheral smear shows 75% blasts. These stain positive for SBB and peroxidase. Given these values, which of the
following disorders is most likely?
a. AML b. CML c. Undiff.leukemia d. ALL
52. A peripheral blood smear demonstrating an absolute increase in small “hypermature” lymphocytes and “smudge” cells
would be suggestive of:
a. Chronic lymphocytic leukemia c. Leukomoid reaction
b. Acute lymphocytic leukemia d. Infectious mononucleosis
53. Chromosome analysis revealed the presence of the Philadelphia chromosome. Based on this information, what
myeloproliferative disorder is present?
a. CML b. PV c. ET d. MMM
17

54. The absence of the Philadelphia chromosome in granulocytic leukemia suggests:


a. Rapid progression of the disease c. Excellent response to therapy
b. Polyclonal origin to the disease d. Conversion to another myeloproloferative disorder
55. Which of the following tests can be useful in differentiating leukemoid reactions from chronic granulocytic leukemias?
a. peroxidase stain b. surface membrane markers c. Sudan black B stain d. LAP
56. The following results were obtained on a leukocyte alkaline phosphatase stain:
Score 4+ 3+ 2+ 1+ 0
No. of cells counted 40 30 20 5 5
These reactions are most consistent with:
a. leukemoid reaction c. chronic granulocytic leukemia
b. nephrotic syndrome d. progressive muscular dystrophy
57. The esterase cytochemical stains are useful to differentiate:
a. Granulocytic from monocytic leukemias
b. Lymphocytic leukemias from myelocytic leukemias
c. Monocytic leukemias from megakaryocytic leukemias
d. Lymphocytic leukemias from monocytic leukemias
58. The FAB classification of a leukemia with large blasts that are myeloperoxidase and specific esterase negative but
have strong Positivity for nonspecific esterase inhibited by sodium fluoride is:
a. M1 b. M4 c. M5 d. M7
59. Which of the following cells are most likely identified in lesions of mycosis fungoides?
a. T lymphocytes b. B lymphocytes c. monocytes d. mast cells
60. The atypical lymphocyte seen in the peripheral smear of patients with infectious mononucleosis is reacting to which of
the following?
a. T lymphocytes b. B lymphocytes c. monocytes d. mast cells
61. Which of the following cells are the atypical lymphocytes seen on the peripheral blood smear of patients with
infectious mononucleosis?
a. T lymphocytes b. B lymphocytes c. monocytes d. mast cells
62. The results of a bone marrow differential are: 40% neutrophils and precursors; 20% lymphocytes, 20% monocytes,
and 20% red cell precursors. The M;E ratio for this patient is:
a. 1:2 b. 2:1 c. 3:1 d. 4:1
63. When Reed-Sternberg cells are found in a lymph node biopsy, they are indicative of:
a. Hodgkin disease c. Sezary syndrome
b. Intermediate-grade non-Hodgkin lymphoma d. High-grade non-Hodgkin lymphoma
64. The antigen referred to as CALLA, the common ALL antigen is:
a. CD 10 b. CD 16 c. CD 56 d. CD 73
65. Normal platelets have a circulating life span of approximately:
a. 5 days b. 10 days c. 20 days d. 30 days
66. Platelet dense granules contain:
1. Calcium 2. Storage ADP 3. Serotonin 4. Plt factor 4
a. 1 and 3 b. 2 and 4 c. 1, 2 and 3 d. 1, 2, 3 and 4
67. The type of microscopy recommended for a manual platelet count is:
a. Fluorescence b. Phase c. Brightfiled d. Oil immersion
68. Which of the following is characteristic of Bernard-Soulier syndrome?
a. giant platelets c. abnormal aggregation with ADP
b. normal bleeding time d. increased platelet count

69. The enzyme inhibited by aspirin is:


a. Thromboxane synthetase c. Lactate dehydrogenase
b. Cyclooxygenase d. Phospholipase
70. A patient has been taking aspirin regularly for arthritic pain. Which one of the following tests is most likely to be
abnormal in this patient?
18

a. platelet count c. prothrombin time


b. template bleeding time d. activated partial thromboplastin time
71. Platelet glycoprotein IIb/IIIa complex is:
a. Membrane receptor for fibrinogen c. secreted by endothelial cells
b. Secreted from the dense bodies d. Also called actin
72. Which set of platelet responses would be most likely be associated with Glanzmann’s thrombasthenia?
a. Normal platelet aggregation response to ADP and ristocetin; decreased response to collagen
b. Normal platelet aggregation response to collagen; decreased response to ADP and collagen
c. Normal platelet aggregation response to ristocetin; decreased response to collagen, ADP and epinephrine
d. Normal platelet aggregation response to ADP; decreased response to collagen and ristocetin
73. Platelet aggregation studies revealed normal aggregation curves with collagen, epinephrine, and ADP, but an
abnormal aggregation curve with ristocetin. Based on these findings, what is the differential diagnosis?
a. Von Willebrand disease and Bernard-Soulier syndrome
b. Glanzmann’s thrombasthenia and von Willebrand disease
c. Storage pool disease and Glanzmann’s thrombasthenia
d. Bernard-Soulier syndrome and storage pool disease
74. The preferred blood product for a bleeding patient with von Willebrand’s disease is transfusion with:
a. Factor II, VII, IX, X concentrates c. Fresh Frozen Plasma and Platelets
b. Platelet Concentrates d. Cryoprecipitated AHF
75. Which of the following is vitamin K dependent?
a. Factor XII b. fibrinogen c. antithrombin III d. Factor VII
76. Last factor to be depressed n vitamin K deficiency:
a. Factor II b. Factor VII c. Factor X d. Factor IX
77. Coagulation factors affected by coumarin drugs are:
a. VIII, IX , and X b. I, II, V, and VII c. II, VII, IX, and X d. II, V, and VII
78. Which of the following factors is not present in BaSO4 adsorbed plasma?
a. VIII b. II c. XII d. V
79. Which of the following is NOT a contact factor?
a. Prekallikrein b. Stable factor c. HMWK d. Hageman factor
80. The most concentrated coagulation factor in the blood is:
a. XII b. IX c. X d. Fibrinogen
81. Which of the following factor deficiencies is associated with either no bleeding or only a minor bleeding tendency,
even after trauma or surgery?
a. Factor X b. Factor XII c. Factor XIII d. Factor V
82. Classic hemophilia is condition in which there may be a:
a. Prolonged BT b. Decreased platelets c. Prolonged PT d. Prolonged APTT
83. Increased APTT with a normal PT would indicate a deficiency of:
a. Factor II b. Factor VII c. Factor IX d. Factor X
84. PTT measures all factors except for:
a. I and V b. VIII and IX c. V and VIII d. VII and XIII
85. The international normalized ratio (INR) is useful for:
a. Determining coagulation reference ranges c. Monitoring thrombolytic therapy
b. Monitoring heparin therapy d. Monitoring warfarin therapy
86. The following results are obtained:
PT Normal
APTT Prolonged
Absorbed plasma Corrects APTT
The factor deficiency is:
a. VIII b. IX c. X d. V
87. Which of the following laboratory findings is associated with Factor XIII deficiency?
a. prolonged activated partial thromboplastin time c. prolonged thrombin time
b. clot solubility in a 5 molar urea solution d. prolonged prothrombin time
88. In factor X deficiency, a prolonged PT or APTT is corrected by the addition of:
19

1. Aged plasma 2. Aged serum 3. Fresh serum 4. Adsorbed plasma


a. 1, 2 and 3 b. 1 and 3 c. 2 and 4 d. 1, 2, 3 and 4
89. A 56-year-old woman was admitted to the hospital with a history of a moderate to severe bleeding tendency of a
several years’ duration. Epistaxis and menorrhagia were reported. Prolonged APTT was corrected with fresh normal
plasma, adsorbed plasma, and aged serum. Deficiency of which of the following is most likely?
a. Factor XII b. Factor VIII c. Factor XI d. Factor IX
90. A patient has a history of mild hemorrhagic episodes. Laboratory resutls include a prolonged prothrombin time and
activated partial thromboplastin time. The abnormal prothrombin time was corrected by normal and adsorbed plasma,
but not aged serum. Which of the following coagulation factors is deficient?
a. prothrombin b. Factor V c. Factor X d. Factor VII
91. A prolonged Stypven (Russell viper venom) time is associated with deficiency of the following factors EXCEPT:
a. Factor I b. Factor II c. Factor X d. Factor VII

92. The observation of a normal reptilase time and a prolonged thrombin time is indicative of:
a. Presence of fibrin degradation products c. Dysfibrinogenemia
b. Hypoplasminogenemia d. Presence of heparin
93. When making a blood film using the spreader slide technique, a thinner film can be obtained by:
a. Increasing the angle of the spreader slide c. Spreading the blood at lower speed
b. Using a larger drop of blood d. All of the above
94. A false increase in ESR value can be caused by:
a. Cold room temperatures c. Reading ESR after exactly 30 minutes
b. Slight tilting of the ESR tube rack d. Overanticoagulation of the blood specimen
95. Which of the following statements concerning reticulocyte count is/are true?
1. The adult reference range is approximately 0.5 to 5.0%
2. Newborns have a higher reference range than adults
3. The material that stains in reticulocytes is DNA
4. The Miller disk is a device used to aid in reticulocyte counting
a. 1 and 3 b. 2 and 4 c. 1, 2 and 3 d. 4 only
96. In an automated instrument, which parameter is calculated rather than directly measured:
a. Hematocrit b. RBC count c. WBC count d. Hemoglobin
97. The cyanmethemoglobin method measures:
a. All hemoglobin pigments c. All hemoglobin pigments except sulfhemoglobin
b. Only oxyhemoglobin d. Only methemoglobin
98. Two hemoglobins that migrate together on cellulose acetate electrophoresis at an alkaline pH are:
a. A1 and A2 b. A1 and E c. S and C d. S and D
99. A decreased result in the osmotic fragility test would be associated with which of the following conditions?
a. Sickle cell anemia b. H. spherocytosis c. HDN d. AIHA
100. A patient has the following results: erythrocyte count, 2.5 x 10 12/L, hemoglobin 5.3 g/dL; hematocrit 0.17 L/L;
reticulocyte count, 1%. What are the erythrocyte indices in the above patient?
a. MCV 47 MCH 28 MCHC 31 c. MCV 80 MCH 21 MCHC 41
b. MCV 68 MCH 21 MCHC 31 d. MCV 80 MCH 28 MCHC 41

END OF THE EXAM


PRE-BOARD EXAMINATION IN HISTOPATHOLOGY AND MEDICAL TECHNOLOGY LAWS (PART 1)

1. Ideal amount of the fixative:


A. 1 – 2X the volume of the specimen C. 10 – 15X the volume of the specimen
B. 5 – 10X the volume of the specimen D. 10 – 20X the volume of the specimen
2. All of the following are microanatomical fixatives, EXCEPT:
A. 10% Formalin C. Bouin’s solution
B. Zenker’s solution D. Flemming’s fluid
3. Best fixative for the nervous system:
A. Formalin fixative C. Lead fixative
B. Permanganate fixative D. Chromate fixative
4. Fixative of choice for the preservation of fats:
A. Helly’s fluid C. Formalin
B. Newcomer’s fluid D. Newcomer’s fluid
5. Helly’s fluid is exactly the same as in Zenker’s fixative, but instead of GLACIAL ACETIC ACID, what component is
added:
A. Mercuric chloride C. Sodium sulfate
B. Potassium dichromate D. Formalin
6. Fixative recommended for fixing small pieces of liver, spleen, connective tissue fibers and nuclei:
A. Zenker’s fluid C. Heidenhain’s Susa solution
B. Orth’s fluid D. Regaud’s fluid
7. Excellent microanatomic fixative for pituitary gland, bone marrow and blood containing organs such as spleen and
liver:
A. Bouin’s solution C. Carnoy’s fluid
B. Helly’s fluid D. Flemming’s solution
8. All mercurial fixing solutions lead to the formation in tissues of diffuse BLACK granules and these mercury deposits
must be removed BEFORE STAINING. Removal of mercuric chloride deposit is accomplished by:
A. Saturated solution of iodine C. Distilled water
B. Sodium thiosulfate D. Saturated solution of picric acid
9. Considered to be the MOST RAPID fixative/ recommended for fixing chromosomes, lymph glands and urgent
biopsies:
A. Gendre’s fixative C. Newcomer’s fluid
B. Carnoy’s fluid D. Flemming’s solution
10. Fixatives used mainly for acid mucopolysaccharides:
A. Lead fixatives C. Chromate fixatives
B. Mercurial fixatives D. Picric acid fixatives
11. Most widely used fixative for electron microscopy:
A. Acetone C. Osmium tetroxide
B. Zenker’s fluid D. Trichloroacetic acid
12. The process of decalcification is best performed:
A. Before fixation C. After impregnation
B. After fixation D. None of these
13. Most ideal and most reliable method of determining extent of decalcification:
A. Physical test C. X-ray or radiological test
B. Chemical test D. Adsorption test
14. The fastest chemical solution in decalcifying tissues is:
A. Trichloroacetic acid C. Formic acid
B. Nitric acid D. Versene
15. All of the following remove intracellular and extracellular water from the tissue following fixation, EXCEPT:
A. Alcohol C. Tetrahydrofuran
B. Chloroform D. Dioxane
16. A TOXIC dehydrating agent, primarily employed for blood and tissue films and for smear preparation:
A. Ethyl alcohol C. Butyl alcohol
B. Methyl alcohol D. Isopropyl alcohol
17. Function/s of tetrahydrofuran:
A. Dehyrating agent C. Both of these
B. Clearing agent D. None of these
18. Excessive exposure to this clearing agent may be extremely toxic to man and may become carcinogenic or it may
damage the bone marrow resulting to APLASTIC ANEMIA:
A. Xylene C. Tetrahydrofuran
B. Benzene D. Toluene
19. Process whereby the clearing agent is completely removed from the tissue and replaced by a medium that will
completely fill all the tissue cavities:
A. Embedding C. Blocking
B. Infiltration D. Casting
20. Simplest, most common and best embedding medium for routine tissue processing:
A. Paraffin wax C. Celloidin
B. Ester wax D. Carbowax
21. A semi-synthetic wax used for embedding the eyes:
A. Paraplast C. Ester wax
B. Bioloid D. Carbowax
22. The DRY celloidin embedding method is employed chiefly for the:
A. Bones and teeth C. Whole organs
B. Large brain blocks D. Eyes
2

23. All of the following are substitutes for paraffin wax, EXCEPT:
A. Paraplast C. Malinol
B. Embeddol D. Tissue Mat
24. Melting point of ester wax:
A. 56-57oC B. 46-48oC C. 54-58oC D. 50-54oC
25. The last container through which tissue pass through in an automatic tissue processor contains:
A. Paraffin C. Formalin
B. Xylol D. Alcohol
26. Microtome knife recommended for frozen sections or for cutting extremely hard and tough specimens embedded in
paraffin blocks, using a base-sledge type or sliding microtome:
A. Plane-concave knife C. Biconcave knife
B. Plane-wedge knife D. None of these
27. Removal of gross nicks on the knife edge:
A. Honing B. Stropping C. Both of these D. None of these
28. Removal of “burr” or irregularities on the knife edge:
A. Honing B. Stropping C. Both of these D. None of these
29. Angle formed between the cutting edge of the microtome knife (27o-32o):
A. Bevel angle C. Both of these
B. Clearance angle D. Neither of these
30. Angle formed between the surface of the block and the cutting edge of the knife (0o-15o):
A. Bevel angle C. Both of these
B. Clearance angle D. Neither of these
31. This type of microtome easily cuts large blocks and serial sections can be obtained with ease because larger knives
can be used:
A. Sliding C. Rocking
B. Rotary D. Freezing
32. The cryostat is an apparatus used in fresh tissue microtomy. It consists of a microtome, kept inside a cold chamber
which has been maintained at a temperature of:
A. 20oC B. -20oC C. 4oC D. -4oC
33. A tissue exposed to short burst of CARBON DIOXIDE for a few minutes will:
A. Freeze C. Dehydrate
B. Harden D. Fix
34. Fat cells and enzymes are best demonstrated in:
A. Paraffin section C. Celloidin section
B. Plastic embedded section D. Frozen section
35. When trimming tissue block, they must be surrounded by at least __ of wax.
A. 1 mm B. 2 mm C. 3 mm D. 4 mm
36. Thickness of paraffin sections for routine histologic procedures:
A. 10-15  B. 4-6  C. 0.5  D. 5-10 
37. The following methods are done for drying sections on slide, EXCEPT:
A. On a hot plate at 45-55oC for 30-45 minutes
B. On a Bunsen flame
C. In an incubator at 37oC for 3 hours
D. In a wax oven at 56-60oC for 2 hours
38. Most probable cause when clearing agent turns milky as soon as the tissue is placed in it:
A. Incomplete fixation C. Incomplete dehydration
B. Prolonged fixation D. Prolonged dehydration
39. It is added to Mayer’s egg albumin to prevent the growth of molds:
A. Sodium chloride C. Thymol crystals
B. Glycerol D. Powdered starch
40. Adhesive added to the water in the floating-out bath – most convenient alternative to direct coating of slides:
A. Plasma B. Gelatin C. Starch paste D. Dried albumin
41. To avoid distortion of the image, the refractive index of the mountant should be near as possible to that of the glass
which is:
A. 1. 581 B. 1.185 C. 1.518 D. 1.155
42. Deparaffinization of tissue sections is accomplished by passing through:
A. Ammonia water C. Alcohol
B. Acetone D. Xylol
43. Coverslips from slides may be removed by immersion in:
A. Ammonia water C. Alcohol
B. Acetone D. Xylol
44. Process by which sections are stained with simple aqueous or alcoholic solutions of the dye:
A. Progressive staining C. Direct staining
B. Regressive staining D. Indirect staining
45. A tissue-mordant-dye complex is needed in:
A. Progressive staining C. Direct staining
B. Regressive staining D. Indirect staining
46. With this staining technique, the tissue is first overstained, and the excess stain is removed or decolorized from
unwanted part of the tissue:
A. Progressive staining C. Direct staining
3

B. Regressive staining D. Indirect staining

47. The regressive staining method employs this procedure:


A. Deparaffinization C. Differentiation
B. Clearing D. Dehydration
48. Accelerate or hasten the speed of the staining power and selectivity of the dye:
A. Oxidizing agents C. Accentuators
B. Acid differentiators D. Mordants
49. Substances which aid in attaching a stain or dye to the tissue:
A. Oxidizing agents C. Accentuators
B. Acid differentiators D. Mordants
50. This technique entails the use of specific dyes, which differentiate particular substances by staining them with a color
that is different from that of the stain itself:
A. Orthochromatic staining C. Counterstaining
B. Metachromatic staining D. Vital staining
51. All of the following are metachromatic stains, EXCEPT:
A. Thionine C. Safranin
B. Toluidine blue D. Eosin
52. Stain used for demonstrating mitochondria during intravital staining:
A. Victoria blue C. Benzidine
B. Acridine orange D. Janus Green B
53. Application of a different color or stain to provide contrast and background to the staining of the structural component
to be demonstrated:
A. Orthochromatic staining C. Counterstaining
B. Metachromatic staining D. Vital staining
54. The routine stain for surgical tissue section is:
A. Gram’s stain C. Pap’s stain
B. Wright’s stain D. H&E stain
55. Routinely used in histopathology as a counterstain/background fro contrasting stain because it gives a pleasing and
colorful contrasts to nuclear stains:
A. Eosin C. Methylene blue
B. Toluidine blue D. Crystal violet
56. In the hematoxylin-eosin stain (H&E), this stain acts as the acid dye:
A. Hematoxylin C. Either of these
B. Eosin D. Neither of these
57. In routine H&E, most fixatives can be used, EXCEPT:
A. Zenker’s formol C. Osmic acid
B. Formaldehyde D. Picric acid
58. Only substance in histopathology that can fix, differentiate or stain tissues all by itself:
A. Osmic acid C. Trichloroacetic acid
B. Picric acid D. Acetic acid
59. The active dye (coloring agent) in hematoxylin solution is:
A. Hematin B. Hematein C. Hematoxylin D. Hematoxylon
60. All of the following are chemical oxidizing agent/ripening agent for hematoxylin, EXCEPT:
A. Sodium iodate C. Hydrogen peroxide
B. Mercuric oxide D. Ammonium alum
61. Ripening agent for Harris’ hematoxylin:
A. Sodium iodate C. Potassium permanganate
B. Mercuric oxide D. Hydrogen peroxide
62. Function of ammonium alum in the Harris hematoxylin formula:
A. Oxidizing agent C. Dye solvent
B. Mordant D. Dye
63. Staining of the nuclei by alum hematoxylin is enhanced by the addition of:
A. Alum C. Glacial acetic acid
B. Ethanol D. Mercuric oxide
64. Glacial acetic acid added to hematoxylin will:
A. Inactivate stain C. Enhance cytoplasmic staining
B. Decrease nuclear staining D. Enhance nuclear staining
65. A sheen of oxidized dye on the surface of hematoxylin solution indicates:
A. Stain is concentrated C. Stain has to be filtered
B. Stain is contaminated by bacteria D. Stain has to be discarded
66. Acid alcohol used in routine H&E acts as:
A. Mordant C. Bluing agent
B. Stain D. Differentiator
67. The acid used in combination with alcohol in an acid alcohol solution:
A. Acetic acid C. Nitric acid
B. Oxalic acid D. Hydrochloric acid
68. Staining result in the routine hematoxylin and eosin method:
A. Nuclei pink, cytoplasm blue to blue-black
B. Nuclei blue to blue-black, cytoplasm pink
4

C. Nuclei and cytoplasm pink


D. Nuclei and cytoplasm blue-black

69. A common BASIC NUCLEAR STAIN for plasma cells and may also be employed in cytological examination of fresh
sputum for malignant cells:
A. Eosin C. Crystal violet
B. Methylene blue D. Aniline blue
70. PAS positive substances are stained:
A. Blue C. Green
B. Brown-black D. Magenta red
71. Staining method for glycogen:
A. PAS technique C. Best Carmine method
B. Langhan’s iodine stain D. All of these
72. Tubercle bacilli in Ziehl-Neelsen stain:
A. Bright red C. Green
B. Grayish blue D. Blue
73. Feulgen’s reaction is the most reliable and most specific histochemical staining technique for:
A. RNA B. DNA C. Both D. Neither
74. In the Masson-Fontana ammoniacal silver reaction, melanin and argentaffin cell granules are stained:
A. Brown B. Red C. Black D. Blue
75. Lipids in Sudan IV are stained:
A. Black B. Blue black C. Red D. Deep blue to violet
76. Positive reaction for cholesterol in Schultz Method:
A. Blue-black C. Pale pink
B. Grayish-blue D. Blue-green
77. The method of choice for staining in exfoliative cytology:
A. Pap’s stain C. H&E stain
B. Wright’s stain D. Giemsa stain
78. Components of the EA-50 stain, EXCEPT:
A. Eosin Y C. Light green
B. Orange green D. Bismarck brown
79. All of the following are fixative used for cytologic smears, EXCEPT:
A. Equal parts of 95% ETOH and ether C. 10% Formalin
B. 95% Ethyl alcohol D. Carnoy’s fluid
80. BEST fixative for virtually all diagnostic cytology studies but flammable and volatile:
A. Equal parts of 95% ETOH and ether C. 10% Formalin
B. 95% Ethyl alcohol D. Carnoy’s fluid
81. All of the following are examples of good adhesive agents for cytologic method, EXCEPT:
A. Egg albumin C. Pooled human serum or plasma
B. Celloidin ether alcohol D. Leuconostoc culture
82. Smears prepared for cytologic examination from cut surfaces of lymph nodes can be obtained by:
A. Spreading C. Touch preparation
B. Pull-Apart D. Streaking
83. Organisms found in the normal vaginal flora that stain BLUE to lavander with Pap’s method:
A. Trichomonas vaginalis C. Doderlein bacilli
B. Streptococci D. Acid fast bacilli
84. Clue cells are diagnostic of what infection:
A. Neisseria gonorrhoeae C. HSV-II
B. Gardnerella vaginalis D. Candida albicans
85. Cells found in a vaginal smear which are thick and round to oval in shape (similar to fried fresh eggs with sunny-side
up) with strongly basophilic cytoplasm and vesicular nucleus:
A. Superficial cells C. Parabasal cells
B. Intermediate cells D. Basal cells
86. Increase in size of an organ or tissue due to an increase in number of cells:
A. Hypertrophy B. Necrosis C. Hyperplasia D. Atrophy
87. Continuous abnormal proliferation of cells without control causes an overgrowth of tissue or tumor cells:
A. Anaplasia B. Hyperplasia C. Neoplasia D. Dysplasia
88. The dissolving of cells by enzymatic action is:
A. Rotting B. Decay C. Autolysis D. Putrefaction
89. A malignant tumor is least characterized by:
A. Metastases C. Encapsulation
B. Mitotic figures D. Invasion of adjacent tissues
90. A medical laboratory technician to be registered must have obtained a grade within this range (Medical
Technology Board Exam):
A. 65-74.5% C. 60-74.9%
B. 70-74.9% D. 73-73%
91. RA 5527 was approved on:
A. June 21, 1969 C. June 21, 1968
B. July 21, 1969 D. June 11, 1978
92. The Blood Banking Law:
5

A. RA 4224 C. RA 1517
B. RA 4688 D. RA 5527
93. National Blood Services Act of 1994:
A. RA 1517 C. RA 7719
B. RA 8504 D. RA 8981

94. Philippine AIDS Prevention and Control Act of 1998:


A. RA 1517 C. RA 7719
B. RA 8504 D. RA 8981
95. Newborn Screening Act of 2004
A. RA 9288 C. RA 9165
B. RA 8504 D. RA 7719
96. All services are done on a primary category laboratory except: (A.O. 2007-0027)
A. Routine urinalysis C. Routine chemistry
B. Routine hematology D. Blood typing
97. Who are the members of the Medical technology Board?
A. Pathologist, medical technologist and medical technician
B. Secretary of health, dean and president of PAMET
C. One internist, one pathologist and one medical technologist
D. A registered pathologist and two registered medical technologist
98. Sections of RA 5527 amended by PD 1534:
A. 3, 8, 13 C. 11, 16, 29
B. 16, 21, 22 D. 11, 21, 29
99. In order to pass examination (MT), a candidate must obtain a general average of:
A. 50% B. 60% C. 70% D. 75%
100. The penalty of revocation of certificate of registration may be imposed by the board if there is:
A. Majority vote B. Unanimous vote C. Either of these D. Neither of these

END OF THE EXAM


PRE-BOARD EXAMINATION IN HISTOPATHOLOGY AND MEDICAL TECHNOLOGY LAWS (PART 2)

1. Nonkeratinizing stratified squamous epithelium would be found lining the:


A. Skin C. Esophagus
B. Trachea D. Urinary bladder
2. The connective tissue cells actively involved in wound healing are:
A. Plasma cells C. Macrophages
B. Mast cells D. Fibroblasts
3. Tissue composed of a network of bony trabeculae separated by interconnecting bone marrow spaces:
A. Cortical bone C. Cancellous bone
B. Woven bone D. Compact bone
4. Muscle that histologically contains cytoplasmic cross-striations and has multiple nuclei located at the edge of the
fibers is classified as:
A. Smooth C. Skeletal
B. Visceral D. Cardiac
5. Malignant tumors of connective tissue are known as:
A. Carcinomas C. Sarcomas
B. Lipomas D. Fibromas
6. The antibody class most frequently used in immunofluorescent and immunoenzyme staining is:
A. IgM C. IgG
B. IgE D. IgA
7. The most specific stain available for the identification of melanomas in paraffin section is the:
A. DOPA reaction C. Warthin-Starry
B. Gridley method D. Immunostain for HMB-45
8. Frozen sections are commonly used for:
1. Rapid pathologic diagnosis during surgery 3. Immunohistochemical staining
2. Diagnostic and enzyme histochemistry 4. Demonstration of lipids and carbohydrates
A. 1 and 2 are correct C. 1, 2 and 3 are correct
B. 2 and 4 are correct D. All are correct
9. Characteristics of a good fixative:
1. Cheap, stable and safe to handle 3. Inhibit bacterial decomposition and autolysis
2. Produce minimum shrinkage of tissue 4. Harden tissue thereby making cutting of sections easier
A. 1 and 2 are correct C. 1, 2 and 3 are correct
B. 2 and 4 are correct D. All are correct
10. Microanatomical fixatives:
1. 10% formol saline 3. Zenker’s solution
2. Heidenhain’s susa 4. Flemming’s fluid
A. 1 and 2 are correct C. 1, 2 and 3 are correct
B. 2 and 4 are correct D. All are correct
11. Mercuric chloride fixatives:
1. Zenker’s fluid 3. B-5 fixative
2. Heidenhain’s Susa solution 4. Orth’s fluid
A. 1 and 2 are correct C. 1, 2 and 3 are correct
B. 2 and 4 are correct D. All are correct
12. All mercurial fixing solutions lead to the formation in tissues of diffuse BLACK granules and these mercury deposits
must be removed BEFORE STAINING. Removal of mercuric chloride deposit is accomplished by:
A. Saturated solution of iodine C. Distilled water
B. Sodium thiosulfate D. Saturated solution of picric acid
13. Most widely used fixative for electron microscopy:
A. Acetone C. Osmium tetroxide
B. Zenker’s fluid D. Trichloroacetic acid
14. When fat is to be preserved, the fixative of choice is:
A. Formalin C. Carnoy solution
B. Zenker solution D. Bouin solution
15. Zenker fluid contains all of the following, except:
A. Mercuric chloride C. Formaldehyde
B. Potassium dichromate D. Glacial acetic acid
16. Orth solution contains all of the following, except:
A. Potassium dichromate C. Sodium sulfate
B. Mercuric chloride D. Formaldehyde
17. Fixative for blood smears:
A. Bouin solution C. B-5
B. Carnoy solution D. Methanol
18. An unknown pigment in tissue section that can be bleached with a saturated alcoholic solution of picric acid is most
likely:
A. Melanin pigment C. Hemosiderin
B. Formalin pigment D. Mercury pigment
19. A common reason for adding acetic acid to fixatives is to:
A. Coagulate proteins C. Preserve carbohydrates
B. Reduce shrinkage D. Change the pH
20. The process of decalcification is best performed:
A. Before fixation C. After impregnation
B. After fixation D. None of these

21. Most ideal and most reliable method of determining extent of decalcification:
A. Physical test C. X-ray or radiological test
B. Chemical test D. Adsorption test
2

22. The fastest chemical solution in decalcifying tissues is:


A. Trichloroacetic acid C. Formic acid
B. Nitric acid D. Versene
23. All of the following remove intracellular and extracellular water from the tissue following fixation, EXCEPT:
A. Alcohol C. Tetrahydrofuran
B. Chloroform D. Dioxane
24. Dioxane is a reagent that can be used:
A. For both fixing and dehydrating tissues C. In very small volume rations
B. For both dehydrating and clearing tissues D. For long periods without changing
25. Function/s of tetrahydrofuran:
A. Dehyrating agent C. Both of these
B. Clearing agent D. None of these
26. Excessive exposure to this clearing agent may be extremely toxic to man and may become carcinogenic or it may
damage the bone marrow resulting to APLASTIC ANEMIA:
A. Xylene C. Tetrahydrofuran
B. Benzene D. Toluene
27. Process whereby the clearing agent is completely removed from the tissue and replaced by a medium that will
completely fill all the tissue cavities:
A. Embedding C. Blocking
B. Infiltration D. Casting
28. The last container through which tissue pass through in an automatic tissue processor contains:
A. Paraffin C. Formalin
B. Xylol D. Alcohol
29. Simplest, most common and best embedding medium for routine tissue processing:
A. Paraffin wax C. Celloidin
B. Ester wax D. Carbowax
30. A semi-synthetic wax used for embedding the eyes:
A. Paraplast C. Ester wax
B. Bioloid D. Carbowax
31. The DRY celloidin embedding method is employed chiefly for the:
A. Bones and teeth C. Whole organs
B. Large brain blocks D. Eyes
32. All of the following are substitutes for paraffin wax, EXCEPT:
A. Paraplast C. Malinol
B. Embeddol D. Tissue Mat
33. Melting point of ester wax:
A. 56-57oC C. 46-48oC
B. 54-58 C o D. 50-54oC
34. Carbowax has a major disadvantage of:
A. Dissolving during flotation C. Making tissues brittle fro sectioning
B. Being a lengthy processing method D. Causing cell shrinkage
35. Microtome knife recommended for frozen sections or for cutting extremely hard and tough specimens embedded in
paraffin blocks, using a base-sledge type or sliding microtome:
A. Plane-concave knife C. Biconcave knife
B. Plane-wedge knife D. None of these
36. Removal of gross nicks on the knife edge:
A. Honing C. Both of these
B. Stropping D. None of these
37. Removal of “burr” or irregularities on the knife edge:
A. Honing C. Both of these
B. Stropping D. None of these
38. Angle formed between the cutting edge of the microtome knife (27o-32o):
A. Bevel angle C. Both of these
B. Clearance angle D. Neither of these
39. This type of microtome easily cuts large blocks and serial sections can be obtained with ease because larger knives
can be used:
A. Sliding C. Rocking
B. Rotary D. Freezing
40. The cryostat is an apparatus used in fresh tissue microtomy. It consists of a microtome, kept inside a cold chamber
which has been maintained at a temperature of:
A. 20oC C. -20oC
B. 4 Co D. -4oC
41. Fat cells and enzymes are best demonstrated in:
A. Paraffin section C. Celloidin section
B. Plastic embedded section D. Frozen section
42. When trimming tissue block, they must be surrounded by at least __ of wax.
A. 1 mm C. 2 mm
B. 3 mm D. 4 mm

43. Most probable cause when clearing agent turns milky as soon as the tissue is placed in it:
A. Incomplete fixation C. Incomplete dehydration
B. Prolonged fixation D. Prolonged dehydration
44. It is added to Mayer’s egg albumin to prevent the growth of molds:
A. Sodium chloride C. Thymol crystals
3

B. Glycerol D. Powdered starch


45. To avoid distortion of the image, the refractive index of the mountant should be near as possible to that of the glass
which is:
A. 1. 581 C. 1.185
B. 1.518 D. 1.155
46. Deparaffinization of tissue sections is accomplished by passing through:
A. Ammonia water C. Alcohol
B. Acetone D. Xylol
47. Coverslips from slides may be removed by immersion in:
A. Ammonia water C. Alcohol
B. Acetone D. Xylol
48. Ripening of hematoxylin is a process of:
A. Hydrolysis C. Mordanting
B. Oxidation D. Reduction
49. Mercuric oxide or sodium iodate is used in hematoxylin to:
A. From hematein C. Prevent oxidation
B. Serve as the mordant D. Stabilize the solution
50. A commonly used connective tissue procedure that stains collagen blue:
A. Masson trichrome C Best carmine
B. van Gieson D. Aldehyde fuchsin
51. The phosphotungstic acid hematoxylin (PTAH) is useful for demonstrating:
A. Edema fluid C. Ground substance
B. Muscle striations D. Reticulin network
52. A method for demonstrating the nucleic acids, in which the DNA stains green and the RNA stains red:
A. Feulgen reaction C. Masson trichrome
B. Methyl green - pyronin D. Gomori trichrome
53. Thin sections for electron microscopy are stained with:
A. Hematoxylin and eosin C. Uranyl acetate and lead
B. Toluidine blue D. fluorescein and rhodamine
54. Feulgen’s reaction is the most reliable and most specific histochemical staining technique for:
A. RNA C. Both
B. DNA D. Neither
55. In the Masson-Fontana ammoniacal silver reaction, melanin and argentaffin cell granules are stained:
A. Brown C. Red
B. Black D. Blue
56. All of the following are fixative used for cytologic smears, EXCEPT:
A. Equal parts of 95% ETOH and ether C. 10% Formalin
B. 95% Ethyl alcohol D. Carnoy’s fluid
57. All of the following are examples of good adhesive agents for cytologic method, EXCEPT:
A. Egg albumin C. Pooled human serum or plasma
B. Celloidin ether alcohol D. Leuconostoc culture
58. Smears prepared for cytologic examination from cut surfaces of lymph nodes can be obtained by:
A. Spreading C. Touch preparation
B. Pull-Apart D. Streaking
59. Organisms found in the normal vaginal flora that stain BLUE to lavander with Pap’s method:
A. Trichomonas vaginalis C. Doderlein bacilli
B. Streptococci D. Acid fast bacilli
60. Clue cells are diagnostic of what infection:
A. Neisseria gonorrhoeae C. HSV-II
B. Gardnerella vaginalis D. Candida albicans
61. Cells found in a vaginal smear which are thick and round to oval in shape (similar to fried fresh eggs with sunny-side
up) with strongly basophilic cytoplasm and vesicular nucleus:
A. Superficial cells C. Parabasal cells
B. Intermediate cells D. Basal cells
62. A malignant tumor is least characterized by:
A. Metastases C. Encapsulation
B. Mitotic figures D. Invasion of adjacent tissues
63. Identified by the presence of pale, pink-staining cytoplasm and dark pyknotic nuclei:
A. Superficial cells C. Intermediate cells
B. Parabasal cells D. None of these
64. May occur in large groups or small sheets and may present a honeycomb appearance when viewed on end:
A. Navicular cells C. Endometrial cells
B. Pregnancy cells D. Endocervical glandular cells
65. If smears cannot be made immediately for cytology, the collected material should be placed in ___ for all types of
effusion.
A. 10% formalin C. Chloroform
B. Glacial acetic acid D. 50% alcohol

66. Absence of alveolar macrophages:


A. Saliva C. Both of these
B. Sputum D. None of these
67. Jelly-like clots in peritoneal, pleural and pericardial fluids may be prevented by addition of:
A. Acetic acid C. Alcohol
B. Formalin D. Heparin
68. For CSF specimens, a minimum of ____ is necessary for cytologic evaluation.
4

A. 1 cc C. 10 cc
B. 15 cc D. 20 cc
69. To obtain a more reliable cytological evaluation, urine specimen may have to be collected and examined twice – one
in the early morning and another later in the day. At least ___mL is needed which must be centrifuged. Smears of
sediments should be prepared and fixed as soon as possible after collection.
A. 10 mL C. 20 mL
B. 40 mL D. 50 mL
70. Ferning in vaginal smear is the basis for early detection of:
A. Early malignancy C. Menopausal cervix
B. Normal cervix D. Early pregnancy
71. Presence of large number of Doderlein bacilli may be found in which of the following conditions:
1. Last trimester of pregnancy 3. Infection
2. Diabetes mellitus 4. Estrogen deficiency
A. 1 and 3 C. 2 and 4
B. 1, 2 and 3 D. 1, 2, 3 and 4
72. Cytologic picture strongly suggestive of malignancy
A.Class II C. Class IV
B.Class III D. Class V
73. Increase in size of an organ or tissue due to an increase in the number of cells resulting from growth of new cells:
A.Hypoplasia C. Atrophy
B.Hypertrophy D. Hyperplasia
74. Marked regressive change in adult cells towards more primitive or embryonic cell types:
A.Metaplasia C. Dysplasia
B.Anaplasia D. Neoplasia
75. Continuous abnormal proliferation of cells without control:
A.Metaplasia C. Dysplasia
B.Anaplasia D. Neoplasia
76. Reversible change involving the transformation of one cell type to another:
A.Metaplasia C. Dysplasia
B.Anaplasia D. Neoplasia
77. Rigidity or stiffening of the muscles occurring 6-12 hours after death:
A.Algor mortis C. Livor mortis
B.Rigor mortis D. Autolysis
78. Inflammation characterized by the presence of large amount of pus:
A.Serous C. Fibrinous
B.Hemorrhagic D. Suppurative or purulent
79. Inflammation characterized by extensive outpouring of a watery, low-protein fluid from blood:
A.Serous C. Fibrinous
B.Hemorrhagic D. Suppurative or purulent
80. It is usually observed in skeletal muscles, heart, kidneys, endocrine organs and smooth muscles of hollow viscera due
to increased workload and endocrine stimulation (e.g. during exercise and pregnancy)
A. Compensatory hypertrophy C. True hypertrophy
B. False hypertrophy D. None of these
81. It is due to edema fluid and connective tissue proliferation (e.g. in cirrhosis and chronic hypertrophic salphingitis or
appendicitis):
A. Compensatory hypertrophy C. True hypertrophy
B. False hypertrophy D. None of these
82. Blood Banking Law of 1956:
A. RA 1517 C. RA 4688
B. RA 5527 D. PD 223
83. Clinical Laboratory Act of 1966:
A. RA 1517 C. RA 4688
B. RA 5527 D. PD 223
84. Creation of Professional Regulation Commission:
A. RA 1517 C. RA 4688
B. RA 5527 D. PD 223
85. The clinical laboratory law requires that:
A. The pathologist owns the clinical laboratory
B. The clinical laboratory is owned by a medical technologist
C. The clinical laboratory has a pathologist, medical technologist and medical laboratory technician
D. The clinical laboratory is headed by a pathologist
86. Who are the members of the Medical technology Board?
A. Pathologist, medical technologist and medical technician
B. Secretary of health, dean and president of PAMET
C. One internist, one pathologist and one medical technologist
D. A registered pathologist and two registered medical technologist
87. A medical technician to be registered must have obtained a grade within this range (Medical Technology Board
Exam):
A. 65-74.9% C. 60-74.9%
B. 70-74.9% D. 73-74.9%
88. Administrative investigations shall be conducted by:
A. At least one member of the Board C. At least one member of the Board w/one legal officer
B. Three members of the Board D. At least two members of the Board w/ one legal officer
89. The Commissioner of PRC is appointed by:
A. President of the Philippines C. Senate president
5

B. Speaker of the House D. Secretary of Health


90. Any member of the Medical Technology Board, if after due hearing, if found guilty of neglect of duty or incompetence
can be removed by:
A. President of the Philippines C. PRC chairman
B. CHED Commissioner D. Civil Service Commissioner
91. The penalty of revocation of certificate of registration may be imposed by the board if there is:
A. Majority vote C. Either of these
B. Unanimous vote D. Neither of these
92. National Blood Services Act of 1994:
A. RA 5527 C. RA 1517
B. RA 7719 D. RA 8504
93. Philippine AIDS Prevention and Control Act of 1998:
A. RA 7719 C. RA 8504
B. RA 1517 D. PD 223
94. Newborn Screening Act of 2004
A. RA 9288 C. RA 9165
B. RA 8504 D. RA 7719
95. All services are done on a primary category laboratory except: (A.O. 2007-0027)
A. Routine urinalysis C. Routine chemistry
B. Routine hematology D. Blood typing
96. Sections of RA 5527 amended by PD 1534:
A. 3, 8 and 13 C. 11, 16 and 29
B. 16, 21 and 22 D. 11, 21 and 29
97. The records of anatomic and forensic pathology shall be kept in the laboratory for:
A. Five years C. Ten years
B. Fifteen years D. Permanently
98. MT CODE OF ETHICS: Accept the responsibilities inherent to being a (98).
A. Professional C. Medical technologist
B. Filipino D. None of these
99. MT CODE OF ETHICS: Perform my task with (99) absolute reliability and accuracy.
A. Within constructive limits C. Full confidence
B. No conflict of interest D. None of these
100.MT CODE OF ETHICS: (100) of the professional organization and other allied health organizations.
A. Uphold the dignity C. Contribute to the advancement
B. Uphold the law D. None of these

END OF THE EXAM


PRE-BOARD EXAMINATION IN HISTOTECHNIQUES AND MEDICAL TECHNOLOGY LAWS (PART3)

1. A fixative made up of mercuric chloride stock solution to which glacial acetic acid has been added just before
use:a. Zenker’s b. Helly’s c. Orth’s d. Flemming’s
2. All of the following are components of Helly’s solution, except:
a. mercuric chloride b. potassium dichromate c. formaldehyde d. glacial acetic acid
3. An excellent microanatomic fixative for pituitary gland, bone marrow and blood-containing organs such as
spleen and liver:
a. Helly’s b. Orth’s c. Carnoy’s d. Heidenhain’s susa solution
4. A fixative recommended mainly for tumor biopsies especially of the skin:
a. Helly’s b. Orth’s c. Carnoy’s d. Heidenhain’ susa solution
5. It is recommended for study of early degenerative processes and tissue necrosis:
a. Carnoy’s b. Zenker’s c. Helly’s d. Orth’s
6. It demonstrates Rickettsia and other bacteria:
a. Carnoy’s b. Zenker’s c. Helly’s d. Orth’s
7. It is recommended for acid mucopolysaccharides:
a. formaldehyde b. mercurial fixatives c. lead fixatives d. picric acid fixatives
8. Picric acid fixatives:
a. Bouin’s solution b. Brasil’s fixative c. both of these d. none of these
9. Glacial acetic acid solidifies at:
a. 1 oC b. 10 oC c. 7 oC d. 17 oC
10. Excellent for fixing blood smears and bone marrow tissues:
a. formalin b. acetic acid c. picric acid d. methyl alcohol
11. Composition of Carnoy’s fluid:
a. mercuric chloride, acetic acid c. alcohol, chloroform, acetic acid
b. picric acid, formaldehyde, acetic acid d. potassium dichromate, formaldehyde
12. It is recommended for fixing chromosomes, lymph glands and urgent biopsies:
a. Carnoy’s b. Zenker’s c. Helly’s d. Orth’s
13. It is considered to be the most rapid fixative:
a. Carnoy’s b. Zenker’s c. Helly’s d. Orth’s
14. Swelling effect on tissues counteract shrinkage produced by other components/fixatives:
a. glacial acetic acid b. trichloroacetic acid c. both of these d. none of these
15. It is used in fixing brain tissues for diagnosis of rabies:
a. formalin b. methyl alcohol c. acetone d. acetic acid
16. It is used to remove excessive mercuric fixatives:
a. tap water b. 50-70% alcohol c. iodine d. none of these
17. It is most common decalcifying agent used, utilized both as simple solution or combined with other reagents:
a. nitric acid b. hydrochloric acid c. citric acid d. chromic acid
18. It decalcifies and softens tissues at the same time:
a. hydrochloric acid b. trichloroacetic acid c. Perenyi’s fluid d. sulfurous acid
19. Composition of Perenyi’s fluid:
a. NaCl soln, HCl, water c. chromic acid, osmium tetroxide
b. formic acid, formol saline d. nitric acid, chromic acid, ethyl alcohol
20. It may be used both as a fixative and decalcifying agent:
a. nitric acid b. hydrochloric acid c. citric acid d. chromic acid
21. A very expensive although the most ideal and most reliable method of determining the extent of decalcification:
a. physical test b. mechanical test c. chemical method d. radiological method
22. Best dehydrating agent because it is fast-acting, it mixes with water and penetrates tissue easily:
a. ethyl alcohol b. acetone c. cellosolve d. triethyl phosphate
23. Dehydrates and clears tissues:
a. dioxane b. tetrahydrofuran c. both of these d. none of these
24. A clearing agent which may damage the bone marrow resulting in aplastic anemia:
a. xylene b. toluene c. benzene d. chloroform
25. A clearing agent recommended for tough tissues, for nervous tissues, lymph nodes and embryos because it
causes minimum shrinkage and hardening of tissues:
a. xylene b. toluene c. benzene d. chloroform
26. A mixture of highly purified paraffin and synthetic plastic polymers with a melting point of 56-57 oC.
a. celloidin b. paraplast c. ester wax d. carbowax
27. It is recommended for bones, teeth, large brain sections and whole organs
a. wet celloidin method b. dry celloidin method c. both of these d. none of these
28. It is preferred for processing of whole eye sections:
a. wet celloidin method b. dry celloidin method c. both of these d. none of these
29. Gilson’s mixture made up of equal parts of chloroform and cedarwood oil, is added to the celloidin block before
hardening, to make the tissue transparent:
a. wet celloidin method b. dry celloidin method c. both of these d. none of these
30. The angle formed between the cutting edges, normally between 27 to 32o:
a. bevel angle b. clearance angle c. both of these d. none of these
31. It involves the removal of gross nicks on the knife edge, to remove blemishes, and grinding the cutting edge of
the knife on a stone:
a. honing b. stropping c. both of these d. none of these
32. It involves the removal of burr and for final polishing of the knife edge:
a. honing b. stropping c. both of these d. none of these
33. The knife is then turned over, and the other surface is again drawn forward, edge first, with a HEEL TO TOE
direction:
a. honing b. stropping c. both of these d. none of these
2

34. It is used for cutting celloidin embedded sections:


a. freezing microtome b. rocking microtome c. rotary microtome d. sliding microtome
35. This was invented by Paldwell Trefall in 1881, the simplest among the different types of microtomes:
a. freezing microtome b. rocking microtome c. rotary microtome d. sliding microtome
36. This was invented by Minot in 1885-86 to cut paraffin embedded tissues, and is the most common type used for
both routine and research laboratories:
a. freezing microtome b. rocking microtome c. rotary microtome d. sliding microtome
37. This was developed by Adam in 1789:
a. freezing microtome b. rocking microtome c. rotary microtome d. sliding microtome
38. This was invented by Queckett in 1848:
a. freezing microtome b. rocking microtome c. rotary microtome d. sliding microtome
39. It is added to Mayer’s egg albumin to prevent the growth of molds:
a. gelatin b. egg white c. glycerin d. thymol
40. A natural dye derived by extraction from the heartwood of a Mexican tree:
a. carmine b. hematoxylin c. orcein d. saffron
41. A vegetable dye extracted from certain lichens:
a. carmine b. hematoxylin c. orcein d. saffron
42. Alum hematoxylin solutions:
a. Ehrlich’s hematoxylin b. Harris hematoxylin c. both of these d. none of these
43. An excellent stain for elastic fibers (Taenzer Unna), especially recommended in dermatological studies due to
its ability to demonstrate the finest and most delicate fibers in the skin:
a. carmine b. orcein c. Prussian blue d. picric acid
44. A process by which sections are stained with simple aqueous or alcoholic solution of the dye:
a. direct staining b. indirect staining c. progressive staining d. regressive staining
45. Tissue is first overstained to obliterate the cellular details, and the excess stain is removed or decolorized from
unwanted parts of the tissue:
a. direct staining b. indirect staining c. progressive staining d. regressive staining
46. In the Masson Fontana technique, melanin is stained:
a. red b. blue c. green d. black
47. In the von Kossa silver nitrate method, calcium is stained:
a. red b. blue c. green d. black
48. Most reliable and specific histochemical staining technique for DNA:
a. von Kossa b. Levaditi’s c. Sakaguchi’s d. Feulgen’s
49. In the methyl green pyronin stain for nucleic acids, DNA is stained:
a. green/blue-green b. rose-red c. purple d. black
50. In the methyl green pyronin stain for nucleic acids, RNA is stained:
a. green/blue-green b. rose-red c. purple d. black
51. All of the following are considered as good adhesive agents for cytologic method, except:
a. pooled serum or plasma b. egg albumin c. celloidin ether alcohol d. Leuconostoc culture
52. All of the following are fixatives for cytologic smears, except:
a. ethyl alcohol and ether b. 95% ethyl alcohol c. Carnoy’s d. formalin
53. Incomplete or defective development of a tissue or organ, represented only by a mass of fatty or fibrous tissue,
bearing no resemblance to the adult structure:
a. agenesia b. aplasia c. atresia d. atrophy
54. It refers to the complete non-appearance of an organ:
a. agenesia b. aplasia c. atresia d. atrophy
55. Failure of an organ to form an opening:
a. agenesia b. aplasia c. atresia d. atrophy
56. An acquired decrease in size of a normally developed or mature tissue or organ resulting from reduction in cell
size or decrease in total number of cells or both:
a. agenesia b. aplasia c. atresia d. atrophy
57. A reversible change involving the transformation in one type of adult cell to another:
a. anaplasia b. dysplasia c. metaplasia d. neoplasia
58. Regressive alteration in adult cells manifested by variation in size, shape and orientation:
a. anaplasia b. dysplasia c. metaplasia d. neoplasia
59. Dedifferentiation:
a. anaplasia b. dysplasia c. metaplasia d. neoplasia
60. Marked regressive change in adult cells towards more primitive or embryonic cell types:
a. anaplasia b. dysplasia c. metaplasia d. neoplasia
61. Continuous abnormal proliferation of cells without control:
a. anaplasia b. dysplasia c. metaplasia d. neoplasia
62. It is characterized by cooling of the body to equalize that of the surrounding environment, occurring at a definite
rate, about 7 oF per hour and usually important in establishing the approximate time of death:
a. algor mortis b. livor mortis c. rigor mortis d. dessication
63. This refers to the rigidity or stiffening of the muscles occurring 6 to 12 hours after death and persisting for 3 to 4
days:
a. algor mortis b. livor mortis c. rigor mortis d. dessication

64. This refers to the purplish discoloration or lividity of the skin after death:
a. algor mortis b. livor mortis c. rigor mortis d. dessication
65. This refers to the drying and wrinkling of the cornea and anterior chamber of the eye due to the absorption of
the aqueous humor.
a. algor mortis b. livor mortis c. rigor mortis d. dessication
66. Identified by the presence of pale, pink-staining cytoplasm and dark pyknotic nuclei:
3

a. superficial cells b. intermediate cells c. parabasal cells d. none of these


67. The penalty of revocation of certificate of registration may be imposed by the board if there is:
a. Majority vote b. Unanimous vote c. Either of these d. Neither of these
68. National Blood Services Act of 1994:
a. RA 5527 b. RA 1517 c. RA 7719 d. RA 8504
69. Newborn Screening Act of 2004:
a. RA 9502 b. RA 9288 c. RA 9211 d. RA 9165
70. What are the disorders included in the newborn screening?
1. congenital hypothyroidism (CH) 3. congenital adrenal hyperplasia (CAH) 5. phenylketonuria
2. galactosemia 4. glucose-6-phosphate dehydrogenase (G6PD) deficiency
a. 1, 3 and 5 b. 1, 3, 4 and 5 c. 1, 2 and 5 d. AOTA
71. Philippine AIDS Prevention and Control Act of 1998:
a. RA 7719 b. RA 8504 c. RA 1517 d. PD 223

72 – 79:
A. RA 1517 B. RA 4688 C. RA 5527 D. PD 223
72. Philippine Medical technology Act
73. Blood Banking Law
74. Clinical Laboratory Act -
75. Creation of Professional Regulation Commission
76. June 22, 1973
77. June 21, 1969
78. June 18, 1966
79. June 16, 1956

80 – 88: SERVICE CAPABILITY OF LABORATORIES


Revised Rules and Regulations Governing the Licensure and Regulation of Clinical Laboratories in the Philippines
(August 22, 2007)

80. Blood typing – for hospital-based A. Done in primary secondary and tertiary laboratories
81. Special Hematology B. Done in secondary and tertiary laboratories
82. Immunology C. Done only in a tertiary laboratory
83. Microbiology – culture and sensitivity
84. Routine Clinical Chemistry
85. Routine Fecalysis
86. Routine Hematology
87. Routine Urinalysis
88. Special Chemistry

89 to 100: FILL IN THE BLANKS: CODE OF ETHICS

A. Contribute to the advancement G. Ethics committee


B. Knowledge and expertise H. No conflict of interest
C. Professional I. Promote life and benefit mankind
D. Spirit of brotherhood J. Strictly confidential;
E. Uphold the dignity K. Uphold the law
F. Within constructive limits L. Full confidence

As I enter into the practice of Medical Technology, I shall:

▪ Accept the responsibilities inherent to being a (89).


▪ (90) and shall not participate in illegal work.
▪ Act in a strict spirit of fairness to all and in a (91) toward other members of the profession.
▪ Accept employment from more than one employer only when there in (92).
▪ Perform my task with (93) absolute reliability and accuracy.
▪ Share my (94) with my colleagues.
▪ (95) of the professional organization and other allied health organizations.
▪ Restrict my praises, criticisms, views and opinions (96).
▪ Treat any information I acquired in the course of my work as (97).
▪ (98) and respect of my profession and conduct myself a reputation of reliability, honesty and integrity.
▪ Be dedicated to the use of clinical laboratory science (99).
▪ Report any violations of the above principles of the professional conduct to authorized agency and to the (100).

END OF THE EXAM


PRE-BOARD EXAMINATION IN IMMUNOLOGY-SEROLOGY AND BLOOD BANKING (PART 1)

1. Metchnikoff first described which of the following?


A. Phagocytosis B. Variolation C. Humoral immunity D. Opsonization
2. Jenner’s work with cowpox, which provided immunity against smallpox, demonstrates which phenomenon?
A. Natural Immunity B. Cross-immunity C. Attenuation of vaccines D. Reactivity of haptens
3. Chronic granulomatous disease represents a defect of:
A. Oxidative metabolism C. Diapedesis
B. Abnormal granulation of neutrophils D. Chemotaxis
4. The major role of neutrophils is phagocytosis. Which one of the following events is not associated with some aspect of
neutrophil function?
A. Recognition of antigen via primitive pattern receptor patterns
B. Recognition of opsonins on bacteria
C. Secretion of perforin
D. Activation of the NADPH oxidase
5. Which one of the following cells destroys tumor cells using ADCC as a recognition mode, and perforin as an effector
molecule?
A. B cells B. CD4+ cells B. CD8+ cells D. NK cells
6. Which of the following is a potent mediator in acute-phase response?
A. IL-1 B. IL-2 C. IL-3 D. IL-4
7. Which of the following enhances the cytolytic activity of lymphokine-activated killer cells (LAK)?
A. IL-1 B. IL-2 C. IL-3 D. IL-4
8. Which of the following stimulates hematopoietic cells?
A. IL-1 B. IL-2 C. IL-3 D. IL-4
9. Which one of the following cells recognizes a cell surface complex consisting of antigenic peptide complexed with an
MHC protein?
A. Phagocytes B. Eosinophils C. T cells D. B cells
10. Which one of the following activates both T and B cells?
A. PHA B. Con A C. LPS D. PWM
11. The type of immunity that follows the injection of an antigen is:
A. Adaptive B. Active C. Passive D. Innate
12. Which of the following is true of MHC (HLA) class II antigens?
A. They are found on all nucleated cells C. They all originate at one locus
B. They are found on B cells and macrophages D. They are coded on chromosome 9
13. Bence-Jones proteins are identical:
A. H chains B. L chains C. IgM molecules D. IgG molecules
14. Mannose-binding protein in the lectin pathway is most similar to which classical component pathway component?
A. C3 B. C1rs C. C1q D. C4
15. In the complement fixation procedure, a negative result is manifested by:
A. Antigen-binding C. Lysis of sheep red blood cells
B. Lysis of guinea pig cells D. Agglutination of sheep red blood cells
16. A positive direct Coomb’s test could occur under which circumstances?
A. Hemolytic disease of the newborn C. Antibodies to drug that bind to red cells
B. Autoimmune hemolytic anemia D. Any of the above
17. Which one of the following antibody isotypes is captured by Protein A?
A. IgG B. IgA C. IgM D. IgD
18. To determine id a patient is allergic to rye grass, the best test to perform is:
A. RAST B. RIST C. DAT D. Complement fixation
19. What is the immune phenomenon associated with Arthus reaction?
A. Tissue destruction by cytotoxic T cells
B. Removal of antibody-coated red blood cells
C. Deposition of immune complexes in blood vessels
D. Release of histamine from mast cells
20. The Mantoux test is an example of:
A. Type I hypersensitivity C. Type III hypersensitivity
B. Type II hypersensitivity D. Type IV hypersensitivity
21. Anaphylaxis as a result of bee sting is an example of:
A. Type I hypersensitivity C. Type III hypersensitivity
B. Type II hypersensitivity D. Type IV hypersensitivity
22. What immune elements are involved in a reaction to poison ivy?
A. IgE antibodies B. T cells and macrophages C. NK cells and IgG D. B cells and IgM
23. What antibodies are represented by the peripheral or rim pattern of IF tests for ANA?
A. Anti-histone antibodies C. Anti-ENA antibodies
B. Anti-dsDNA antibodies D. Anti-RNA antibodies
24. Destruction of the myelin sheath of axons caused by the presence of antibody is characteristic of which disease?
A. Multiple sclerosis C. Graves’ disease
B. Myasthenia gravis D. Goodpasture’s syndrome
25. It is suggestive of Goodpasture’s disease:
A. Acetylcholine receptor-blocking antibodies C. Anti-DNA antibodies
B. Anti-cardiolipin antibodies D. Anti-glomerular basement membrane antibodies
26. It is strongly suggestive, in a high titer, of primary biliary cirrhosis:
A. Anti-myelin antibody C. Anti-centromere antibody
B. Anti-intrinsic factor antibody D. Anti-mitochondrial antintibody
2

27. A defect in C1INH results in which one of the following disorders?


A. Bruton’s agammaglobulinemia C. Chronic granulomatous disease
B. Selective IgA deficiency D. Hereditary angioneurotic edema
28. Individuals who are at risk for ankylosing spondylitis have inherited which one of the following alleles?
A. HLA-A3 B. HLA-B8 C. HLA-B27 D. HLA-B7
29. Individuals who are at risk for rheumatoid arthritis have inherited which one of the following alleles?
A. HLA-A3 B. HLA-B27 C. HLA-B7 D. HLA-DR4
30. A kidney transplantation between one identical twin to another is an example of:
A. An allograft B. An autograft C. A heterograft D. A syngeneic graft
31. CA-15.3 is used conditionally in the monitoring of:
A. Pancreatic adenocarcinoma C. Breast adenocarcinoma
B. Colonic adenocarcinoma D. Hairy cell leukemia
32. A biological false-positive reaction is least likely with which test for syphilis?
A. VDRL C. RPR
B. FTA-ABS D. All are equally likely to detect a false positive
33. A 24-year-old man who had just recovered from infectious mononucleosis had evidence of a genital lesion. His RPR
was positive. What should the technologist do next?
A. Report out as false positive
B. Do a confirmatory treponemal test
C. Do a VDRL
D. Have the patient return in 2 weeks for a repeat test
34. The serologic marker during the “window period” of hepatitis B is:
A. Anti-HBs B. Anti-HBc C. Anti-HBe D. HBsAg
35. The specific diagnostic test for hepatitis C is:
A. Absence of anti-HAV and anti-HBs C. Detection of non-A, non-B antibodies
B. An increase in serum ALT D. Anti-HCV
36. Antibodies to which of the following retroviral antigens are usually the first to be detected in HIV infection?
A. gp120 B. gp160 C. gp41 D. p24
37. Which of the following combinations of bands would represent a positive Western blot for HIV antibody?
A. p24 and p55 B. p24 and p31 C. gp41 and gp120 D. p31 and p55
38. The confirmation of a heterophile antibody of infectious mononucleosis would be:
A. Agglutination with beef erthrocytes
B. Agglutination of sheep cells after incubation with guinea pig cells; no agglutination of sheep cells after
incubation with beef erythrocytes
C. Agglutination of sheep cells after incubation with beef erythrocytes; no agglutination of sheep cells after
incubation with guinea pig cells
D. Agglutination with guinea pig cells
39. Which of the following identifies the pattern of antibody cross-reactivity that is generated during infection with R.
rickettsii?
A. P. vulgaris OX-19 (+), P. vulgaris OX-2 (+), P. mirabilis OX-K (-)
B. P. vulgaris OX-19 (-), P. vulgaris OX-2 (+), P. mirabilis OX-K (-)
C. P. vulgaris OX-19 (-), P. vulgaris OX-2 (+), P. mirabilis OX-K (+)
D. P. vulgaris OX-19 (-), P. vulgaris OX-2 (-), P. mirabilis OX-K (+)
40. The least immunogenic transplant tissue:
A. Bone marrow B. Cornea C. Heart D. Skin
41. Streptococcus MG agglutinins occur in normal serum at low titers (1:10). A titer of 400 or greater is considered to be
suggestive of:
A. Paroxysmal cold hemoglobinuria C. Lupus erythematosus
B. Primary atypical pneumonia D. Rheumatoid arthritis
42. The most common cause of congenital infections in humans, affecting 0.5 to 2.4% live births:
A. Cytomagelovirus B. Rubella C. Toxoplasmosis D. Hepatitis
43. Which one of the following tests measures the production of parasitic lactate dehydrogenase?
A. RIDASCREEN Entamoeba D. OptiMal Malaria test
B. ProSpect Entamoeba histolytica E. Bordier Immunoassay for E. granulosus
C. MalaQuick Standby Malaria test
44. Hybridoma is produced from the fusion of:
A. Natural killer cell and plasma cell C. Myeloma cell and plasma cell
B. T cell and plasma cell D. Myeloma cell and T cell
45. Sensitivity:
A. The detection of specific antibody in the serum of an individual in whom the antibody was previously
undetectable
B. The frequency of positive results obtained in the testing of a population of individuals who are truly positive for
antibody
C. The proportion of negative test results obtained in the population of individuals who actually lack the antibody in
question
D. The time of recovery from conditions such as illness, injury or surgery
46. Which of the following conditions can result in rouleaux formation or pseudoagglutination:
1. Elevated levels of globulin 3. Presence of plasma expanders
2. Elevated levels of fibrinogen 4. Presence of Wharton’ s jelly
A. 1 and 3 C. 1, 2 and 3
B. 2 and 4 D. 1, 2, 3 and 4
3

47. Determine what incompatibility is demonstrated:


Group A (donor) with group O (patient):
A. Incompatible in minor crossmatch C. Both of these
B. Incompatible in major crossmatch D. None of these
48. Inheritance of Sese and the Lewis gene produces the following phenotype:
A. Le (a+b-) B. Le (a+b+) C. Le (a-b+) D. Le (a-b-)
49. Which Duffy phenotype offers the greatest resistance to invasion by malarial parasites?
A. Fy (a+b-) B. Fy (a+b+) C. Fy (a-b+) D. Fy (a-b-)
50. A previously named HLA that is not uncommonly detected on erythrocytes is:
A. Dia B. Sda C. Bga D. Coa
51. A low-incidence antigen that serves as a useful anthropologic marker for Mongolian ancestry:
A. Xga B. Doa C. Dia D. Yta
52. Mutations in the carrier molecule for this blood group system may result in changes of reb blood cell shape in the
forms of acanthocytosis or ovalocytosis?
A. DI B. DO C. CO D. SC
53. Antigen is found on the petite arm of the X chromosome and is noted with higher frequency in females than in males.
A. Xga B. Doa C. Dia D. Yta
54. Rh immune globulin provides ____ protection against fetal D antigen.
A. Active B. Passive C. Antigen-stimulated D. Antibody-stimulated
55. If an Rh negative woman recently delivered an Rh positive baby and the Kleihauer-Betke test result is 5%, how many
vials of Rh Ig should be administered?
A. 6 B. 7 C. 8 D. 9
56. If a prospective allogeneic donor has received blood or blood components known to be sources of hepatitis (e.g.,
surgery), the donor should be deferred from donating for _____ after the transfusion.
A. 6 weeks B. 3 months C. 6 months D. 12 months
57. Minimum number of platelets in a platelet concentrate prepared from whole blood by centrifugation:
A. 5.5 x 1011 B. 3.0 x 10 10 C. 3.0 x 1011 D. 5.5 x 1010
58. Additive solutions are approved for blood storage for how many days?
A. 21 days B. 42 days C. 35 days D. 7 days
59. Graft-versus-host disease is caused by:
A. Granulocytes B. Platelets C. Lymphocytes D. Erythrocytes
60. The radiation source for irradiation of blood products is:
A. 131I B. 137Ce C. 14C D. 131Te
61. Once defrosted, cryoprecipitate must be administered within _____ hours of thawing.
A. 2 B. 4 C. 6 D. 12
62. Perfluorocarbons have been investigated as:
A. Platelet substitutes C. Red blood cell substitutes
B. Granulocyte substitutes D. Plasma substitutes
63. Allogeneic donor blood collected and processes from outside sources must have the following tests repeated by the
hospital blood bank:
1. ABO 2. Rh 3. HBsAg 4. Anti-HIV1
A. 1 and 2 B. 3 and 4 C. 1, 2 and 3 D. All
64. The minimum hemoglobin concentration in g/dL in a fingerstick from a male blood donor is:
A. 12.0 B. 13.5 C. 12.5 D. 15.0
65. The required hemoglobin and hematocrit for autologous donation should be at least:
A. 11 g/dL hgb, 33% hct C. 12.5 g/dL hgb, 33% hct
B. 11 g/dL hgb, 38% hct D. 12.5 g/dL hct, 38% hct
66. Autologous blood donor units must be tested for:
1. ABO 2. Rh 3. HBsAg 4. Anti-HIV1
A. 1 and 2 B. 3 and 4 C. 1, 2 and 3 D. All
67. Samples of recipient’s blood and donor units must be stored for _____ days after transfusion.
A. 1 B. 3 C. 5 D. 7
68. A febrile transfusion reaction is defined as a rise in body temperature of _____ occurring in association with the
transfusion of blood or components and without any other explanation.
A. 1 oC or more B. 1 oF or more C. 5 oC or more D. 5 oF or more
69. Blood component most frequently associated with transfusion reaction due to bacterial contamination:
A. Red cells C. Cryoprecipitate
B. Fresh frozen plasma D. Platelet concentrate
70. The most common cause of transfusion-related sepsis is:
A. Whole blood B. Platelet concentrates C. Packed red cells D. Leukocytes conc.
71. Of the deaths caused by bacterial contamination of blood components reported to Centers for Disease Control (CDC),
most are caused by blood components contaminated by:
A. Escherichia coli C. Yersinia enterocolitica
B. Pseudomonas species D. None of these
72. Polyspecific AHG reagent contains:
A. Anti-IgG B. Anti-IgG and anti-IgM C. Anti-IgG and anti-C3d D. Anti-C3d
73. A positive DAT may be found in which of the following situations?
A. A weak-D positive patient C. Hemolytic disease of the newborn
B. A patient with anti-K D. An incompatible crossmatch
74. Each unit of whole blood will yield approximately how many units of cryoprecipitated AHF?
A. 40 B. 130 C. 80 D. 250
4

75. According to AABB standards, 75% of all platelets, pheresis units shall contain how many platelets per uL?
A. 5.5 x 1010 B. 3.0 x 1011 C. 6.5 x 1010 D. 5.5 x 1011
76. Which of the following blood components is the best source of factor IX?
A. Prothrombin complex C. Fresh frozen plasma
B. Cryoprecipitated AHF D. Single-donor plasma
77. Hives and itching are symptoms of which of the following transfusion reactions?
A. Febrile B. Circulatory overload C. Allergic D. Anaphylactic
78. Cold agglutinin syndrome is best associated with which of the following blood groups?
A. Duffy B. Ii C. P D. Rh
79. Rejuvenation of a unit of red blood cells is a method used to:
A. Remove antibody attached to rbc C. Restore 2,3 DPG and ATP to normal levels
B. Inactivate viruses and bacteria D. Filter blood clots and other debris
80. According to AABB standards, what is the minimum pH required for platelets?
A. 4 B. 6 C. 5 D. 7
81. Which of the following transfusion reactions occurs after infusion of only a few milliliters of blood and gives no history
of fever?
A. Febrile B. Anaphylactic C. Circulatory overload D. Hemolytic
82. Which of the following antigens gives enhanced reactions with its corresponding antibody following treatment of the
red cells with proteolytic enzymes?
A. Fya B. S C. E D. M
83. A lectin with anti-N specificity can be made from:
A. Bandeirae simplicifolia C. Iberis amara
B. Dolichos biflorus D. Vicia graminea
84. Which of the following would be the component of choice for treatment of von Willebrand’s disease?
A. Platelets C. Cryoprecipitated AHF
B. Factor IX concentrate D. Fresh frozen plasma
85. If the seal is entered on a unit of whole blood stored at 1-6 oC, what is the maximum allowable storage period, in
hours?
A. 6 B. 48 C. 24 D. 72
86. The drug cephalosporin can cause a positive direct antiglobulin test by which of the following mechanisms?
A. Immune-complex formation C. Autoantibody production
B. Complement fixation D. Membrane modification
87. Which of the following is a characteristic of anti-I?
A. Often associated with HDN C. Reacts best at 37 oC
B. Frequently a cold agglutinin D. Is usually IgG
88. The mechanism that best explains hemolytic anemia due to penicillin is:
A. Drug adsorption C. Immune complex formation
B. Membrane modification D. Autoantibody production
89. Posttransfusion anaphylactic reactions occur often in patients with:
A. Leukocyte antibodies C. IgA deficiency
B. Erythrocyte antibodies D. Factor VIII deficiency
90. Hydroxyethyl starch (HES) is a rouleaux-promoting agent used to:
A. Increase the harvest of granulocytes in leukapheresis
B. Treat patients following hemolytic transfusion reaction
C. Resolve ABO typing discrepancies
D. Stabilize the pH of stored platelets
91. Which of the following is the proper storage temperature requirements for granulocytes?
A. 1 to 6 oC C. Room temperature with constant agitation
B. 10 to 18 oC D. Room temperature without agitation
92. Which of the following best reflects the discrepancy seen when a person’s red cells demonstrated the acquired-B
phenotype?
Forward Grouping Reverse Grouping
A. B O
B. AB A
C. O B
D. B AB
93. The process of separation of antibody from its antigen is known as:
A. Diffusion B. Lyophilization C. Absorption D. Elution
94. To validate the reaction obtained in the antiglobulin test, one can:
A. Use green antiglobulin reagent
B. Add IgG-coated red cells to each test tube
C. Add IgG-coated red cells to each positive reaction
D. Add IgG-coated red cells to each negative reaction
95. This type of transfusion reaction may occur in IgA-deficient patients who demonstrate potent IgG anti-IgA and who are
exposed to IgA containing plasma products:
A. Anaphylactic B. Circulatory overload C. Allergic D. Hemolytic
96. An iron chelating agent which is important in lowering the body iron stores of patients with thalassemia:
A. Deferroxamine B. Desmopressin C. Steroids D. Aspirin
97. For autologous blood donation, blood should not be drawn from the donor-patient within _____ hours of the time of
the anticipated operation or transfusion.
A. 12 hours B. 24 hours C. 48 hours D. 72 hours
5

98. Paroxysmal cold hemoglobinuria is often associated with antibodies in which system?
A. MNS B. P C. Lewis D. Rh
99. Cryoprecipitated antihemophilic factor (AHF) is not recommended for the treatment of:
A. Hemophilia A B. Hemophilia B C. vWD D. Hypofibrinogenemia
100. Which of the following is usually employed to start an IV liner prior to blood transfusion?
1. Normal (0.9%) saline 3. 5% Dextrose in water (D5W)
2. Ringer’s lactate 4. Distilled water
A. 1 only C. 1, 2 and 3
B. 1 and 3 D. 1, 2, 3 and 4

END OF THE EXAM


PRE-BOARD EXAMINATION IN IMMUNOLOGY-SEROLOGY AND BLOOD BANKING (PART 2)

1. Macrophages have specific names according to their tissue location. Macrophages in the liver are:
A. Alveolar macrophages C. Microglial cells
B. Kupffer cells D. Histiocytes
2. In which area of the lymph node are T cells mainly found:
A. Germinal center C. Paracortex
B. Primary follicles D. Sinusoids
3. Lymphocyte mitogen that acts on both B cell and T cell
A. E. coli endotoxin C. Concanavalin A (ConA)
B. Lipopolysaccharides D. Pokeweed mitogen (PWM)
4. Antigen given intravenously are mainly trapped in the:
A. Lung C. Spleen
B. Lymph node D. Tonsils
5. Nitroblue tetrazolium test is of value in the diagnosis of:
A. Bruton’s agammaglobulinemia C. Kahler’s disease
B. Franklin’s disease D. Chronic granulomatous disease
6. Variations in variable regions that give individual antibody molecules specificity:
A. Isotype C. Allotype
B. Idiotype D. All of these
7. The hinge region of an immunoglobulin is flexible because it contains a large amount of which amino acid?
A. Serine C. Threonine
B. Cystine D. Proline
8. Secretory IgA contains:
1. Four light chains 3. Secretory component
2. Four heavy chains 4. One J chain
A. 1 and 3 C. 1, 2 and 3
B. 2 and 4 D. 1, 2, 3 and 4
9. Complement component C3 is cleaved by:
A. C3b C. C3bBb
B. Factor B D. Factor D
10. Which of the following statements does not apply to IgG?
A. Appears early in the immune response
B. Can fix complement
C. Crosses the human placenta
D. Opsonizes bacteria
11. The key cells involved in mediating delayed hypersensitivity are:
A. Neutrophils C. T cells
B. Mast cells D. B cells
12. RAST measures:
A. Antigen concentration C. IgE antibodies
B. IgM antibodies D. IgG antibodies
13. Helper T cells recognize antigen on antigen-presenting cells as antigen:
A. With MHC class I product C. With both class I and class II products
B. With MHC class II product D. With complement
14. MHC class II molecules are found on:
A. Virtually all cells in the body
B. B cells, dendritic cells and macrophages
C. Virtually all nucleated cells in the body
D. Only on virally infected cells
15. At least one of the functions of CD8 on the surface of lymphocytes interacting with another cell is to:
A. Bind antigen C. Bind to MHC class I molecules
B. Bind surface immunoglobulin D. Bind to MHC class II molecules
16. Transfer of tissues and organs between genetically identical individuals such as identical twins:
A. Autograft C. Xenograft
B. Syngraft D. Allograft
17. Fetus can be considered as an:
A. Autograft C. Isograft
B. Allograft D. Xenograft
18. Transplanted cells are mainly destroyed by:
A. B cells C. Macrophages
B. T cells D. Neutrophils
19. Which term applies to the most accelerated type of graft rejection?
A. Acute rejection C. Hyperacute rejection
B. Chronic rejection D. Accelerated rejection
20. Carcinoembryonic antigen is characteristically secreted by tumors of the:
A. Kidney C. Lungs
B. Bones D. Gastrointestinal tract
21. LAK (lymphokine-activated killer) cells are induced by:
A. Interleukin 1 C. Tumor necrosis factor - α
B. Interleukin 2 D. Interferon - γ
2

22. A hybridoma is a cell formed by fusion of a:


A. Plasma cell with a plasma cell of another species
B. T cell with a myeloma cell
C. Plasma cell with a myeloma cell
D. T cell with a B cell
23. The prozone phenomenon can:
1. Result from excessive antibody concentration
2. Result in false positive reaction
3. Result in false negative reaction
4. Be overcome by serially diluting the antibody containing serum
A. 1 and 3 B. 2 and 4 C. 1, 2 and 4 D. 1, 3 and 4
24. Reverse passive agglutination:
1. Antigen is attached to carrier particle
2. Antibody is attached to carrier particle
3. Agglutination occurs if patient antibody is present
4. Agglutination occurs if patient antigen is present
A. 1 and 3 B. 2 and 4 C. 1, 2, and 3 D. 1, 2, 3 and 4
25. ANA pattern associated with SLE in the active stage of the disease:
A. Diffused or homogenous pattern C. Peripheral pattern
B. Nucleolar pattern D. Speckled pattern
26. Jenner’s work with cowpox, which provided immunity against smallpox, demonstrates which phenomenon?
A. Natural Immunity B. Cross-immunity C. Attenuation of vaccines D. Reactivity of haptens
27. In searching for a cure for TB, Koch was first to observe which phenomenon?
A. Bacterial aggln. B. Precipitation C. Phagocytosis D. Delayed hypersensitivity
28. Chronic granulomatous disease represents a defect of:
A. Oxidative metabolism C. Diapedesis
B. Abnormal granulation of neutrophils D. Chemotaxis
29. Which one of the following cells destroys tumor cells using ADCC as a recognition mode, and perforin as an
effector molecule?
A. B cells B. CD4+ cells B. CD8+ cells D. NK cells
30. Measurement of CRP levels can be used for all of the following except:
A. Monitoring drug therapy with anti-inflammatory agents
B. Tracking the normal progress of surgery
C. Diagnosis of specific bacterial infection
D. Determining the active phases of rheumatoid arthritis
31. Which one of the following cells recognizes a cell surface complex consisting of antigenic peptide complexed
with an MHC protein?
A. Phagocytes B. Eosinophils C. T cells D. B cells
32. In the complement fixation procedure, a negative result is manifested by:
A. Antigen-binding C. Lysis of sheep red blood cells
B. Lysis of guinea pig cells D. Agglutination of sheep red blood cells
33. The Mantoux test is an example of:
A. Type I hypersensitivity C. Type III hypersensitivity
B. Type II hypersensitivity D. Type IV hypersensitivity
34. Anaphylaxis as a result of bee sting is an example of:
A. Type I hypersensitivity C. Type III hypersensitivity
B. Type II hypersensitivity D. Type IV hypersensitivity
35. What immune elements are involved in a reaction to poison ivy?
A. IgE antibodies B. T cells and macrophages C. NK cells and IgG D. B cells and IgM
36. Rheumatoid factor is directed against which of the following molecules?
A. Whole IgM molecules C. Fab region of the IgG molecule
B. Fc portion of the IgG molecule D. Fc portion of the IgM molecule
37. Destruction of the myelin sheath of axons caused by the presence of antibody is characteristic of which
disease?
A. Multiple sclerosis C. Graves’ disease
B. Myasthenia gravis D. Goodpasture’s syndrome
38. It is suggestive of Goodpasture’s disease:
A. Acetylcholine receptor-blocking antibodies C. Anti-DNA antibodies
B. Anti-cardiolipin antibodies D. Anti-glomerular basement membrane antibodies
39. It is strongly suggestive, in a high titer, of primary biliary cirrhosis:
A. Anti-myelin antibody C. Anti-centromere antibody
B. Anti-intrinsic factor antibody D. Anti-mitochondrial antintibody
40. A defect in C1INH results in which one of the following disorders?
A. Bruton’s agammaglobulinemia C. Chronic granulomatous disease
B. Selective IgA deficiency D. Hereditary angioneurotic edema
41. Individuals who are at risk for ankylosing spondylitis have inherited which one of the following alleles?
A. HLA-A3 B. HLA-B8 C. HLA-B27 D. HLA-B7
42. Individuals who are at risk for rheumatoid arthritis have inherited which one of the following alleles?
A. HLA-A3 B. HLA-B27 C. HLA-B7 D. HLA-DR4
43. A kidney transplantation between one identical twin to another is an example of:
A. An allograft B. An autograft C. A heterograft D. A syngeneic graft
3

44. CA-15.3 is used conditionally in the monitoring of:


A. Pancreatic adenocarcinoma C. Breast adenocarcinoma
B. Colonic adenocarcinoma D. Hairy cell leukemia
45. In infected blood, Treponema pallidum does not appear to survive at 4oC for more than:
A. 1 day B. 2 days C. 3 days D. 4 days
46. The serologic marker during the “window period” of hepatitis B is:
A. Anti-HBs B. Anti-HBc C. Anti-HBe D. HBsAg
47. Antibodies to which of the following retroviral antigens are usually the first to be detected in HIV infection?
A. gp120 B. gp160 C. gp41 D. p24
48. Which of the following combinations of bands would represent a positive Western blot for HIV antibody?
A. p24 and p55 B. p24 and p31 C. gp41 and gp120 D. p31 and p55
49. Which of the following identifies the pattern of antibody cross-reactivity that is generated during infection with
R. rickettsii?
A. P. vulgaris OX-19 (+), P. vulgaris OX-2 (+), P. mirabilis OX-K (-)
B. P. vulgaris OX-19 (-), P. vulgaris OX-2 (+), P. mirabilis OX-K (-)
C. P. vulgaris OX-19 (-), P. vulgaris OX-2 (+), P. mirabilis OX-K (+)
D. P. vulgaris OX-19 (-), P. vulgaris OX-2 (-), P. mirabilis OX-K (+)
50. The least immunogenic transplant tissue:
A. Bone marrow B. Cornea C. Heart D. Skin
51. According to AABB standards, 75% of all platelets, pheresis units shall contain how many platelets per uL?
A. 5.5 x 1010 C. 3.0 x 1011
10
B. 6.5 x 10 D. 5.5 x 1011
52. Which of the following is proper procedure for preparation of platelets from whole blood?
A. Light spin followed by a hard spin C. Two light spins
B. Light spin flowed by two hard spins D. Hard spin followed by a light spin
53. The purpose of low-dose irradiation of blood components is to:
A. Prevent post transfusion purpura C. Sterilize components
B. Prevent GVHD D. Prevent noncardiogenic pulmonary edema
54. The most effective component to treat a patient with fibrinogen deficiency is:
A. Fresh frozen plasma C. Fresh whole blood
B. Platelets D. Cryoprecipitated AHF
55. Which of the following blood components is the best source of factor IX?
A. Platelets C. Fresh frozen plasma
B. Cryoprecipitated AHF D. Prothrombin complex
56. Hives and itching are symptoms of which of the following transfusion reactions?
A. Febrile C. Circulatory overload
B. Allergic D. Anaphylactic
57. Cold agglutinin syndrome is best associated with which of the following blood groups?
A. Duffy C. Ii
B. P D. Rh
58. Cryoprecipitated AHF, if maintained in the frozen state at –18 oC or below, has a shelf-life of:
A. 42 days C. 12 months
B. 6 months D. 36 months
59. A blood component used in treatment of hemophilia A?
A. Factor VIII concentrate C. Platelets
B. Fresh frozen plasma D. Whole blood
60. According to AABB standards, platelets prepared from whole blood shall have at least:
A. 5.5 x 1010 platelets per unit in at least 75% of the units tested
B. 6.5 x 1010 platelets per unit in 75% of the units tested
C. 7.5 x 1010 platelets per unit in 100% of the units tested
D. 8.5 x 1010 platelets per unit in 95% of the units tested
61. Which of the following transfusion reactions occurs after infusion of only a few milliliters of blood and gives no
history of fever?
A. Febrile C. Anaphylactic
B. Circulatory overload D. Hemolytic
62. According to AABB standards, fresh frozen plasma must be infused within what period of time following
thawing?
A. 24 hours C. 48 hours
B. 36 hours D. 72 hours
63. The most frequent transfusion-associated disease complication of blood transfusion is:
A. Cytomegalovirus C. Hepatitis
B. Syphilis D. AIDS
64. Which of the following antigens gives enhanced reactions with its corresponding antibody following treatment of
the red cells with proteolytic enzymes?
A. Fya C. S
B. E D. M
65. Which of the following would be the component of choice for treatment of von Willebrand’s disease?
A. Platelets C. Cryoprecipitated AHF
B. Factor IX concentrate D. Fresh frozen plasma
4

66. If the seal is entered on a unit of whole blood stored at 1-6 oC, what is the maximum allowable storage period,
in hours?
A. 6 C. 48
B. 24 D. 72
67. Which of the following red cell antigens are found on glycophorin A?
A. M, N C. S, s
B. Lea, Leb D. P, P1, Pk
68. Which of the following is a characteristic of anti-I?
A. Often associated with HDN C. Reacts best at 37 oC
B. Frequently a cold agglutinin D. Is usually IgG
69. Which of the following tests is most commonly used to demonstrate antibodies that have become attached to a
patient’s red cells in vivo?
A. Direct antiglobulin C. Indirect antiglobulin
B. Complement fixation D. Immunofluorescence
70. A previously named HLA that is not uncommonly detected on erythrocytes is:
A. Dia B. Sda C. Bga D. Coa
71. A low-incidence antigen that serves as a useful anthropologic marker for Mongolian ancestry:
A. Xga B. Doa C. Dia D. Yta
72. Antigen is found on the petite arm of the X chromosome and is noted with higher frequency in females than in
males.
A. Xga B. Doa C. Dia D. Yta
73. Antibodies formerly classified as HTLAs:
A. Anti-Coa/Cob B. Anti-Ch/Rg C. Anti-CROM D. Anti-Dia/Dib
74. The minimum hemoglobin concentration in g/dL in a fingerstick from a male blood donor is:
A. 12.0 C. 13.5
B. 12.5 D. 15.0
75. To qualify as a donor for autologous transfusion, a patient’s hemoglobin should be at least:
A. 8 g/dL C. 13 g/dL
B. 11 g/dL D. 15 g/dL
76. Hydroxyethyl starch (HES) is a rouleaux-promoting agent used to:
A. Increase the harvest of granulocytes in leukapheresis
B. Treat patients following hemolytic transfusion reaction
C. Resolve ABO typing discrepancies
D. Stabilize the pH of stored platelets
77. Leukocyte-poor red blood cells would most likely be indicated for patients with a history of:
A. Febrile transfusion reaction C. Hemophilia A
B. Iron deficiency anemia D. Von Willebrand’s disease
78. Proteolytic enzyme treatment of red cells usually destroys which antigen?
A. Jka C. Fya
B. E D. k
79. Glycophorin B is associated with the antigenic activity of:
A. MN C. Wra Wrb
B. Ss D. Lua Lub
80. Which of the following is the proper storage temperature requirement for granulocytes?
A. 1 to 6 oC C. Room temperature with constant agitation
B. 10 to 18 oC D. Room temperature without agitation
81. Which of the following best reflects the discrepancy seen when a person’s red cells demonstrated the
acquired-B phenotype?
Forward Grouping Reverse Grouping
A. B O
B. AB A
C. O B
D. B AB
82. Blood group with the greatest amount of H substance:
A. A C. O
B. B D. AB
83. The process of separation of antibody from its antigen is known as:
A. Diffusion C. Lyophilization
B. Absorption D. Elution
84. Following compatibility testing, the patient’s blood sample and the donor red cells must be retained for:
A. 7 days after crossmatching C. 10 days after crossmatching
B. 7 days after transfusion D. 10 days after transfusion
85. Hemolytic transfusion reactions result in all the following laboratory finding except:
A. Hemoglobinuria C. Hemoglobinemia
B. Haptoglobinemia D. Bilirubinemia
86. The required tests that must be performed by the hospital blood bank before issuing autologous units of blood
for transfusion include:
A. Recipient antibody screening C. HBsAg on the patient
B. ABO and Rh on the unit D. Antibody to HIV1 on the unit
5

87. An iron chelating agent which is important in lowering the body iron stores of patients with thalassemia:
A. Deferroxamine C. Desmopressin
B. Steroids D. Aspirin
88. Rh antibodies are primarily of which immunoglobulin class?
A. IgM B. IgG C. IgA D. IgE
89. Testing for weakened expression of the D antigen is done by:
A. Enzyme techniques, in conjunction with an IgM anti-D
B. Performance of an IAT using an IgG anti-D
C. Increasing the time for the incubation of the saline phase
D. Don't need to do it in a blood bank
90. The immunogenicity of the common Rh antigens may be described from greatest to least:
A. D>C>E>c>e C. c>D>C>E>e
B. D>c>E>e>C D. D>c>E>C>e
91. Cold autoagglutinins often occur in:
A. Acute inflammatory reactions C. Rheumatoid arthritis
B. Lupus erythematosus D. Primary atypical pneumonia
92. The autoagglutinin formed in Mycoplasma pneumoniae infection:
A. Anti-P B. Anti-H C. Anti-I D. Anti-i
93. In an emergency transfusion where typing and cross-matching cannot be done, it is the SAFEST to give:
A. “O” red cells C. “AB” plasma
B. “O” whole blood D. “O” plasma
94. Individuals who do not have the Rh(D) factor are classified as:
A. Rh null B. Du C. Rh (-) D. Rh (+)
95. Which of the following phenotypes show natural resistance against P. vivax infection?
A. Fy (a+b+) B. Fy (a-b-) C. Fy (a+b-) D. Fy (a-b+)
96. What substance ca n neutralize anti-Lea?
A. Urine B. Saliva C. Human milkD. Hydatid cyst fluid
97. Storage condition for frozen red cells:
A. –18oC for 1 year C. 20-24oC for 3-5 years
o
B. 1-6 C for 24 hours D. -65oC for 10 years
98. The major crossmatch involves testing:
A. Donor’s serum vs patient’s cells C. Patient’s serum vs patient’s cells
B. Patient’s serum vs donor’s cells D. Donor’s serum vs donor’s cells
99. Type A recipient is crossmatched with type O blood, what incompatibility is expected?
A. Major crossmatch C. Both of these
B. Minor crossmatch D. Neither of these
100. Specimen needed in the investigation of hemolytic transfusion reaction, EXCEPT:
A. New blood sample from recipient C. Patient’s urine
B. New blood sample from donor D. Remaining blood in the unit

END OF THE EXAM


PRE-BOARD EXAMINATION IN IMMUNOLOGY-SEROLOGY AND BLOOD BANKING (PART3)

1. Macrophages have specific names according to their tissue location. Macrophages in the liver are:
A. Alveolar macrophages C. Microglial cells
B. Kupffer cells D. Histiocytes
2. In which area of the lymph node are T cells mainly found:
A. Germinal center C. Paracortex
B. Primary follicles D. Sinusoids
3. Lymphocyte mitogen that acts on both B cell and T cell
A. E. coli endotoxin C. Concanavalin A (ConA)
B. Lipopolysaccharides D. Pokeweed mitogen (PWM)
4. Antigen given intravenously are mainly trapped in the:
A. Lung C. Spleen
B. Lymph node D. Tonsils
5. Nitroblue tetrazolium test is of value in the diagnosis of:
A. Bruton’s agammaglobulinemia C. Kahler’s disease
B. Franklin’s disease D. Chronic granulomatous disease
6. Variations in variable regions that give individual antibody molecules specificity:
A. Isotype C. Allotype
B. Idiotype D. All of these
7. The hinge region of an immunoglobulin is flexible because it contains a large amount of which amino acid?
A. Serine C. Threonine
B. Cystine D. Proline
8. Secretory IgA contains:
1. Four light chains 3. Secretory component
2. Four heavy chains 4. One J chain
A. 1 and 3 C. 1, 2 and 3
B. 2 and 4 D. 1, 2, 3 and 4
9. Complement component C3 is cleaved by:
A. C3b C. C3bBb
B. Factor B D. Factor D
10. Which of the following statements does not apply to IgG?
A. Appears early in the immune response
B. Can fix complement
C. Crosses the human placenta
D. Opsonizes bacteria
11. The key cells involved in mediating delayed hypersensitivity are:
A. Neutrophils C. T cells
B. Mast cells D. B cells
12. RAST measures:
A. Antigen concentration C. IgE antibodies
B. IgM antibodies D. IgG antibodies
13. Helper T cells recognize antigen on antigen-presenting cells as antigen:
A. With MHC class I product C. With both class I and class II products
B. With MHC class II product D. With complement
14. MHC class II molecules are found on:
A. Virtually all cells in the body
B. B cells, dendritic cells and macrophages
C. Virtually all nucleated cells in the body
D. Only on virally infected cells
15. At least one of the functions of CD8 on the surface of lymphocytes interacting with another cell is to:
A. Bind antigen C. Bind to MHC class I molecules
B. Bind surface immunoglobulin D. Bind to MHC class II molecules
16. Transfer of tissues and organs between genetically identical individuals such as identical twins:
A. Autograft C. Xenograft
B. Syngraft D. Allograft
17. Fetus can be considered as an:
A. Autograft C. Isograft
B. Allograft D. Xenograft
18. Transplanted cells are mainly destroyed by:
A. B cells C. Macrophages
B. T cells D. Neutrophils
19. Which term applies to the most accelerated type of graft rejection?
A. Acute rejection C. Hyperacute rejection
B. Chronic rejection D. Accelerated rejection
20. Carcinoembryonic antigen is characteristically secreted by tumors of the:
A. Kidney C. Lungs
B. Bones D. Gastrointestinal tract
21. LAK (lymphokine-activated killer) cells are induced by:
A. Interleukin 1 C. Tumor necrosis factor - 
B. Interleukin 2 D. Interferon - 
2

22. A hybridoma is a cell formed by fusion of a:


A. Plasma cell with a plasma cell of another species
B. T cell with a myeloma cell
C. Plasma cell with a myeloma cell
D. T cell with a B cell
23. The prozone phenomenon can:
1. Result from excessive antibody concentration
2. Result in false positive reaction
3. Result in false negative reaction
4. Be overcome by serially diluting the antibody containing serum
A. 1 and 3 B. 2 and 4 C. 1, 2 and 4 D. 1, 3 and 4
24. Reverse passive agglutination:
1. Antigen is attached to carrier particle
2. Antibody is attached to carrier particle
3. Agglutination occurs if patient antibody is present
4. Agglutination occurs if patient antigen is present
A. 1 and 3 B. 2 and 4 C. 1, 2, and 3 D. 1, 2, 3 and 4
25. ANA pattern associated with SLE in the active stage of the disease:
A. Diffused or homogenous pattern C. Peripheral pattern
B. Nucleolar pattern D. Speckled pattern
26. Jenner’s work with cowpox, which provided immunity against smallpox, demonstrates which phenomenon?
A. Natural Immunity B. Cross-immunity C. Attenuation of vaccines D. Reactivity of haptens
27. In searching for a cure for TB, Koch was first to observe which phenomenon?
A. Bacterial aggln. B. Precipitation C. Phagocytosis D. Delayed hypersensitivity
28. Chronic granulomatous disease represents a defect of:
A. Oxidative metabolism C. Diapedesis
B. Abnormal granulation of neutrophils D. Chemotaxis
29. Which one of the following cells destroys tumor cells using ADCC as a recognition mode, and perforin as an
effector molecule?
A. B cells B. CD4+ cells B. CD8+ cells D. NK cells
30. Measurement of CRP levels can be used for all of the following except:
A. Monitoring drug therapy with anti-inflammatory agents
B. Tracking the normal progress of surgery
C. Diagnosis of specific bacterial infection
D. Determining the active phases of rheumatoid arthritis
31. Which one of the following cells recognizes a cell surface complex consisting of antigenic peptide complexed
with an MHC protein?
A. Phagocytes B. Eosinophils C. T cells D. B cells
32. In the complement fixation procedure, a negative result is manifested by:
A. Antigen-binding C. Lysis of sheep red blood cells
B. Lysis of guinea pig cells D. Agglutination of sheep red blood cells
33. The Mantoux test is an example of:
A. Type I hypersensitivity C. Type III hypersensitivity
B. Type II hypersensitivity D. Type IV hypersensitivity
34. Anaphylaxis as a result of bee sting is an example of:
A. Type I hypersensitivity C. Type III hypersensitivity
B. Type II hypersensitivity D. Type IV hypersensitivity
35. What immune elements are involved in a reaction to poison ivy?
A. IgE antibodies B. T cells and macrophages C. NK cells and IgG D. B cells and IgM
36. Rheumatoid factor is directed against which of the following molecules?
A. Whole IgM molecules C. Fab region of the IgG molecule
B. Fc portion of the IgG molecule D. Fc portion of the IgM molecule
37. Destruction of the myelin sheath of axons caused by the presence of antibody is characteristic of which
disease?
A. Multiple sclerosis C. Graves’ disease
B. Myasthenia gravis D. Goodpasture’s syndrome
38. It is suggestive of Goodpasture’s disease:
A. Acetylcholine receptor-blocking antibodies C. Anti-DNA antibodies
B. Anti-cardiolipin antibodies D. Anti-glomerular basement membrane antibodies
39. It is strongly suggestive, in a high titer, of primary biliary cirrhosis:
A. Anti-myelin antibody C. Anti-centromere antibody
B. Anti-intrinsic factor antibody D. Anti-mitochondrial antintibody
40. A defect in C1INH results in which one of the following disorders?
A. Bruton’s agammaglobulinemia C. Chronic granulomatous disease
B. Selective IgA deficiency D. Hereditary angioneurotic edema
41. Individuals who are at risk for ankylosing spondylitis have inherited which one of the following alleles?
A. HLA-A3 B. HLA-B8 C. HLA-B27 D. HLA-B7
42. Individuals who are at risk for rheumatoid arthritis have inherited which one of the following alleles?
A. HLA-A3 B. HLA-B27 C. HLA-B7 D. HLA-DR4
43. A kidney transplantation between one identical twin to another is an example of:
A. An allograft B. An autograft C. A heterograft D. A syngeneic graft
3

44. CA-15.3 is used conditionally in the monitoring of:


A. Pancreatic adenocarcinoma C. Breast adenocarcinoma
B. Colonic adenocarcinoma D. Hairy cell leukemia
45. In infected blood, Treponema pallidum does not appear to survive at 4oC for more than:
A. 1 day B. 2 days C. 3 days D. 4 days
46. The serologic marker during the “window period” of hepatitis B is:
A. Anti-HBs B. Anti-HBc C. Anti-HBe D. HBsAg
47. Antibodies to which of the following retroviral antigens are usually the first to be detected in HIV infection?
A. gp120 B. gp160 C. gp41 D. p24
48. Which of the following combinations of bands would represent a positive Western blot for HIV antibody?
A. p24 and p55 B. p24 and p31 C. gp41 and gp120 D. p31 and p55
49. Which of the following identifies the pattern of antibody cross-reactivity that is generated during infection with R.
rickettsii?
A. P. vulgaris OX-19 (+), P. vulgaris OX-2 (+), P. mirabilis OX-K (-)
B. P. vulgaris OX-19 (-), P. vulgaris OX-2 (+), P. mirabilis OX-K (-)
C. P. vulgaris OX-19 (-), P. vulgaris OX-2 (+), P. mirabilis OX-K (+)
D. P. vulgaris OX-19 (-), P. vulgaris OX-2 (-), P. mirabilis OX-K (+)
50. The least immunogenic transplant tissue:
A. Bone marrow B. Cornea C. Heart D. Skin

BLOOD BANKING

51. According to AABB standards, 75% of all platelets, pheresis units shall contain how many platelets per uL?
A. 5.5 x 1010 C. 3.0 x 1011
10
B. 6.5 x 10 D. 5.5 x 1011
52. Which of the following is proper procedure for preparation of platelets from whole blood?
A. Light spin followed by a hard spin C. Two light spins
B. Light spin flowed by two hard spins D. Hard spin followed by a light spin
53. The purpose of low-dose irradiation of blood components is to:
A. Prevent post transfusion purpura C. Sterilize components
B. Prevent GVHD D. Prevent noncardiogenic pulmonary edema
54. The most effective component to treat a patient with fibrinogen deficiency is:
A. Fresh frozen plasma C. Fresh whole blood
B. Platelets D. Cryoprecipitated AHF
55. Which of the following blood components is the best source of factor IX?
A. Platelets C. Fresh frozen plasma
B. Cryoprecipitated AHF D. Prothrombin complex
56. Hives and itching are symptoms of which of the following transfusion reactions?
A. Febrile C. Circulatory overload
B. Allergic D. Anaphylactic
57. Cold agglutinin syndrome is best associated with which of the following blood groups?
A. Duffy C. Ii
B. P D. Rh
58. Cryoprecipitated AHF, if maintained in the frozen state at –18 oC or below, has a shelf-life of:
A. 42 days C. 12 months
B. 6 months D. 36 months
59. A blood component used in treatment of hemophilia A?
A. Factor VIII concentrate C. Platelets
B. Fresh frozen plasma D. Whole blood
60. According to AABB standards, platelets prepared from whole blood shall have at least:
A. 5.5 x 1010 platelets per unit in at least 75% of the units tested
B. 6.5 x 1010 platelets per unit in 75% of the units tested
C. 7.5 x 1010 platelets per unit in 100% of the units tested
D. 8.5 x 1010 platelets per unit in 95% of the units tested
61. Which of the following transfusion reactions occurs after infusion of only a few milliliters of blood and gives no
history of fever?
A. Febrile C. Anaphylactic
B. Circulatory overload D. Hemolytic
62. According to AABB standards, fresh frozen plasma must be infused within what period of time following
thawing?
A. 24 hours C. 48 hours
B. 36 hours D. 72 hours
63. The most frequent transfusion-associated disease complication of blood transfusion is:
A. Cytomegalovirus C. Hepatitis
B. Syphilis D. AIDS
64. Which of the following antigens gives enhanced reactions with its corresponding antibody following treatment of
the red cells with proteolytic enzymes?
A. Fya C. S
B. E D. M
4

65. Which of the following would be the component of choice for treatment of von Willebrand’s disease?
A. Platelets C. Cryoprecipitated AHF
B. Factor IX concentrate D. Fresh frozen plasma
66. If the seal is entered on a unit of whole blood stored at 1-6 oC, what is the maximum allowable storage period,
in hours?
A. 6 C. 48
B. 24 D. 72
67. Which of the following red cell antigens are found on glycophorin A?
A. M, N C. S, s
B. Lea, Leb D. P, P1, Pk
68. Which of the following is a characteristic of anti-I?
A. Often associated with HDN C. Reacts best at 37 oC
B. Frequently a cold agglutinin D. Is usually IgG
69. Which of the following tests is most commonly used to demonstrate antibodies that have become attached to a
patient’s red cells in vivo?
A. Direct antiglobulin C. Indirect antiglobulin
B. Complement fixation D. Immunofluorescence
70. A previously named HLA that is not uncommonly detected on erythrocytes is:
A. Dia B. Sda C. Bga D. Coa
71. A low-incidence antigen that serves as a useful anthropologic marker for Mongolian ancestry:
A. Xga B. Doa C. Dia D. Yta
72. Antigen is found on the petite arm of the X chromosome and is noted with higher frequency in females than in
males.
A. Xga B. Doa C. Dia D. Yta
73. Antibodies formerly classified as HTLAs:
A. Anti-Coa/Cob B. Anti-Ch/Rg C. Anti-CROM D. Anti-Dia/Dib
74. The minimum hemoglobin concentration in g/dL in a fingerstick from a male blood donor is:
A. 12.0 C. 13.5
B. 12.5 D. 15.0
75. To qualify as a donor for autologous transfusion, a patient’s hemoglobin should be at least:
A. 8 g/dL C. 13 g/dL
B. 11 g/dL D. 15 g/dL
76. Hydroxyethyl starch (HES) is a rouleaux-promoting agent used to:
A. Increase the harvest of granulocytes in leukapheresis
B. Treat patients following hemolytic transfusion reaction
C. Resolve ABO typing discrepancies
D. Stabilize the pH of stored platelets
77. Leukocyte-poor red blood cells would most likely be indicated for patients with a history of:
A. Febrile transfusion reaction C. Hemophilia A
B. Iron deficiency anemia D. Von Willebrand’s disease
78. Proteolytic enzyme treatment of red cells usually destroys which antigen?
A. Jka C. Fya
B. E D. k
79. Glycophorin B is associated with the antigenic activity of:
A. MN C. Wra Wrb
B. Ss D. Lua Lub
80. Which of the following is the proper storage temperature requirement for granulocytes?
A. 1 to 6 oC C. Room temperature with constant agitation
B. 10 to 18 oC D. Room temperature without agitation
81. Which of the following best reflects the discrepancy seen when a person’s red cells demonstrated the acquired-
B phenotype?
Forward Grouping Reverse Grouping
A. B O
B. AB A
C. O B
D. B AB
82. Blood group with the greatest amount of H substance:
A. A C. O
B. B D. AB
83. The process of separation of antibody from its antigen is known as:
A. Diffusion C. Lyophilization
B. Absorption D. Elution
84. Following compatibility testing, the patient’s blood sample and the donor red cells must be retained for:
A. 7 days after crossmatching C. 10 days after crossmatching
B. 7 days after transfusion D. 10 days after transfusion
85. Hemolytic transfusion reactions result in all the following laboratory finding except:
A. Hemoglobinuria C. Hemoglobinemia
B. Haptoglobinemia D. Bilirubinemia
5

86. The required tests that must be performed by the hospital blood bank before issuing autologous units of blood
for transfusion include:
A. Recipient antibody screening C. HBsAg on the patient
B. ABO and Rh on the unit D. Antibody to HIV1 on the unit
87. An iron chelating agent which is important in lowering the body iron stores of patients with thalassemia:
A. Deferroxamine C. Desmopressin
B. Steroids D. Aspirin
88. Rh antibodies are primarily of which immunoglobulin class?
A. IgM B. IgG C. IgA D. IgE
89. Testing for weakened expression of the D antigen is done by:
A. Enzyme techniques, in conjunction with an IgM anti-D
B. Performance of an IAT using an IgG anti-D
C. Increasing the time for the incubation of the saline phase
D. Don't need to do it in a blood bank
90. The immunogenicity of the common Rh antigens may be described from greatest to least:
A. D>C>E>c>e C. c>D>C>E>e
B. D>c>E>e>C D. D>c>E>C>e
91. Cold autoagglutinins often occur in:
A. Acute inflammatory reactions C. Rheumatoid arthritis
B. Lupus erythematosus D. Primary atypical pneumonia
92. The autoagglutinin formed in Mycoplasma pneumoniae infection:
A. Anti-P B. Anti-H C. Anti-I D. Anti-i
93. In an emergency transfusion where typing and cross-matching cannot be done, it is the SAFEST to give:
A. “O” red cells C. “AB” plasma
B. “O” whole blood D. “O” plasma
94. Individuals who do not have the Rh(D) factor are classified as:
A. Rh null B. Du C. Rh (-) D. Rh (+)
95. Which of the following phenotypes show natural resistance against P. vivax infection?
A. Fy (a+b+) B. Fy (a-b-) C. Fy (a+b-) D. Fy (a-b+)
96. What substance ca n neutralize anti-Lea?
A. Urine B. Saliva C. Human milkD. Hydatid cyst fluid
97. Storage condition for frozen red cells:
A. –18oC for 1 year C. 20-24oC for 3-5 years
o
B. 1-6 C for 24 hours D. -65oC for 10 years
98. The major crossmatch involves testing:
A. Donor’s serum vs patient’s cells C. Patient’s serum vs patient’s cells
B. Patient’s serum vs donor’s cells D. Donor’s serum vs donor’s cells
99. Type A recipient is crossmatched with type O blood, what incompatibility is expected?
A. Major crossmatch C. Both of these
B. Minor crossmatch D. Neither of these
100. Specimen needed in the investigation of hemolytic transfusion reaction, EXCEPT:
A. New blood sample from recipient C. Patient’s urine
B. New blood sample from donor D. Remaining blood in the unit

END OF THE EXAM


CLS2-BLOOD BANKING & SEROLOGY the first thing the blood bank
technologist should do?
Serologic principle which refers to the a. Place cold compresses on the
random movement of antigen or donor’s forehead.
antibody to form a complex in semisolid b. Remove the tourniquet and
medium is withdraw the needle
a. Agglutination c. Raise the donor’s legs above the
b. Complement fixation level of the head
c. Gel diffusion d. Loosen tight clothing and secure
d. Immunoelectrophoresis airway

Cryoprecipitated antihemophilic factor Carcinoembryonic antigens (CEA) is a


contains the following: test associated with a type of malignant
(1) - 80 units of AHF tumor:
(2) - 150 mg of fibrinogen a. carcinoma of the breast
(3) - Factor IX b. Carcinoma of the colon
(4) - Factor XIII c. carcinoma of the liver
(5) - von Willebrand factor d. carcinoma of the lung
a. 1 and 2 only
b. 1, 2, 4 and 5 only Which of the following statements does
c. 1, 2, 3 and 4 only NOT describe coagglutination tests?
d. 1, 2, 3 and 5 only a. These tests use bacteria as the inert
particles to which antibody is
How often can blood be drawn for attached.
autologous transfusion? b. Staphylococcus aureus is most
a. Every 3 days, but not within 3 hours commonly used. (for N.
before the surgery meningitidis)
b. Every 3 days, but not within 3 c. Protein A naturally adsorbs the Fc
days before the surgery portion of the antibody molecules.
c. Every 3 weeks, but not within 3 days d. Since these tests use bacteria,
before the surgery reactions are more easily read
d. Every 3 weeks, but not within 3 compared to tests which use latex
hours before the surgery particles

Which of the following is INCORRECT Pooled cryoprecipitate is indicated in


about RBC storage lesion? patients with: (1) - Hypofibrinogenemia
a. Decreased glucose (2) - Factor XIII deficiency (3) - Factor VIII
b. Decreased ATP deficiency (4) - TTP
c. Decreased lactic acid a. 1 and 2 only
d. Increased plasma hemoglobin b. 1, 2 and 3 only
c. 1, 2 and 4 only
During blood donation, the donor d. 2 and 3 only
suffered nausea and vomiting. What is
A patient with 28% hematocrit is Paul-Bunnell test and the Davidsohn's
transfused with 2 units of packed red Differential absorption tests are tests for:
blood cells. What is his hematocrit after a. syphilis
transfusion? b. Infectious mononucleosis
a. 28% c. lupus erythematosus
b. 30% d. rheumatoid arthritis
c. 31%
d. 34% It is an acute non-immune transfusion
reaction presenting with body
In a quantitative serological test, the temperature usually 2°C or more above
results shown below were obtained. How normal and rigors that can be
should this result be reported (+) means accompanied by hypotension.
antibody detected; (0) means no a. Febrile Nonhemoytic Transfusion
antibody detected? Reaction
Serum dilution and result = 1:10 (+++); b. Allergic Transfusion Reactions
1:20 (++); 1:40 ( - ); 1:80 (+); 1:160 (+); c. Transfusion-Associated Sepsis
1:320 (+) d. Delayed Hemolytic Transfusion
a. Report “negative” for antibody Reactions
b. Repeat the test because this pattern
is impossible Which of the following autoantibodies is
c. Perform testing on additional diagnostic for SLE?
dilutions until an end point is a. Anti-Sm
reached b. Anti-Jo-1
d. Report “positive” for antibody c. Anti-Scl-70
d. Anti-ss-DNA
Which of the following immunoglobulins
are found on the surface of mature B Which of the following lectin is used to
cells? agglutinate B cells?
(I) IgG a. Dolichos biflorus
(II) IgM b. Bandeiraea simplicifolia
(III) IgA c. Ulex europaeus
(IV) IgD d. None of the above
a. I and III only
b. II and IV only A healthy medtech student goes to
c. I and II only donate blood while in a clinical
d. III and IV only internship program at a community
blood bank. He meets all the donor
Application of DAT includes which criteria but reports that he received
among the following? tetanus booster a week ago, after
a. HDN stepping on a rusty nail in his bare feet.
b. Drug induced hemolytic anemia What decision can be made from this
c. RBC phenotype determination situation?
d. A and B a. Defer the donor for 2 weeks
b. Defer the donor for 1 month
c. Defer the donor for 12 months 2. Bacterial H antigen agglutination
d. Accept the donor reaction is more compact than O antigen
agglutination
Aside from the ABO and Rh antibodies, a. Both statements are true
what is the MOST common antibody b. Both statements are false
seen in the blood bank? c. Only statement 1 is true
a. Anti-MN d. Only statement 2 is true
b. Anti-K
c. Anti-Fy With reference to cold agglutinins, which
d. Anti-Jk one of the following is CORRECT?
a. agglutinate type O human
The following serological dilutions were erythrocytes at 37°C
made: 0.9 ml of diluent was pipetted into b. Cold agglutinins are IgM
each of 4 tubes. In the first tube, 0.1 ml antibodies
of serum was added. This was mixed and c. Are reactive with the P antigen on
0.1 ml was transferred into tube #2. Upon red blood cells
addition of a reagent which has a limit d. Are also found in response to
sensitivity of 2 U/L, tube 2 gave a Herpes Simplex Virus
positive result. What will be the antibody
titer of your patient? All of the following would lead to
a. 100 diagnosis of acute immune hemolytic
b. 200 transfusion reaction, EXCEPT
c. 300 a. DAT positive
d. 400 b. Decreased hemoglobin
c. Decreased LDH
1 - Rh HDN is the most common type of d. Decreased haptogobin
HDN.
2 - Antibodies to ABO cause the most ABO typing of a certain patient yields the
severe form of HDN. following reactions:
a. First statement is true; second A1 cells (2+) ; B cell (4+) ; Anti-A (4+) ;
statement is false Antin-B ( 0 ); Anti-A1 ( 0 ); Anti-AB (4+).
b. First statement is false; second The reaction may be because
statement is true a. patient is AB with anti-A1
c. Both statements are true b. patient is A1 with acquired AB
d. Both statements are false c. patient is A2 with anti-A1
d. patient has increased concentration
In which organ do T-cells mature? of proteins in serum
a. spleen
b. lymph nodes Which of the following is used in the
c. bone marrow immunocytochemical tests for dengue?
d. Thymus a. Dengue antigen + peroxidase
b. Monoclonal antibody + phosphatase
1. The H and O antigens of Salmonella c. Dengue antigen + monoclonal
spp. are heat labile. antibody
d. Peroxidase + phosphatase II - Oxygenation saturation of <90% on
room air
Diapedesis is defined as III - Bilateral infiltrates on frontal chest
a. The movement of phagocytes radiography
towards a chemical stimulus IV - No evidence of left atrial hypertension
(Chemotaxis) a. II and III only
b. The rolling of neutrophils along the b. I and IV only
vascular endothelium c. I, II and III only
c. Movement of cells from d. II, III and IV only
circulating pool to tissue pool by e. Decreased LDH
squeezing through the wall of a f. Decreased haptogobin
blood vessel
d. Enhancement of phagocytosis by
coating of foreign particles with All RBC additive solutions contain the
serum proteins following, EXCEPT
a. saline
Which of the following are markers for b. adenine
NK cells? c. glucose
I) CD16 II) CD34 III) CD 56 IV) CD71a. d. phosphate
a. I and II only
b. II and IV only Three victims of a car accident arrived at
c. I and III only the emergency room at the same time.
d. I, II and III only Patients include an 18-year old female, a
70 year old female and a 40 year old
Prozone maybe seen male. They all require immediate blood
a. When there is a high transfusion. Findings show that they are
concentration of antibody all O negative. Only one unit of O
b. when there is a great antigen excess negative is available. Using these
c. when either antigen or antibody is assessments, the patient who should
present in excess receive the blood would be ____.
d. None of the above a. the 40 year-old man
b. the 18 year old female
Fresh frozen plasma is made from the c. the 70-year old woman
whole blood donations. It must be d. All three patients can be given O
harvested within ___ hours of whole positive blood since the situation is
blood collection ane mergency
a. 4 to 6
b. 6 to 8 (frozen within 8 hours) Which of the following statements about
c. 8 to 10 hyperacute graft rejection is
d. 8 to 24 INCORRECT?
a. Tissue damage occurs within
Criteria for the diagnosis of TRALI minutes.
include: b. It is predominantly humoral in
I - Onset is within 4 hours of transfusion nature.
c. Previous sensitization to donor b. Anti-streptolysin O
antigen is required. c. Streptozyme test
d. It is caused by preformed cytotoxic d. None of the above
antibodies to donor antigens.
All of the following are destroyed by
Which of the following stages of syphilis enzymes, EXCEPT
is characterized by condylomata lata? a. Ch/Rg
a. Primary b. JMH
b. Secondary c. Lewis
c. Latent d. A and B
d. Tertiary
All screen and panel cells are negative.
Washing of cells with saline buffered to a There is incompatibility in the
pH ___ before the addition of AHG Antiglobulin crossmatch. No ABO
reagent should be done at least ___ incompatibility is found and the donor
times to ___. has a negative DAT. This is most likely
a. 7.0 - 7.5 / two / removed unwanted due to
antibodies a. Presence of multiple antibody
b. 7.2 - 7.4 / three / remove unbound b. Presence of high frequency antigen
immunoglobulins c. Presence of low frequency
c. 7.4-7.8 / two / remove unbound antigen
immunoglobulin d. Presence of warm reacting
d. 7.5 - 8.5 / five / remove excess autoantibodies
antibodies
Peter undergone platelet apheresis
Which of the following is a Non- donation yesterday. He wants to donate
penetrating cryoprotective agent(s): whole blood again today. What should
a. Glycerol the blood bank technologist do?
b. HES a. Defer him for 2 weeks
c. DMSO b. Defer him for 4 weeks
d. B and C c. Defer him for 2 days
d. Defer him for 12 weeks
Comprises the membrane attack
complex. Double negative thymocytes have which
a. C5b6789 of the following markers?
b. C5a6789 I. CD 2
c. C4b2a II. CD 4
d. C4b2a3b III. CD 5
IV. CD 7
Which of the following can V. CD 8
presumptively diagnose Streptococcal a. I, II, III and IV
rheumatic fever, BUT not Acute b. I and V
glomerulonephritis? c. I, III, IV
a. Anti- DNAse B titer d. I and III
Which of the following are correctly It is rare in most populations but is
matched? polymorphic in people of Mongolian
(1). Oudin Test: Single Diffusion-Double ancestry
Dimension (must be single diffusion) a. Dia
(2). Mancini Test: Single Diffusion-Double b. Yt
Dimension c. Xg
(3). Laurell Method: Single Reactant Moving d. SC
in One Dimension
(4). Ressler’s Method: Single Reactant The following are included in the CREST
Moving in One Dimension syndrome, EXCEPT
a. 1 and 2 a. Raynaud’s Phenomenon
b. 1 and 4 b. Esophageal Dysmotility
c. 2 and 3 c. Scleroderma
d. 3 and 4 d. Telangiectasia

OX K refers to which of the following? Which of the following HIV enzymes is


a. Antigens of Rickettsia prowazeki involved in viral replication?
b. Antibodies to Rickettsia prowazeki a. Integrase
c. Strains of Proteus mirabilis b. Nuclease
d. Strains of Proteus vulgaris c. Protease
d. Reverse transcriptase
Fresh frozen plasma is thawed at
temperatures between 30°C and 37°C . A female donor had sexual contact with
After thawing, the product must be a male who had sexual contact with
stored at ____ for ____. another male in exchange for money or
a. 20 to 24°C; 24 hours drugs. How long the woman should be
b. 1-6°C; 24 hours deferred?
c. 20 to 24°C; 5 days a. 12 months
d. 1-6°C; 5 days b. Indefinite
c. 3 years
Autoantibodies associated with Graves d. Permanent
Disease.
a. Anti-thyroid peroxidase antibodies This is the first inflammatory response.
b. Anti-microsomal antibodies a. Vasoconstriction
c. Anti-TSH receptor antibodies b. Vasodilation
d. Anti-thyroglobulin antibodies c. Chemotaxis
d. B and C
Antibody associated with Mycoplasma
pneumoniae infection CD8+ T cells are classified as
a. Anti-i a. Cytotoxic/ Suppressor
b. Anti-I b. Regulatory
c. Donath-Landsteiner antibody c. Helper
d. Forsmann antibody d. Natural Killer
d. Administer oxygen
All of the following are indications for
platelet transfusion, EXCEPT Which of the following best describes
a. Thrombocytopenia with bleeding affinity?
b. Chemotherapy for malignancy a. Initial force of attractionion
c. Disseminated intravascular b. Measure of overall stability of
coagulation antigen-antibody complexes
d. Hemophilia A c. Multivalent antigen-antibody binding
d. All of the above
Jenner's work on cowpox, which
provided immunity against small pox, 1 - Autosomal refers to traits that are
demonstrates which phenomenon? carried on the sex chromosomes.
a. Cross immunity 2 - A recessive trait is carried by either
b. Phagocytosis parent or both parents but is not
c. Natural Immunity generally seen at the phenotypic level
d. Attenuation of vaccines unless both parents carry the trait.
a. First statement is true; second
These are proteins expressed by cells of statement is false
the innate immune system to identify b. First statement is true; second
pathogen-associated molecular patterns statement is true
(PAMPs). c. First statement is false; second
a. Primitive Pattern Recognition statement is true
Receptors d. First statement is false; second
b. Pathogen Recognition Receptors statement is false
c. Pattern Recognition Receptors
d. All of the above Which of the following autoimmune
disorders is caused by unregulated
Blood for intrauterine transfusion should secretion of T3 and T4 due to stimulation
be of TSH receptors by antibodies?
a. O positive, CMV negative a. Rheumatoid arthritis
b. A negative, CMV positive b. Pernicious anemia
c. O negative, CMV positive c. Grave's disease
d. O negative, CMV negative d. Hashimoto's thyroiditis

A first time blood donor is noticed to All of the following forms of bacterial
experience rapid breathing and antigens are used for artificial
involuntary twitching of his fingers immunization in man, EXCEPT
shortly after starting phlebotomy. The a. attenuated organism
phlebotomist should: b. killed organism
a. Raise his feet above his head c. toxoid
b. Have him rebreathe air from paper d. virulent organism
bag
c. Have him inhale from an ammonia In this formula, letter “A” refers to: %
capsule RCS = “A” X 100 / “B” (“A” + “C”)
a. Initial volume a. Accepted for blood donation
b. Total volume b. Deferred because the hematocrit is
c. Volume of diluent low
d. Volume of red cell c. Deferred because the blood
pressure is high
During deglycerolization of frozen RBC, d. Deferred because his temperature is
saline washed RBCs are suspended in a high
final solution of
a. 1.6% NaCl + 2.0% dextrose In immunoelectrophoresis, when the
b. 0.9 % NaCl + 2.0% dextrose equivalence point of antigen and
c. 0.9% NaCl + 0/2% dextrose antibody is reached, which of the
d. 0.9 NaCl only following will be observed?
a. Line of intersection
Which of the following tests uses red b. Precipitin band
cells as a carrier? c. Spur
a. FTA-ABS d. Any of the above
b. MHA-TP
c. RPR Which of the following is/are
d. VDRL characteristics of a crystalloid solution
that is sued as a synthetic volume
The donor unit collected must be tested expander? (1). It is easily excreted (2). It
and processed by blood bank is quite expensive (3). Albumin is an
technologists before it can be made example of a colloid (4). Ringer’s lactate
available for transfusion. The tests solution is an example of a colloid
performed on donor blood include: a. 1, 2, 3 and 4
I. ABO/Rh; b. 1, 3 and 4 only
II. Antibody Screen ; c. 1 and 4 only
III. HBsAg ; d. 2 and 3 only
IV; Anti-HBc ;
V. Syphilis ; What is the immunodominant sugar
VI. WNV RNA responsible for H specificity?
a. III, IV, V only a. L-fucose
b. I and II only b. N-acetyl-D -galactosamine
c. I, II, III, IV and V only c. D-galactose
d. I, II, III, IV, V and VI d. GalNAc sugar

Physical examination of a possible A _________ rise in the titer is


donor: considered indicative of current
Weight: 54 kg; infection.
Temperature: 100°F ; a. Fourfold
Blood pressure: 170/90; b. twofold
Hematocrit: 40%. c. threefold
d. eightfold
The donor is
Which of the following is/are FALSE What blood component should be
about agglutination? transfused to patients who are
a. It is more sensitive than precipitation immunocompromised or who are
reaction receiving a bone marrow or stem cell
b. It is a reaction between a soluble transplant and recipients of blood from
antigen and an antibody relatives?
c. It is a reaction between an insoluble a. Irradiated RBCs
antigen with an antibody b. Leukoreduced RBCs
d. The antibody involved in strong c. Deglycerolized RBCs
reactions is IgM d. Leukoreduced Platelets

The predominant immunoglobulin class This helps as adsorption centers in


of antibodies in group O serum is ___. flocculation tests making results
a. IgM macroscopically visible:
b. IgG a. cholesterol
c. IgA b. lecithin
d. IgD c. Reagin
d. Cardiolipin
Which among the following is TRUE
regarding Bennett-Goodspeed (Bg) A patient undergoing treatment against
antigens? rheumatoid arthritis was asked by his
a. Bga is related to HLA-B8 physician to have a rheumatoid factor
b. Bgb is related to HLA-B18 test in order to determine if the
c. Bgc is related to HLA-B28 medicines are working or not. Which of
d. Reactivity of Bg antibodies are the following RF isotypes must you look
destroyed by treating RBC antigens for?
with chloroquinine or EDTA-glycine- a. IgA
HC1 b. IgE
c. IgG
Exposure to less than ___ of Rh-positive d. IgM
RBCs can stimulate antibody production
in an Rh-negative person. A prospective female blood donor came
a. 1 mL to the laboratory for screening. Upon
b. 10 mL serological testing of her blood, the test
c. 0.5 ml for HIV gave a reactive result. As part of
d. 0.1 ml the standard operating procedures, the
sample was sent to the reference
Re-inactivation of serum should be laboratory. Which of the following tests
performed at ___ °C for ___ minutes will be performed?
a. 56, 10 a. Enzyme Linked Immunosorbent
b. 56, 30 Assay
c. 62, 3 b. Immunochromatography
d. 62, 10 c. Radioallergosorbent Test
d. Western blot
Which of the following pertains to the
Which of the following chemicals in function of CRP?
anticoagulant solutions functions to a. Binds hemoglobin
produce ATP thus extending shelf-life b. Complement activation
from 21 to 35 days? c. Protease inhibitor
a. Citrate d. Removal of cholesterol
b. Monobasic sodium phosphate
c. Dextrose Which of the following is known as the
d. Adenine sheep red blood cell receptor, making it
the classical T-cell surface marker?
The strongest link found to date between a. CD2
certain HLA molecules and specific b. CD3
diseases is between HLA-B27 and c. CD4
a. multiple sclerosis d. CD8
b. myasthenia gravis
c. diabetes mellitus type 1 Platelet concentrates are pooled using
d. Ankylosing spondylitis an open system at 10:00 AM. The nurse
arrived at the blood bank to get the units
Alpha-fetoprotein (AFP) is a test at around 4:00 PM for transfusion. What
associated with a kind of malignant should the blood bank technologist do?
tumor: a. Issue the unit because the platelet
a. carcinoma of the breast unit is still viable after 6 hours of
b. carcinoma of the liver pooling
c. carcinoma of the colon b. Issue the unit because the storage
d. carcinoma of the lung of platelet is 5 days
c. Agitate the platelet pooled units
If the father is homozygous B and the before issuing
mother is heterozygous A what is the d. Do not issue the platelet units
probability of having a child of blood
type AB? Which of the following is the MOST
a. 25% common infectious complication of
b. 50% transfusion?
c. 75% a. Hemolytic transfusion reaction
d. 100% b. Transfusion Related Acute Lung
Injury
Condition in which platelet count does c. Sepsis due to bacterial
not increase as expected following contamination
transfusion of platelets d. Febrile nonhemolytic transfusion
a. Platelet refractoriness reaction
b. Dilutional coagulopathy
c. Heparin induced thrombocytopenia Absence of C1 inhibitor (C1INH) is
d. Post transfusion hepatitis associated with
a. Recurrent pyogenic infections
b. Lupus-like syndrome
c. Pneumococcal disease b. Anti-i
d. Hereditary angioedema c. Anti-P
d. Anti-PP1Pk
All of the following are correct of Lewis
antibodies, EXCEPT Antigens employed in the Widal test are
a. Antibodies are naturally occurring a. sheep's red blood cells
b. Antibodies are generally IgM b. inert particles
c. Antibodies cause hemolytic c. bacterial suspension
disease of the fetus and newborn d. human serum proteins
d. Lewis antibodies occur frequently in
the sera of pregnant women who Principle utilized by the rapid typhidot
transiently exhibit the Le(a-b-) test in diagnosing Malaria.
phenotype. a. Enzyme immunoassay
b. Immunochromatography
Pre-transfusion sample must be stored c. Latex agglutination
at ___ for at least ___ days after testing d. Microhemagglutination
is completed.
a. 1 to 6°C; 3 days Interpret the ABO Forward Grouping:
b. -20°C; 3 days Patient RBCs with anti-A = 0 ;
c. 1 to 6°C; 7 days Patient RBCs with anti-B = 0;
d. -20°C; 7 days What is the blood type of the patient?
a. A
Co-infection and superinfection occurs b. B
in which of the following types of c. AB
hepatitis infection? d. O
a. A
b. C Deficiency of which of the following
c. D complement components can lead to
d. G severe, recurrent pyogenic infections?
a. C2
This blood product is indicated in b. C3
patients who are hypovolemic, c. C4
hypoproteinemic and in clinical settings d. C4 and C2
for shock and burn patient.
a. Synthetic volume expanders
b. Fresh Frozen Plasma
c. Normal Serum Albumin
d. Plasma Protein Fraction

This antibody is associated with the


cold-reactive IgG autoantibody in
patients with paroxysmal cold
hemoglobinuria
a. Anti-I
CLS2-HEMATOLOGY
b. 2 and 4
1. Laboratory results are the following: c. 2,3,4
PT-normal; APTT-abnormal; corrected by d. 1,2,3,4
adsorbed plasma; not corrected by aged
serum. 7. What is the reference range for
a. Factor VIII deficiency Reptilase time?
b. Factor IX deficiency a. 18-22 seconds
c. Factor VII deficiency b. 65-68 seconds
d. Factor V deficiency c. 12-15 seconds
d. 30-35 seconds
2. Plasma cells evolve from which cell
line? 8. In the Coulter STKS hematology
a. Monocytic analyzer, what is the volume of sample
b. Lymphocytic aspirated?
c. CML a. 100 ul
d. CGD b. 250 ul
c. 500 ul
3. Which of the following diseases is d. 10 ul
associated with a positive TdT and PAS
stain? 9. The MCV in the Coulter STKS analyzer
a. ALL is derived from?
b. AML a. It is computed
c. CML b. Red cell histogram
d. CGD c. Platelet histogram
d. All of the above
4. Clot retraction is abnormal if:
a. Platelet count is <100,000/mm3 10. Using the Technicon H system,
b. Platelet count is >100,000/mm3 hemoglobin measurement is done by
c. Platelet count is <200,000/mm3 diluting the blood sample:
d. Platelet count is <150,000/mm3 a. 1:100 with cyanmethemoglobin reagent
b. 1:250 with cyanmethemoglobin reagent
5. The Activated Partial thromboplastin c. 1:150 with cyanmethemoglobin reagent
time measures all factors except: d. 1:10 with cyanmethemoglobin reagent
a. Factor II and Factor V
b. Factor VII and Factor I 11. Which of the following is involved
c. Factor VII and Factor XII in primary hemostasis?
d. Factor VII and Factor IX a. Platelet receptor sites are exposed
b. Extrinsic pathway starts with the release
6. Which of the following are in vitro of tissue factor from injured blood vessel
aggregating agents? c. Generation of sufficient thrombin to
1.ADP 2.Collagen 3.Epinephrine convert fibrinogen to fibrin clot
4.Thrombin d. Intrinsic pathway is activated
a. 1 and 3
12. What is the major site of hemostatic b. IgG (50%)
protein synthesis? c. IgM
a. Liver d. IgD
b. Kidney
c. Stomach 18. Laboratory results are the following:
d. Gallbladder PT normal; APTT-abnormal; corrected by
adsorbed plasma; corrected by aged
13. Which of the following results are serum.
consistent with Disseminated a. Factor V deficiency
intravascular coagulation? b. Factor IX deficiency
1.Prolonged PT c. Factor XII deficiency
2.Normal APTT d. Factor VIII deficiency
3.Thrombin time prolonged
4.Normal platelet count (decreased) 19. Laboratory results are the following:
PT-normal; APTT-abnormal; not
a. 1,2,3 corrected by adsorbed plasma;
b. 1,2,3,4 corrected by aged serum.
c. 1 and 3 a. Factor V deficiency
d. 2 and 4 b. Factor IX deficiency
c. Factor VII deficiency
14. When performing the bleeding time d. Factor XII deficiency
Ivy method, punctures are made?
a. 2 mm apart 20. Factor VIII:
b. 1 mm apart a. Antihemophilic A factor
c. 1 cm apart b. Antihemophilic globulin
d. 5 mm apart c. Both a and b
d. None of the above
15. Philadelphia chromosome is present
in: 21. All of the following are
a. Leukemoid reaction myeloproliferative disorders except:
b. Chronic myelogenous leukemia a. Myelofibrosis
c. Aplastic anemia b. Chronic myelogenous leukemia
d. May-Hegglin anomaly c. Polycythemia
d. Sickle cell anemia
16. Dohle bodies can be seen in which of
the following? 22. The different types of leukemia are
a. Leukemoid reaction classified according to the following
b. Chronic myelogenous leukemia except:
c. Both a. Duration of untreated disease
d. Neither b. Number of white blood cells present in
the peripheral blood
17. What is the type of immunoglobulin c. Type of white blood cell involved
involved in Multiple myeloma? d. None of the above
a. IgA (20%)
23. The myeloblastic leukemias have
been separated into how many types? 28. All of the following are zymogens
a. Six types except:
b. Seven types a. Factor II
c. 3 types b. Kallikrein
d. 8 types c. Factor XII
d. Prekallikrein
24. Which of the following are
characteristics of Prolymphocytic 29. What is the graduation scale of the
leukemia: Westergren tube?
1. Marked lymphocytosis exceeding 100 X a. 200 mm
103/ul. b. 100 mm
2. Hairy cells in the blood and bone marrow c. 250 mm
3. Predominant cell type is the d. 202 mm
prolymphocyte
4. Characterized by massive splenomegaly 30. Normal value for the modified
Westergren ESR for children?
a. 1,2,3 a. 0-15mm/hour
b. 1,3,4 b. 0-10 mm/hour
c. 2,3,4 c. 0-5mm/hour
d. 1,2,3,4 d. 0-20mm/hour

25. According to the FAB classification 31. This stains mucoproteins,


of acute leukemias, Acute glycoproteins, and high molecular
megakaryocytic leukemia is classified weight carbohydrates present in all
as: blood cell types.
a. M3 a. Acid phosphatase
b. M4 b. Sudan Black B
c. M5 c. Periodic Acid Schiff
d. M7 d. Peroxidase

26. Which of the following results are 32. These are nucleated red blood cells
normal in Von Willebrand’s disease? containing siderotic granules.
a. Bleeding time a. Siderocytes
b. Platelet count b. Sideroblasts
c. Aggregation with ristocetin c. Pappenheimer bodies
d. Activated Partial thromboplastin time d. Hairy cells

27. The Mean Platelet volume in Bernard 33. There is decreased alkaline
Soulier syndrome is: phosphatase activity in:
a. Increased a. Chronic myelogenous leukemia
b. Decreased b. Leukemoid reaction
c. Both a and b c. Either
d. None of the above d. Neither
34. Peroxidase is present in the granules a. Forward light scatter
of: b. Right angle scatter
1.Neutrophils c. Cell size
2.Basophils d. Relative number of cells
3.Monocytes
4.Lymphocytes 40. In the Coulter-S-Plus series, which
parameter is calculated rather than
a. 1and 3 directly measured?
b. 2 and 4 a. RBC count
c. 1,2,3 b. Hemoglobin
d. 1,2,3,4 c. Hematocrit
d. WBC count
35. A screening procedure for detecting
metabolic defects of the neutrophil. 41. What specimen is used for the
a. Chloracetate Esterase stain autohemolysis test?
b. Nitroblue Tetrazolium Reduction Test a. Platelet rich plasma
c. Both b. Anticoagulated whole blood
d. None of the above c. Defibrinated blood
d. Platelet poor plasma
36. A patient suspected with a Filaria
infection particularly Wuchereria 42. What is the screening test employed
bancrofti should have blood collected at for PNH?
what time? a. Sucrose hemolysis test (confirmatory)
a. Noon time b. Sugar water screening test
b. At night c. Acid-Serum test (confirmatory)
c. Either d. L.E preparation
d. Neither
43. The red blood cells of patient’s with
37. Which of the following will hasten the PNH are unusually susceptible to lysis
sickling of erythrocytes? by complement. The specimen of choice
a. Sodium citrate for Acid-Serum test is defibrinated
b. Sodium metabisulfite blood.
c. Sodium chloride a. First statement is True while the
d. Potassium oxalate second statement is false.(must be
washed RBC)
38. The manual eosinophil count using b. First statement is False while the second
the hemocytometer is called? statement is True
a. Relative eosinophil count c. Both statements are true
b. Differential count d. Both statements are False
c. Direct absolute eosinophil count
d. Indirect eosinophil count 44. The property of fluid that resists the
force causing it to flow.
39. The X axis on a blood cell histogram a. Viscosity
represents? b. Turbidity
c. Fluidity b. 1 minute
d. Flexibility c. 5 minutes
d. 30 seconds
45. What is the affinity of
carboxyhemoglobin for carbon 51. Increased red blood cell count:
monoxide than oxygen? 1.Anemia
a. 100 times 2.Polycythemia vera
b. 200 times 3.Dehydration
c. 50 times 4.Altitude
d. 20 times
a. 1 and 3
46. Which of the following is not b. 1,2.3
included in complete blood count? c. 2,3,4
a. White blood cell differential d. 2 and 4
b. Platelet count
c. Hemoglobin 52. What is the volume of the large
d. Hematocrit center square of the Neubauer counting
chamber?
47. Disintegrating nucleus of a white a. 0.004 uL
blood cell. b. 0.1 uL (0.004x25)
a. Basket cell c. 0.002 uL
b. Smudge cell d. 0.001 uL
c. Ferrata cell
d. Both a and b 53. If the large center square is
divided into 25 smaller squares for RBC
48. Microhematocrit tubes hold count, what will be the volume of each 25
approximately: squares?
a. 0.5 mL of whole blood a. 0.02 uL
b. 0.05 ml of whole blood b. 0.002uL
c. 1ml of whole blood c. 0.004 uL
d. 1.5 ml of whole blood d. 0.4 uL

49. All of the following are criteria for 54. The cells are counted in consecutive
assessing cell maturity except: fields as the blood film is moved from
a. Cell size side to side.
b. Nuclear maturation a. Crenellation Technique
c. Cytoplasmic change b. Longitudinal method
d. Number of cells c. Battlement method
d. None of the above
50. In white blood cell count, after
charging the counting chamber, how 55. What is effect of increased albumin
many minutes will you wait for white concentration to the erythrocyte
blood cells to settle? sedimentation rate?
a. 3 minutes (2-3 min) a. Decreases ESR
b. Increases ESR
c. Either 61. Eosinophil granules contain:
d. Neither a. Lysozyme
b. Phagocytin
56. What is the effect of temperature to c. Phospholipase
the sedimentation rate of erythrocytes? d. Bactericidal cationic proteins
a. As temperature decreases sedimentation
rate increases 62. What is daily platelet turnover rate?
b. Temperature has no effect in the a. 35,000 platelets (+-4,300) per µL daily
sedimentation rate of erythrocytes b. 20,000 platelets (+-4,300) per µL daily
c. Sedimentation rate increases as the c. 30,000 platelets (+-3000) per µL daily
temperature increases d. 10,000 platelets (+-4,300) per µL daily
d. All of the above
63. The red cell indices show: MCV=120
57. What is the recommended External fL and MCHC= 33%. The morphology of
diameter of Westergren tube by the the red cells would be:
NCCLS? a. Hypochromia and microcytosis
a. 4.0 mm (±0.5mm) b. Normocytic, normochromic
b. 5.5 mm (±0.5mm) c. Normocytic hypochromic
c. 3.5 (±0.5mm) d. Macrocytic normochromic
d. 4.6 (±0.5mm)
64. What part of the platelet contains the
58. They are rough endoplasmic microfilaments and the microtubules?
reticulum containing RNA and may a. Sol-gel zone
represent localized failure of the b. Organelle zone
cytoplasm to mature. c. Membranous system
a. Dohle bodies d. None of the above
b. Toxic granulation
c. Atypical platelets 65. What is the main energy source of
d. Vacuolated neutrophil the platelet?
a. Protein
59. Enhance phagocytosis and increase b. Glucose
chemotaxis. c. Sucrose
a. Opsonins d. Fatty acids
b. Filaments
c. Uropod 66. The platelet is composed of ____
d. Protopod Lipid?
a. 60%
60. Highest eosinophil count occurs: b. 30%
a. Early in the morning c. 8%
b. Early afternoon d. 15%
c. At night
d. There is no diurnal variation in eosinophil 67. This condition indicates failure of the
count neutrophil nucleus to segment properly.
a. Chediak-Higashi syndrome d. None of the above
b. Pelger-Huet anomaly
c. Alder-Reilly anomaly 73. Hereditary sideroblastic anemia is
d. Auer rods inherited as:
a. Autosomal dominant
68. The first four stages of erythrocyte b. Autosomal recessive
development include all of the following c. Sex linked recessive
except: d. None of the above
a. Basophilic normoblast
b. Orthochromic nomoblast 74. Which of the following classification
c. Pronormoblast of anemia is not caused by a decreased
d. Reticulocyte or impaired production of red blood
cells?
69. What is the positive result in the a. Pure red cell aplasia
sodium metabisufite method? b. Mutiple myeloma
a. Holly-leaf shaped red blood cells c. Lymphoma
b. Sickle cells d. Abetalipoproteinemia
c. Slightly crenated red blood cells
d. Both a and b 75. If a blood sample has not been
obtained, It is advisable that a
70. In the indirect method for absolute technologist or student should not stick
eosinophil count, what component of the to a patient more than ____.
diluting fluid serves to stain the a. One time
eosinophils? b. Three times
a. Phyloxine c. Two times
b. Sodium carbonate d. It is okay to make how many punctures
c. Propylene glycol until blood sample is obtained
d. Heparin
76. This cellular element is not
71. Purple staining threadlike filaments considered as a true cell.
in the shape of a ring in the red blood a. Platelet
cell. b. Red blood cell
a. Rouleaux formation c. Basophil
b. Hemoglobin C crystals d. Eosinophil
c. Cabot rings
d. Basophilic stippling 77. Blood cell production in the bone
marrow is called:
72. Using the Mini prep Automatic blood a. Extramedullary hematopoiesis
smearing instrument, the thickness of b. Medullary hematopoiesis
the smear may be changed by adjusting c. Both
the? d. Neither
a. Pause control adjuster
b. Spreader holder 78. A red blood cell precursor with a size
c. Smear control knob of 14-20 µm in diameter, cytoplasm is
deeply basophilic and non-granular. 83. The changes in electrical resistance
Nucleus is relatively large with a N/C produced by cells as they pass through
ratio of 8:1, round or slightly oval, the sensing aperture is the principle of?
reddish purple in color, fine chromatin a. Light scattering
pattern. Usually with 1 to 2 nucleoli. b. Electronic impedance
a. Polychromatophilic normoblast c. Pattern recognition
b. Orthochromic normoblast d. Flow cytometry
c. Reticulocyte
d. Pronormoblast 84. What is the principle employed by the
fibrometer?
79. When 10 Sickle cells/ field and a. Electromechanical
basophilic stippling are seen in a b. Visual detection
peripheral blood smear, how are they c. Photo-optical
reported? d. All of the above
a. Reported as 2+
b. Reported as 3+ 85. What is the effect of an excess
c. Reported as positive only anticoagulant in the hematocrit value?
d. Reported as 4+ a. Increase
b. Decrease
80. Which of the following are reported c. Neither
as 1+ if 3-10 are seen per field? d. Either
a. Burr cells
b. Spherocytes 86. Which of the following erythrocyte
c. Acanthocytes abnormalities don’t have a central
d. Tear drop cells pallor?
a. Dacryocytes
81. Dark blue-black cytoplasmic granules b. Burr cells
in the neutrophil. Thought to be primary c. Spherocytes
granules showing increased alkaline d. Stomatocytes
phosphatase activity and found in acute
infections, drug poisoning and burns. 87. What tube is used for methemoglobin
a. Dohle bodies measurement?
b. Toxic granulation a. Violet top tube
c. Atypical platelets b. Green top tube
d. Vacuolated neutrophil c. Blue tube
d. Red top tube
82. White blood cells are counted in
consecutive fields from the tail toward 88. Which of the following will not be
the head of the smear: affected by an error in a hematocrit
a. Crenellation technique determination?
b. Battlement method a. Mean Cell Volume
c. Longitudinal method b. Mean Cell Hemoglobin
d. Cross sectional method c. Both
d. Neither
c. Ovalocytes
89. Using manual technique, what test is d. Pyknocytes
the most reproducible?
a. RBC count 95. All are cytoplasmic changes of white
b. WBC count blood cells except:
c. Hematocrit determination a. Hypersegmented cells
d. Hemoglobin determination b. Amato bodies
c. Giant neutrophils
90. The computation of MCHC requires? d. Smudge cells
a. Hemoglobin and RBC
b. Hemoglobin and Hematocrit 96. All are sickling hemoglobin except:
c. Red blood cell count and hematocrit a. Hemoglobin A
b. Hemoglobin C harlem
91. Bite cells are suggestive of? c. Hemoglobin S travis
a. Heinz body d. Hemoglobin C Ziguinchor
b. Amato bodies
c. Dohle bodies 97. A patient has a hemoglobin of 13.5
d. Basophilic stippling g/dL, what is the expected hematocrit of
the patient?
92. What is the corrected reticulocyte a. 40.5%
count of a patient if the hematocrit is b. 38%
30% (0.30L/L) and a reticulocyte count of c. 35%
3%? d. 50%
a. 1.8%
b. 2% 98. What is the characteristic abnormal
c. 3% morphology in myelofibrosis?
d. 4% a. Target cells
b. Tear drop cells
93. LE formation depends on the c. Ovalocytes
following factors: d. Schistocytes
1.LE factor
2.Extruded nuclei 99. Screening test for the presence of
3.Phagocytic leukocytes Hemoglobin S, where opacity of the
4.Red cells solution is the positive result.
a. Schilling’s test
a. 1,2,3,4 b. Dithionite solubility test
b. 1,2,3 c. Kelihauer Betke Test
c. 2 and 4 d. All of the above
d. 1 and 3
100. Thrombocytosis is observed in:
94. The presence of these cells indicate a. Bernard –Soulier syndrome
permanent red cell damage: b. May-Hegglin anomaly
a. Sickle cells c. Wiskott-Aldrich syndrome
b. Acanthocytes d. Polycythemia vera
Page 1 of 1
Never submit passwords through Google
Forms.
This form was created inside of University of
Baguio. Report Abuse
Google Forms

You might also like